You are on page 1of 200

From the founder’s desk,

Greetings!
“The purpose of education is to make the learner confident of
oneself.

What the world needs is more relatable, enjoyable way of learning that
would give the learner hope and confidence. What the world needs is
for the child to have faith in its ability.

At First Bench, thankfully, we have devised solutions that blend


academic focus and self-belief.

We start with fundamentals. We make sure that the child enjoys the
process of learning and able to understand the progress he/she is
making. More importantly, we ensure that this results in an enduring
and continual learning experience.

At First Bench, we work for the success of our students in exams


and in life. And that’s where we are different from others.”

Best Regards,

Salai Arjun
Learning Checklist
Tell students what they will be learning about in
the section.

Class Room Activity


Each spread has interesting
science activities to help
students explore the subject.

Did you know...?


Contains fascinating extra facts and
information about the subject.

Quick Response Code


It gives students in the virtual and immediate

FEATURES
access to learning materials through mobile
devices.
ASK is a discussion forum where students
can leave and expect to see responses to
messages you have left.

Brain Power On
It makes up an integral part of
recreational mathematics.

Mind Map
Mind Map Students can visually organize information.
It is hierarchical and shows relationship
among piecesof the whole.

Recapitulate
Students can gain intimate understating of
previously learnt chapters in a short glance.
What's inside?
Section I: MATH
Number System - IV ...... 10
• Recapitulate
• Irrational numbers
• The wheel of theodorus
• Real numbers-properties
• Comparison between rational and irrational Numbers
• Surds and its types
• Rationalization

Algebra - IV (polynomials) ...... 40


• Recapitulate
• Division of a polynomial by a polynomial
• Remainder Theorem
• Factor Theorem
• Factorisation
• HCF and LCM of polynomials
• Square roots of algebraic expressions

Basics of trigonometry ...... 75


• Right angle Triangle
• Trigonometric ratios
• Angle of elevation and depression
• Standard angles
• Trigonometric ratios of complementary angles

Section II: PHYSICS


Motion in a straight line ...... 106
• Some basic aspects of motion
• Speed, velocity and acceleration
• Graphical representation of motion
• Equations of uniformly accelerated motion
• Uniform circular motion
Section III: CHEMISTRY
Matter and its surroundings ...... 142
• Matter and its classification
• Kinetic theory of matter
• Properties of gases
• Gases- Ideal gas & Real gas
• Properties of Liquids
• Properties of Solids
• Phase changes

Section IV: BIOLOGY


Cell - The fundamental unit of life ...... 170
• Discoveries in cell biology
• Cell size, shape and Number
• Plant and animal cell
• Structural organisation of a cell

Section V: LOGICAL REASONING


Analogy ...... 202
Artificial language ...... 208

Section VI: SOLUTION


Math ...... 214
Physics ...... 224
Chemistry ...... 225
Biology ...... 226
Reasoning ...... 227
x
y

r=d/2

Scope
The scope of mathematics have no boundaries.This subject
teach us to be a good problem solver. There is a demand of math
students in fields such as statistics, engineering, physical science,
computer science, insurance, economics, astronomy, banking and
accountancy. There is also a wide scope in teaching and research
fields.
Chapter 1 Learning Checklist
Recapitulate
Irrational numbers
NUMBER The wheel of theodorus
Real numbers-properties
SYSTEM - IV Comparison between rational and irrational
Numbers
Surds and its types
Rationalization

Recapitulate
Number system
The number system consists of natural numbers, whole numbers, integers, rational numbers and irrational numbers (real
numbers)etc.

Natural numbers
The numbers used for counting objects are called natural numbers. These are 1,2,3,4,5,6…. and so on.
The set of natural numbers is denoted by N.
Thus, N={1,2,3,4,5,6…..}
Since there cannot be a last counting number, therefore, there is no largest natural number. The set of natural numbers
is an infinite set.

Whole numbers
The natural numbers together with zero are called the whole numbers. The set of whole numbers is denoted by W.
Thus, W={0,1,2,3,4,5,6….}

Integers
All natural numbers, 0 and negatives of natural numbers together are known as integers. The set of integers is denoted
by I or Z = {…..-5, -4, -3, -2, -1, 0, 1, 2, 3, 4….}

Definition of Rational Numbers


A number is said to be rational if it can be written in the form of a fraction such as p/q where both p (numerator) and q
(denominator) are integers and denominator is a a non-zero number. Integers, fractions including mixed fraction, recurring
decimals, finite decimals, etc., are all rational numbers.

Examples of Rational Number


• 1/9 – Both numerator and denominator are integers.
• 7 – Can be expressed as 7/1, wherein 7 is the quotient of integers 7 and 1.
• √16 – As the square root can be simplified to 4, which is the quotient of fraction 4/1
• 0.5 – Can be written as 5/10 or 1/2 and all terminating decimals are rational.
• 0.3333333333 – All recurring decimals are rational.

10 CLASS - IX
NUMBER SYSTEM - IV

Definition of Irrational Numbers


A number is said to be irrational when it cannot be simplified to any fraction of an integer (x) and a natural number (y).
It can also be understood as a number which is irrational. The decimal expansion of the irrational number is neither finite
nor recurring. It includes special numbers like π (‘pi’ is the most common irrational number).
Examples of Irrational Number
• √2 – √2 cannot be simplified and so, it is irrational.
• √7/5 – The given number is a fraction, but it is not the only criteria to be called as the rational number. Both
numerator and denominator need to integers and √7 is not an integer. Hence, the given number is irrational.
• 3/0 – Fraction with denominator zero, is irrational.
• π – As the decimal value of π is never-ending, never-repeating and never shows any pattern. Therefore, the value of
pi is not exactly equal to any fraction. The number 22/7 is just and approximation.
• 0.3131131113 – The decimals are neither terminating nor recurring. So it cannot be expressed as a quotient of a
fraction.

Properties of Rational Numbers


Property 1:
a a ´m
If a/b is a rational number and m is a nonzero integer, then=
b b´ m
In other words, a rational number remains unchanged, if we multiply its numerator and denominator by the same non-
zero integer.
For examples:
-2 (-2) ´ 2 -4 (-2) ´ 3 -6 (-2) ´ 4 -8
= = , = , = and soon......
5 5´ 2 10 5 ´ 3 15 5 ´ 4 20

Property 2:
a a a÷m
If is a rational number and m is a common divisor of a and b, then =
b b b÷m

In other words, if we divide the numerator and denominator of a rational number by a common divisor of both, the
rational number remains unchanged.
For examples:
-32 -32 ÷ 8 -4
= =
40 40 ÷ 8 5

Property 3:
a c
Let and be two rational numbers.
b d
a c a ´d
Then = ⇔
b d b´c
a c

b d

a×d=b×c
MATH 11
For examples:

2 2 4
If and 4 are the two rational numbers then, = Û (2´ 6) = (3 ´ 4)
3 6 3 6
Property 4:
For each rational number m, exactly one of the following is true:
(i) m > 0 (ii) m = 0 (iii) m < 0
For examples:
2
The rational number is greater than 0.
3
0
The rational number is equal to 0.
3
The rational number -2 is less than 0.
3
Property 5:
For any two rational numbers a and b, exactly one of the following is true:
(i) a > b (ii) a = b (iii) a < b

For examples:
1 1
If 1 and are the two rational numbers then, 1 is greater than .
3 5 3 5
2 6 2 6
If and are the two rational numbers then, is equal to .
3 9 3 9
3 3
If -2 and are the two rational numbers then, -2 is less than .
7 8 7 8
Property 6:
If a, b and c be rational numbers such that a > b and b > c, then a > c.
For examples:
-8 17 17 -8
If 3 , 17 and are the three rational numbers where 3 is greater than and is greater than then 3 is
5 30 15 5 30 30 15 5
-8
also greater than
15
Denseness property of rational numbers
Rational numbers are different from integers in many ways. For example. on a number line there is not always another
integer between any two given integers

0 1 2 3 4 5
There are exactly four integers between 0 and 5. viz.1,2,3 and 4. there are no integers between 0 and 1. Between any
1 3
two rational numbers however it is always possible to find another rational number. For example, consider and you
2 4
1 3 1 3
can find the rational number that is one half of the way from and by adding one half the difference between and
2 4 2 4
to 1
2

12 CLASS - IX
NUMBER SYSTEM - IV

1 1 æç 3 1 ö÷ 1 1 æç 1 ö÷ 1 1 5
+ ç - ÷= + ç ÷= + = 1 5 3
2 2 çè 4 2 ÷ø 2 2 çè 4 ÷ø 2 8 8 2 8 4

To find many rational numbers between two given distinct rational numbers:
a c a+c a c
If and are two distinct rational numbers, then is also a rational number lying between and .Therefore,
b d b+d b d
a rational number lying between two distinct given rational numbers is easily obtained by adding the numerators and
denominators.For example.
1 1 1+1 2
Rational number lying between and = =
3 2 3+2 5
1 2 1+ 2 3
Rational number lying between and = =
3 5 3+5 8
2 1 2 +1 3
Rational number lying between and = =
5 2 5+2 7

Decimal representation of rational numbers:


Every rational number can be named by either a terminating decimal or a repeating decimal. All terminating and
repeating decimals are called rational numbers. Decimals that do not terminate and are non-repeating are called irrational
numbers.
A fraction in simplest form that has a denominator having the prime factors only 2, only 5, or both 2 and
5 can be represented by a terminating decimal.
Example 01: Which of the following rational numbers can be represented as terminating decimals?
3 -27 2
(i) (ii) (iii)
5 40 13
Solution:
3 3
(i) In , 5 is a factor of the denominator 5. ∴ has terminating decimal representation.
5 5
-27 27
(ii) In , 40=23 × 5 , i.e., it has factors in 2s and 5s. ∴ has terminating decimal representation.
40 40
2 2
(iii) In , the denominator 13 does not have any factor in 2s or 5s. ∴ does not have terminating decimal
13 13
representation.

Example 02: Find out 100 rational numbers lying between -9/19 and 5/19.

Solution:
-9 10 -90 5 10 50
We have, = -9 × × 10 = and, =5× × 10 =
19 19 190 19 19 190
We know that
-90 < -89 < -88 < -87 < -86 < -85 < …….. < -25 < -24 < -23 < -22 < …….. < -1 < 0 < 1 < 2 < …….. < 9 < 10
-90 -89 -88 -25 -24 -23 -1 0 1 9 10
< < < …….. < < < < …….. < < < < …….. < <
190 190 190 190 190 190 190 190 190 190 190
-89 -88 -25 -24 -23 -1 0 1 9 10
Hence, < < < …….. < < < < …….. < < < < …….. < <
190 190 190 190 190 190 190 190 190 190
-9 -90 5 50
are the 100 rational numbers between = and = .
19 190 19 190

MATH 13
RECAP WORKOUTS
1. Which of the two rational numbers in each of the following pairs of rational numbers is greater?
3 -3 -7 5
(i) or 0 (ii) or 0 (iii) or
8 8 12 -8

(iv) 2 or 3 (v) 4 or -3
3 4 -9 -7
2. Which of the two rational numbers in each of the following pairs of rational numbers is smaller?
-6 7 -4 8
(i) -4 or -8 (ii) or (iii) or
3 7 -13 13 3 -7
9 7
(iv) or
-13 -12
3. Arrange the following rational number ascending order:
2 5 -4 1 4 -5 7 -2
(i) , , , (ii) , , ,
3 7 -9 4 -9 12 -18 3
4.Arrange the following rational number descending order:
-13 8 1 -3 17 -8 7
(i) -2, , , (ii) , , ,
6 -3 3 -5 30 15 -10

5. Determine If the number is rational (R) or Irrational (IR)


(i) 61π (ii) 42 (iii) 75.082106 (iv) 101
20
(v) 65.427 (vi) (vii) π (viii) π
6
98
(ix) 5.6213 (x) (xi) 39 (xii) 89.396668...
16
(xiii) 17 (xiv) 67.714813.

6. Find out a rational numbers lying between the


1 1 -1 1
(i) and (ii) 2 and 3 (iii) and
4 3 3 2
4 3
(iv) -3 and -2. (v) and
8 8
7. Represent the following rational numbers on number line
4 2 7
(i) (ii) (iii)
5 3 3
-3 -1
(iv) (v)
4 3
æ -14)ö÷ æ 2 6 ö÷ (-14)
8. Verify that 2 ´çç 6 ´ ( ÷÷ = çç ´ ÷´
3 ççè 7 15 ÷ø çè 3 7 ÷ø 15
-5 13
9. What number should be added to to get ?
6 15
-1 1
10. Find out a rational number lying between and ?
3 2
14 CLASS - IX
NUMBER SYSTEM - IV

Note Representation of Irrational Numbers on The Number


.............................................. Line
Except zero every rational number is Before going to the topic, let’s understand a simple concept of Pythagoras
either positive or negative. Theorem, which states that:
Every pair of rational numbers can “If ABC is a right angled triangle, right angled at B with AB, BC and AC as the
be compared. perpendicular, base and hypotenuse of the triangle respectively with AB = x units
and BC = y units.Then, the hypotenuse of the triangle, AC is given by x 2 + y 2 .
A

√ x2 + y2
x

The Mathematician And Cult


C y B
Leader
Pythagoras was an Ancient Now let’s get back to the original topic ,
Greek mathematician and philosopher. To have better understanding of the concept lets take an example of
He was born on the Greek 570 BC and representation of square root of 2 (2–√2) on the number line. For the representation
died in Greece probably around 495 following steps must be followed:
BC. In 530 BC he moved to Italy and
established a religious group known Step I: Draw a number line and mark the centre point as zero.
as the Pythagoreans. He believed that Step II: Mark right side of the zero as (1) and the left side as (-1).
science and religion were connected.
Pythagoras’s followers really believed
-1 0 1
that he was a demigod. They called him
“the divine Pythagoras” .Pythagoras Step III: We won’t be considering (-1) for our purpose.
told his followers that he had already
Step IV: With same length as between 0 and 1, draw a line perpendicular to
lived four lives, all of which he could
point (1), such that new line has a length of 1 unit.
remember.
Step V: Now join the point (0) and the end of new line of unit length.
Silence, Pythagoras believed,
was very important. Staying quiet Step VI: A right angled triangle is constructed.
was a way to learn self-control, and so Step VII: Now let us name the triangle as ABC such that AB is the height
he made sure that anyone who wanted (perpendicular), BC is the base of triangle and AC is the hypotenuse of
to join his cult could do it. Anyone who the right angled triangle ABC, right angled at B.
signed up, had to close his mouth and
keep it shut for five years straight. Step VIII: 
Now length of hypotenuse, i.e, AC can be found by applying
pythagoras theorem to the triangle ABC.
Pythagoras had a rare (A)
condition known as synesthesia. A
person with the condition is said to 1 unit
be able to hear colors and see music, or
associate smells with people’s names. -1 0 1 unit 1
(C) (B)

AC2 = AB2 + BC2


AC2 = 12 + 12 A
2 1 unit
AC2 = 2
AC = 2 0 1 unit 1
-1
(C) (B)
MATH 15
Step IX: Now with AC as radius and C as the centre cut an arc on the same
number line and name the point as D. Brain Power On
.............................................
Step X: Since AC is the radius of the arc and hence, CD will also be the radius In the following problem, replace
of the arc whose length is 2 the letters of the English alphabet
Step XI: Hence, D is the representation of 2 on the number line. by digits (two or more letters may
have the same value) to complete the
A
procedure of division.
2 1 5C 1FG
9 4 AB AB 4CDE
-1 0 1 1 2 - DE -28
(C) (B) (D) H56
3F
- GH -IJK
Applications of irrational numbers:
0 LMN
(i) In compound interest: Let us have a look at the following example to -PQR
understand how irrational number helps us in case of calculating compound 0
interest:
For Example: .......................................................
.......................................................
An amount of Rs. 2,00,000 is given to Anand by his friend for a tenure of 2 .......................................................
years at interest of 2% per annum compounded annually. Calculate the amount .......................................................
which Anand needs to return his friend after 2 years. .......................................................
Solution: .......................................................

Principal = Rs 2,00,000
Time = 2 years
Interest rate (r) = 2% p.a.
t
æ r ö÷
Amount =pçç1+
çè 100 ÷÷ø
2
æ 2 ö÷ There is a connection between Pi and
So, amount = 2,00,000 çç1+
çè 100 ÷÷ø gravity !
æ 102 ö÷
2
Get out your calculator. Use 9.8 m/s2
= 2,00,000 çç
çè 100 ÷÷ø for the local gravitational constant (g).
10, 404 Now try this:
= 2,00,000 ´
10, 000 9.8 = 3.1305
= 2, 08,0080 That's pretty close to the accepted value
of Pi—and it's not a coincidence. Here
Hence, the amount that Anand needs to return to his friend is Rs. 2,08,080.
is a relationship between period and
So, compound interest is one of the applications of irrational numbers where length for a pendulum (with a small
we use sum of infinite series. oscillation amplitude):
(ii). Finding area or perimeter (circumference) of any circular part: L
We know that area and circumference of a circular part is given by πr2 and 2πr T = 2p
g
respectively, where ‘r’ is the radius of the circle and ‘pi’ is the irrational we use in
finding area and circumference of the circle whose value is 3.14 (approx.). Put in 1 meter for the length and 2
(iii). Use of cube root: Cube roots are basically used in finding area and seconds for the period and there is your
perimeter of three dimensional structures such as cubes and cuboids. connection.

16 CLASS - IX
NUMBER SYSTEM - IV

(iv). Used to find gravity equation: Equation for acceleration of gravity is


Brain Power On Gm
............................................. given by g = 2
r
Using the Pythagorean Theorem where g = acceleration due to gravity
and Approximating non-perfect m = mass of the object
Square Roots construct wheel of r = radius of earth
theodorus. G = gravitational constant
Here ‘G’ is the irrational number whose value is 6.67 × 10−11.
Similarly, there are many such examples where we use irrational numbers.
The wheel of theodorus
Theodorus of Cyrene, who lived around 425 B.C., was a philosopher of
ancient Greece. It is said that he discovered the construction below, which is
therefore called the ‘the wheel of Theodorus’.
1
1
1
4=2
1. Use any given length (less than 2 1 5 3
inches) for your first triangle in the
2 1
center and list it as 1 unit. 1 6
1=1
2. Determine the hypotenuse for the
triangle. That hypotenuse becomes Example 01:
the new leg for your second triangle, Placement of 2 , 3 , 5 on the number line
and so on. The other leg will always On the number line we construct 2 , 3 , 5 as follows:
be 1 unit.
Y
3. If the hypotenuse is a non A
C E
1 T 5
perfect square, you must show each 2 3
1 1
approximation without a calculator. X'
1

O 1 L B D M F N
4. Continue making at least 25 1 2 3 X

triangles, attempting to keep them


2 3 5
right triangles throughout.
On the number line X’OX let O represent zero, L represent 1 unit, M represents
5. When you get to the stage where 2 units, and N represent 3 units. Thus, OL = 1, OM = 2, ON = 3.
your right triangles will overlap
Let OY be perpendicular to OX at O. Let OT = 1 unit, LA is perpendicular to
previous right triangles, draw your
OL and LA = 1 unit. Join OA.
hypotenuse towards the center of
the spiral but do not mark over the By Pythagoras Theorem, OA = OL2 + LA 2 = 1 + 1 = 2 with O as centre and
previous drawing. OA as radius draw on are cutting the number line OX at B.Thus, OB = OA = 2 .
6. Decorate your wheel in a way that At B draw the perpendicular BC equal to 1 unit. Join OC.
demonstrates this spiral in the real
( 2)
2
By Pythagoras Theoram, OC = OB2 + BC2 = + 12 = 2 + 1 = 3 .
world. Here is the one done for you!
With O as centre and OC as radius draw an are cutting the number line OX at
D. Thus, OD = OC = 3.
At M draw the perpendicular ME equal to 1 unit.
Join OE. By Pythagoras Theorem, OE = OM 2 + ME2 = 22 + 12 = 4 + 1 = 5
With O as centre and OE as radius draw an are cutting the number line OX at
F. Then OF = OE 5 .
Hence, OB = 2 , OD = 3 , OF = 5.
MATH 17
The real numbers
If we combine the rational numbers and the irrational numbers, we get real
numbers which we denote by R. The following diagram shows the relationships
among the different kinds of numbers of the real number system.
The Fibonacci Sequence
Real Number (R) The Fibonacci Sequence begins with a
1, followed by another 1. Later terms
Rational numbers (Q) Irrational numbers (P) are found by adding together the two
p previous terms.
;p,q Î Integers, q ¹ 0 2 , 3 , 5 , etc
q
an = an-1 + an-2 for a1=1 and a2 = 1
1, 1, 2, 3, 5, 8, 13, 21, 34, 55, 89,
Integers (I) Non-integral rational numbers
144,......
...-2,-1,0,1,2... -7 -1 3 9
, , , ,etc The Fibonacci numbers are a complete
5 2 4 5
sequence. This means that any positive
integer can be expressed as the sum of
Negative integers Whole numbers(W)
0,1,2,3, etc various Fibonacci numbers, without
...-4,-3,-2,-1
repeating any of the Fibonacci numbers.
When examining the Fibonacci
Zero Natural numbers(N) sequence, it is interesting to note:
1,2,3,4, etc
• Every third term is even.
Also, Real numbers • Every fourth term is a multiple of 3.
• Every fifth therm is a multiple of 5
Rational numbers(Q) Irrational numbers Applications to Art and Architecture
All terminating or recurring All non-terminating or
The Fibonacci spiral is can be used as
decimals non-recurring decimals
a guide to placing features in art and
Properties of the real number system architecture, in order to create a pleasing
All the properties of the system of rational numbers may be proved to extend visual effect.
to the system of real numbers. These may be summarized as under.
Property Addition Multiplication
1. Closure a + b is a real number a × b is a real number.
2. Commutative a + b = b + a a×b=b×a
3. Associative (a + b) + c = a + (b + c) (a × b) × c = a × (b × c)
For each a, a × 1 = 1 × a = a
For each a, a + 0 = 0 + a = a i.e.,
i.e., ‘one’ is the identity
4. Identity 'zero' is the identity element
element for the operation of
for the operation of addition.
multiplication.
For each 'a', there is For the operation of
a unique real number multiplication. For each 'a', Applications to Biology
(-a), called the additive 1
there is a real number , called The lengths of the bones in the human
5. Inverse inverse of a, such that a a finger are proportionate to Fibonacci
+ (-a) = (-a) + a = 0 the multiplicative inverse of 'a',
1 1 numbers.
such that a ´ = ´ a = 1
a a
a × (b + c) = (a × b) + (a × c) Left distributive property
6. Distributive
(b + c) × a = ab + ac Right distributive property

18 CLASS - IX
NUMBER SYSTEM - IV

Note 7. Order: The real number system is ordered, i.e., if a and b are different real
.............................................. numbers. Then either a < b or a > b.
1. The sum of an irrational number 8. Density : The real number system is dense, that is, between any two distinct
and a rational number gives an real numbers there is always another real number. Consequently, between any
irrational number. two numbers we can find as many real numbers as we wish. The ninth property
2. Multiplying an irrational number of the real number system is one which is not shared by the rationals.
with any non-zero rational
9. Completeness: The real number system is complete. We went on filling
number gives an irrational number
more and more points of the number line by repeated extensions of the number
3. The lowest common multiple
system. The process is complete with the creation of real numbers and no more
(LCM) of two irrational numbers
gaps are left.
may or may not exists.
4. The sum or product of two For each real number there is one and only one point on a number line.
irrational numbers may be For each point on a number line there is one and only one real number.
rational therefore unlike the set In other words, there is a one-to-one correspondence between the set of real
of rational numbers, the set of numbers and the set of points on a number line. The number line is complete.
irrational numbers is not closed Together, the two statements above are called the completeness property of
under addition & multiplication the set of real numbers.

0 1 2
Because of the completeness property, we can shade parts of numberline to
represent the fact that every point in say the shaded part is the graph of a real
number. It would be inappropriate to suggest, for example, that the graph shown
here is the graph of a set of rational numbers, because ther are infinitely many
points in the shaded portion that do not have rational coordianates.
Numbers such as
Example 02 : (a) Examine whether 2 is rational or irrational.
-2 , -10 and -17 are called unreal
4

or imaginary numbers and cannot be (b) Show that 2 is not a rational number.
graphed on a number line (that is, their Solution:
values cannont be ordered). Imaginary 1.4 1 4 2 1 3 5
numbers were first noticed by Hero of 1 2.00 00 00 00 00 00 00 ...
Alexandria in the 1st century. In 1545, 1
the Italian mathematician Girolamo
24 1 00
Cardano wrote about them, but believed
96
negative numbers did not have a square
root. Imaginary numbers were largely 281 4 00
2 81
ignored until the 18th ccentury when
they were studied by Leonhard Euler 2824 1 19 00
and the Carl Friedrich Gauss. 1 12 06
28282 6 04 00
−1 is defined to be the imaginary
number i, so −1 = i. 5 65 64
282841 38 36 00
∴ −36 = 36 ×(−1)
28 28 41
= 36 × −1 = 6i 2828423 10 07 59 00
Imaginary numbers are useful for 8 48 52 69
solving physics and engineering 28284265 1 59 06 31 00
problems involving heat conduction, 1 41 42 13 25
elasticity, hydrodynamics and the flow 17 64 17 75
of electric current.
2 = 1.4142135….

MATH 19
We observe from the above that 2 is non-terminating and non-
repeating. Hence, 2 is not a rational number.

(b) To prove that 2 is not a rational number, we will show by contradiction


method that it is not a rational number. In this method, we assume that the
given number is a rational number and then establish that our supposition is
wrong.

Let 2 be a rational number.


p
∴ = 2 , p and q are integers that have no common factor and q ≠ 0.
q
Squaring both sides, we get
p2
2
= 2 Þ p2 = 2q 2 Þ p2 is an even number
q
⇒ p is also even number ...(1)

Let p = 2r. r ∈ N ⇒ p2 = 4r 2 ...(2)

By substituting 2q2 for p2 in (2), we get 2q2 = 4r2 ⇒ q = 2r2

q is also an even number ...(3)


From (1) and (3), we arrive at the result that p and q are both even numbers.
p and q are not co-primes. Therefore, our supposition is wrong.
2 is not a rational number.
It is an irrational number.

Example 03 :
The sum and product of two rational numbers are always rational, but neither Brain Power On
.............................................
the sum nor the product of two irrational numbers is always an irrational number.
Minesweeper puzzle:
Solution: In the magic box there are 10
hidden mines. Around the numbered
Let us consider two irrational numbers -5 + 7 and 2 - 7 . Their
square the total number of mines
sum = -5 + 7 + 2 - 7 = -3 , which is not an irrational number.
hidden in the 8 squares, indicates the
Now consider the irrational numbers 7 and 28 . Their product numbers in various squares. Now
= 7 ´ 28 = 7´ 28 = 196 = 14 , which is not an irrational number. you have to find 10 mines.
Hence, the set of irrational numbers is not closed under (i) addition,
Note that: The numbered squares do
(ii) multiplication.
not have any mines
Example 04: 2 0 1
You have seen that 2 is not a rational number. Show that 2 + 2 is not a 3 1 2
rational number. 1 0 1
2 1 0
Solution: 1 1
2 is a rational number and 2 is an irrational number.We know that the sum 1 1 1 1
of a non-zero rational number and an irrational number is an irrational number. 1 1 1 2 2
∴2+ 1 1
2 is an irrational number, i.e., it is not rational.

20 CLASS - IX
NUMBER SYSTEM - IV

Note Example 05:


.............................................. Prove that 3 3 is not a rational number.
Other irrational numbers. Solution:
The cube root of 8 is denoted by
3 is a rational number and 3 is an irrational number. We know that the
3
8 = 3 2´2´2 = 2 . product of a non-zero rational number and an irrational number is an irrational
The fourth root of 16 is denoted by number.
4
16 = 4 2 ´ 2 ´ 2 ´ 2 = 2 . ∴ 3 3 is an irrational number, i.e., it is not rational.
The fifth root of 32 is denoted by
5
32 = 5 2 ´ 2 ´ 2 ´ 2 ´ 2 = 2 Example 06:
Similarly, we have other roots. In Show that 3
6 , 4 3 , 5 5 are not rational numbers.
general, n a denotes the nth root
of a. Solution:
3
p æ p ö÷
Also, 3 4 is not a rational number. (i) Suppose to the contrary, that there is a rational number such that çç ÷ = 6.
q çè q ÷÷ø
It is an irrational numbers. Likewise,
p
we may have irrational numbers like Since 13 = 1, and 23 = 8, it follows that 1 < < 2.
q
4
9 , 5 64 , 6 2 , and so on. p
Then q > 1; if q = 1, will be an integer, and there is no integer between 1 and 2.
The symbols , 3 , 4 , 5 ....... n a .... q
are radical signs and numbers like We can also suppose that p and q have no common factors, because if they
3 , 3 3 , 4 5, 5 6.... are irrational had any common factor, that would cancel out and the new numerator and
numbers. These are also known as denominator would have no common factor.
radicals. p3 p3
Now, 6 = 3 . Multiplying both sides by q2 we get 6q 2 =
q q
Order of an irrational number. If
n
a is an irrational number, where a q being an integer 6q is an integer, and since q > 1 and q does not have a
2

p3
is a positive rational number and n is common factor with p and consequently with p3. So is a fraction different
q
a positive integer, then the radical or p 3
1 from an integer. Thus 6q 2 ¹ . This contradiction proves the result.
n
irrational number a or a n is ‘said’ q
to be of order n and a is called the
radicand. Finding irrational numbers between two given irrational numbers
Example 07. Find two irrational numbers between
(i) 3 and 5 (ii) 2 and 2.5

Solution:

(i) Irrational number between 3 and 5 = 3´ 5 = 15


Similarly, irrational number between 3 and 15 = 3 15 .
Two irrational numbers between 3 and 5 are 15 and 3 15

Note (ii) Irrational number between 2 and 2.5 = 2´ 2.5 = 5


.............................................. 2 < 5 < 2.5
Given two distinct positive rational
Irrational number between 2 and 5 = 2 ´ 5 = 21/ 2.51/ 4
numbers a and b such that ab is not
a perfect square, then the irrational Hence, 2 < 21/ 2.51/ 4 < 5 < 2.5
number ab lies between a and b. Two irrational numbers between 2 and 2.5 are 2 , 4 5 and 5

MATH 21
Example 08. Insert a rational number and an irrational number between 2 and 9.
Brain Power On
Solution: .............................................
Since, 2 and 9 are positive rational numbers and 2 × 9 = 18 is not a perfect Patterns in Pascal's Triangle
square, therefore, This is Pascal's triangle but you
2+9
(i) a rational number between 2 and 9 = = 5.5 need to fill it in. Down the left and
2 right sides, the blocks will have
(ii) an irrational number between 2 and 9 = 2 × 9 = 2 × 9 = 3 2 number 1's. Then, to fill in the other
space, add the two numbers right
Example 09. Insert two rational numbers and two irrational numbers between
above it (slightly to the right and to
2 and 8 . the left). For example, the colored
Solution: ( 2 )2 = 2 and ( 8 )2 = 8 number 3 comes from adding the 1
and 2 above it.
(i) To find two rational numbers between 2 and 8 , choose any two 1
numbers between 2 and 8 which are perfect squares. The square roots of 1 1
such numbers will be required rational numbers. 1 2 1
1 3 1
Let the numbers be 4 and 6.25. 1

Since, 4 = 2 and 6.25 = 2.5, therefore, required rational numbers are


2 and 2.5.
(ii) To find two irrational numbers between 2 and 8 , choose any two
rational numbers between 2 and 8 which are not perfect squares. The Now answer to the following
square root of such numbers will be the required irrational numbers. Let questions
the numbers be 3 and 5. Then 3 and 5 are the required irrational
numbers. 1.What do you notice about the row
sums?
Comparison between rational and irrational numbers
.......................................................
(i) if a and b are irrational numbers, then
2. Can you find a diagonal with the
a > b if a > b numbers 1, 2, 3, 4, 5, 6, 7?
a < b if a < b .......................................................
a = b if a = b 3.Triangular numbers start like this:
(ii) If a is a positive rational number and b is an irrational number, then
a > b if a 2 > b
1 3 6 10
a < b if a 2 < b
a. Continue the pattern of triangular
a = b if a 2 = b numbers.

Example 10: Compare the values of 5 , 3 9 and 6


7. b. Find a diagonal with triangular
numbers in Pascal's triangle.
Solution. The order of the given irrational numbers are 2,3 and 6 respectively.
c. Can you find something special
L.C.M. of 2,3,6=6
1 3 about the triangular numbers?
∴ 5 = 5 2 = 5 6 = 6 53 = 6 125
1 2
3
9 = (9) 3 = 9 6 = 6 92 = 6 81 Note
..............................................
Since 125 > 81 > 7, therefore
6
125 > 6 81 > 6 7 , i.e., 5 > 3 9 > 6 7 .
If a + b = c + d , where a and
(iii) If a,b,c,d are rational numbers and n is an irrational number, then c are rational and b and d are
a + b n =c+d n , if and only if a = c and b = d. irrational, then a = c and b = d.

22 CLASS - IX
NUMBER SYSTEM - IV

Surds
Can numbers like 2,3,5,7,…. be written as the squares of any rational numbers?
The answer is obviously ‘No’. Can 4 be written as the square of a number? The
The term surd traces answer is ’Yes’. We have (2)2 = 4.
back to the Arab mathematician al-
Can 4 be written as the cube of any rational number? The answer is obviously
Khwarizmi (about 825 AD) who
‘No’. Can 8 be written as the cube of any rational number. The answer is ‘Yes’.
referred to rational and irrational
We have (2)3 = 8.
numbers as ‘audible’ and ‘inaudible’
respetively. This eventually led to Irrational numbers of the type 2 , 3 , 5 , 17 ,...... which are square roots of
the Arabic asamm (deaf, dumb) for positive rational numbers that cannot be expressed as the squares of any
irrational numbers being translated as rational numbers are called surds.
surdus (deaf, mute) in Latin. Similarly, 3 2 , 3 4 , 3 19 , etc., are numbers which are the cube roots of positive
rational numbers that cannot be expressed as the cube of any rational numbers
and so are surds.
In general, if x is a positive rational number and n is a positive integer,
such that x l/n , i.e., n x is irrational, then n x is called a surd or a radical.
Example 11: State with reason which of the following are surds and which are
not:
(i) 98 (ii) 7 ´ 28
(iii) 3 2 ´ 3 32 (iv) 15 20 ÷ 4 45
(v) 27 7 ÷ 15 21
Solution: (i)  98 = 7´ 7´ 2 = 7 2 , which is not a rational number. 98 is
a surd.
(ii)  7 ´ 28 = 7 ´ 7´ 4 = 7´ 7´ 4 = 7´ 2 = 14 , which is a rational
number.
∴ 7 ´ 28 is not a surd.
(iii) 3 2 ´ 3 32 = 3 2´ 32 = 3 64 = 4, which is a rational number.
3
2 ´ 3 32 is not a surd.
15 20 15 5 ´ 4 15 4
(iv) 15 20 ÷ 4 45 = = . = .
4 45 4 5´9 4 9
15 2 5
= . = , which is a rational number.
4 3 2
∴ 15 20 ÷ 4 45 is not a surd.

Note 20 21 4 3
(v) 20 21 ÷ 15 7 = . = which is not a rational number.
.............................................. 15 7 3
Every surd is an irrational number ∴ 20 21 ÷ 15 7 is a surd.
but every irrational number is not
a surd. Types of Surds
For Example: 3 3 + 2 is an 1.Unit surds and multiples of surds:
irrational number, it is not a surd If n a is surd, it is also referred to as a unit surd. If k is a rational number, k n a
because 3 + 2 is not a rational is a multiple of a surd.
number. All multiples of surds can be expressed as unit surds as k n a = n k n ´ a

MATH 23
2.Mixed surds:
If a rational number (not equal to 0) and n b is a surd , then a + n b , a - n b
are called mixed surds . If a = 0, they are called pure surds. Fractals
Examples: 2 + 3 , 5 - 3 6 are mixed surds, while 3 , 3 6 are pure surds. A fractal is a never-ending pattern.
Fractals are infinitely complex
3.Compound surd:
patterns that are self-similar across
A surd which is the sum or difference of two or more surds is called a different scales. They are created by
compound surd. repeating a simple process over and
Examples: 2 + 3 3 , 3 + 5 7 - 3 2 and 1 + 2 - 3 are compound surds. over in an ongoing feedback loop.

4.Bionomial surd:
A compound surd consisting of two surds is called a bionomial surd.
Stage 0
Examples: 3 + 3 5, 6 + 4 5, 8 - 3 7 Stage 1
5.Similar surds / Like surds:
If two surds are different multiples of the same surd, they are called similar
surds.Otherwise they are dissimilar surds.

Examples: 2 2 , 5 2 , 7 2 are similar surds. ( 2 + 3 3 ),( 2 2 + 6 3 ) are Stage 2 Stage 3


similar surds and 1 + 2 , 2 + 2 2 are similar surds 3 3 and 6 5 are dissimilar
To create the snowflake you also
surds . start with a triangle and repeatedly
add a smaller triangle to every
Equality of two mixed surds of the a + c b form and d + e b: segment of its edge. After a while, the
Two mixed surds a + c b and d + e b are equal if and only if their respective edge looks exactly the same at small
rational parts and the irrational parts are equal, i.e, a = d and c = e. and large scales.
Example 12: Identify the following types of surds:
(i) 6 +5 3 (ii) 15 + 8 - 11 (iii) 5 (iv) 5 + 7 Note
..............................................
Solution: Laws Of Surds: if a > 0, b > 0 n is
(i) 6 + 5 3 It is the sum of two surds. the positive rational number then
n n
\ It is a compound surd of two surds i.e., a bionomial surds 1. ( a )n = an = a
(ii) 15 + 8 - 11 .It is the comibnation of three surds. 2. n n
a´n b = ab
\ It is a compound surd.
(iii) 5 . It is a monomial surd or a simple surd. 3. n
a¸n b = n a
b
(iv) 5 + 7 . It is the sum of a rational number and a surd.
\ It is a mixed surd. 4. nm
a= mn
a
Example 13: Which of the following surds are similar? m n
4
= a
(i) 2 5 (ii) 3 3 5 (iii) 4 5 (iv) 5 5
Solution: 5. n
a = n´p ap
2 5 and 4 5 are multiples of the same surd, 5 p
n p nm
\ They are similar. 6. n
a = a & n ap = (ap(m

24 CLASS - IX
NUMBER SYSTEM - IV

Note Addition and Subraction of surds


.............................................. Addition and subraction of similar surds can be done using the distributive
When adding or subtracting surds, law,
a c ±b c = (a± b) c .
write each surd in its simplest form
and add or subtract like surds only. Example14: Simplify the following by combining similar surds.
Also remember, (i) 2 5 +5 5
x + y does not equal x + y and
(ii) 3 6 + 216
x - y does not equal x - y
(iii) 2 3 -5 12 + 3 48

Solution:
(i) 2 5 + 5 5 = ( 2 + 5) 5 = 7 5

(ii) 3 6 + 216 = 3 6 + 62 (6)

= 3 6 +6 6
=9 6
(iii)
2 3 - 5 12 + 3 48 = 2 3 - 5 22 (3) + 3 42 (3)
= 2 3 - 5( 2) 3 + 3(4 ) 3
= ( 2 - 10 + 12) 3
=4 3

Multiplication and division of surds


Surds of the same order can be multiplied according to the law,
( n x )( n y ) = ( n xy )
When the surds to be multiplied or divided which are not of the same order,
they have to be necessarily brought to the same order before the operation is
performed.
Example 15:
(i) 15 ´ 35 (ii) 2 3 ÷ 3 27
(iii) Multiply 3 3 by 4 2 (iv) Divide 6 5 by 3 10

Solution
(i) ( 15 )( 35 ) = (15)(35)
Note
.............................................. = (5)(3)(5)( 7)

Order of a surd = 5 21
In the surd n a , n is called the 2 3
(ii) 2 3 ÷ 3 27=
order of the surd. For example: 3 27
orders of 2 , 3 3 , 4 5 are 2, 3 and
2 3 2 3 2
4 respectively. = = =
2
(3) 3 (3) (3)(3) 3 9

MATH 25
SURDS CODE PUZZLE

9 15 23
16 6 23 22
3 21 24 23
7 23 2
12 25 4
3 1 19 10
25 4 3 17
15 9 16 5
5 15

16 3 4 22
16 6 12 21
1 14 5
9 1 19 12
.
16 23 15 1
11 3 16
17 23 9 15
2
each question.
ers . Sim plify the expression in
question numb e letters that
nu mb ers in the gr id above match the e ‘Key ’. Fil l in th e grid above with th
The own in th
rresponding letter, sh
Each answer has a co answ er to the riddle:
the qu est ion s, to discover the
match n with a surd?
satio
ver get into a conver
Why should you ne

Questions: 2. 5 2 + 2
1. 2 3 + 5 3 7
4. 4 7 + 3 7 – 2
3. 8 5 – 2 5 + 3
5
6. 2 16 + 25
5. 9 – 4
8. 8 + 6 2
7. 12 – 3
10. 32 + 5 2
9. 75 – 27
12. 3 20 – 20
11. 90 + 40
14. 4 3 + 3 2 –
3 3
13. 50 – 18
16. 10 2 – 6 3 –
4 2
15. 4 2 + 3 + 6 3 + 6 2
18. 3 18 – 2 8
17. 6 2 + 8 – 4 2
20. 175 – 28
19. 11 7 – 28
22. 60 – 15
21. 192 – 75
24. 45 – 80 + 20
23. 3 20 + 2 180

25. 150 – 54 4 5 S 5
K 8 2 O
3 5 E 18 5
A
T 6 2 2 6 3
4 2 O 2 6
F 3 7 L
A 2 3
U 3 3
P 5 10
9 2 M 5 2
G
Key:
B 3 3 2 1 3
2 2 W
N 7 3 Q
H 13
C 6 2 15
7 3 1 10 2 Y
N 5 7 R
9 7 I 1
D

26 CLASS - IX
NUMBER SYSTEM - IV

1 1
Brain Power On (iii) 3
3 = 3 3 and 4 2 = 2 4
.............................................
The LCM of 3 and 4 is 12.
Each shape contained within the 1 4
largest square is also a square. The ∴ 3 3 = 312 = 12 34
number in each square gives the 1 3

length of its sides. What are the 2 = 2 = 12 23


4 12

( 3 )( 2 ) = ( )( 2 )
values of A, B, C and D? 12
3 4 12
34 3

What is the length of the sides of the


big square? = 12 (34 )( 23 )
= 12 (81)(8) = 12 648
10 10
15
1
(iv) 6
5 = 56
C
B 12 1

12 A
4 3
10 = 10 3
D
The LCM of 3 and 6 is 6.
4 13 1 2
8 10
4
3
10 = 10 3 = 10 6 = 6 102 = 6 100
6 6
5 5
∴ =
3
10 6 100
5 1
=6 =6
100 20

Rationalising
When radicals occur in the denominator of a fraction, it is customary to rid the
denominators of the radicals. The process is called rationalizing the denominator.
Method: The radical in the denominator is multiplied by conjugate of
denominator, so that the new denominator contains no radical.

A major reason for For Example,


rationalising the denominator used
to be to make it easier to evaluate If Radical
Multiply by To get product free of radicals
the fraction (before calculators were is
available). It is easier to divide by a 7 7 7´ 7 = 49 = 7
rational number than an irrational 3
2 3
4 3
8 = 3 2´ 2´ 2 = 2
one; for example,
3 5 +2 5 -2 ( 5 ) 2 - 22 = 5 - 4 = 1
= 3 ¸ 2.236
5 11 - 7 11 + 7 ( 11)2 - 72 = 11 - 49 = -38
This is hard to do 6- 3 6+ 3 ( 6 )2 - ( 3 )2 = 6 - 3 = 3
without a calculator.

Rationalisation
3 5
= 3 ´ 2.236 ¸ 5 Definition: If the product of two irrational numbers is rational, each is called the
5
rationalizing factor of the other.
This is easier to calculate.

MATH 27
For example (a + b ) is the rationalizing factor of (a - b ) as
Brain Power On
(a + b ) ´ (a - b ) = a 2 - ( b ) 2 = a 2 - b .............................................
Applying math
Similarly, ( a + b ) is the rationalizing factor of ( )
a - b as
The largest cube that can fit into a
( a + b )( a - b ) = ( a )2 - ( b )2
sphere must have its eight vertices's
= a - b. touching the surfaces of the sphere.
The process of multiplication by a rationalizing factor is called rationalization. Express the side length ,S, of the
Conjugate surds: Two binomial irrational numbers like a + b and a - b cube in the terms of the diameter ,
which differ only in sign connecting their terms are said to be conjugate. Their D of the sphere.
main feature is that their product is always rational.

Simplifying expressions by rationalising the denominator of a fraction


Type 1: When the denominator is a monomial irrational number.
1
n
Rule: If the monomial irrational number is a ie.,a n , then its rationalizing
1 1 1 1 1
1- 1- +1- Squares are formed inside squares by
factor is a a n
, because a n , a n
= an n
= a1 = a , which is a rational
joining the midpoints of the sides of
number.
the squares as shown. If AB=4 cm,
Example16: Rationalize the denominator and simplify: find the exact length of the side of
26 3 9 16 30 the shaded square.
(i) (ii) (iii) (iv)
13 27 18 5 48

Solution:
26 26 13 26 13
(i) = ´ = = 2 13 .
13 13 13 13
3 3 27 3 27 3 × 9 × 3 3× 3 3 1
(ii) = × = = = = 3.
27 27 27 27 27 27 3
Six pipes, each 2 mm in diameter,
9 9 18 9× 9× 2 9× 3× 2 3
(iii) = × = = = 2. are stacked as shown. Find the exact
18 18 18 18 18 2
height ,h,of this stacking

(iv) 16 30 = 16 30 ´ 48 = 16 3´10 ´ 16 ´ 3
5 48 5 48 48 5´ 48
h
16 × 3 × 10 × 4 × 3 16 × 3 × 4 × 10 4
= = = 10
5× 48 5× 48 5

Type 2: When the denominator is a binomial irrational number.


Method: Multiply and divide the given irrational number by the conjugate Note
number of the irrational number in the denominator.
..............................................
The following formulae are
Example17: Rationalise the denominators of the following: helpful in finding the rationalizing
factors of mixed quadratic and
1 2 1 2 3 +6 7
(a) (b) (c) (d) cubic surds.
2- 3 3- 2 2+ 3 2- 3
(i). (a + b)(a - b) = a2 - b2
5-3 2 6+ 3 (ii). (a + b)(a2 - ab + b2) = a3 + b3
(e) (f)
5+3 2 9 + 2 18 (iii). (a - b)(a2 + ab + b2) = a3 - b3

28 CLASS - IX
NUMBER SYSTEM - IV

Solution:
Brain Power On 1 1 2+ 3 2+ 3 2+ 3
(a) 2 − 3 = 2 − 3 × 2 + 3 = 22 − ( 3 )2 = 4 − 3 = 2 + 3
.............................................
In the diagram, find BA and the 2 2 3+ 2 2( 3 + 2) 2( 3 + 2) 2( 3 + 2)
perimeter of the rectangle in surd (b) = × = = = 2( 3 + 2 )
3− 2 3− 2 3 + 2 ( 3 )2 − ( 2 )2 3−2 1
form.
1 2− 3 2− 3 2− 3 2− 3
4√7 = = = = = 3− 2
B C (c)
( ) ( )
2 2
2 + 3 ( 2 + 3 )( 2 − 3 ) 2 − 3 2−3 −1

2 3 +6 7 2 3 +6 7 2 + 3 (2 3 + 6 7 )( 2 + 3 )
2√35 (d) = ´ =
2- 3 2- 3 2+ 3 ( 2 )2 - ( 3 )2
2 6 + 2 9 + 6 14 + 6 21 2 6 + 2 ´ 3 + 6 14 + 6 21
A D

= =
2-3 -1
Find the area and perimeter of the
= -(2 6 + 6 + 6 14 + 6 21 )
following triangle.
5-3 2 5-3 2 5-3 2 (5 - 3 2 )2
(e) = ´ = 2
5 + 3 2 5 + 3 2 5 - 3 2 5 - (3 2 )2

X 25 + 9 × 2 − 30 2 1
√15 + √3
= = (43 −30 2 )
25 − 18 7

6+ 3 6 + 3 9 - 2 18 ( 6 + 3 )(9 - 2 18)
(f) = ´ =
9 + 2 18 9 + 2 18 9 - 2 18 92 - (2 18 )2
√15 - √3
9 6 - 2 108 + 9 3 - 2 54

=
81 - 4 ´18
9 6 − 12 3 + 9 3 − 6 6 3 6 − 3 3 6− 3
=
= =
Note 9 9 3
..............................................
Important results on rationalising 1
Example18: Rationalise the denominator of .
factor (RF) 2 + 3 + 10
Solution:
1. n a is a RF of n a n-1 1 1 2 + 3 − 10
= ×
and vice - versa. 2 + 3 + 10 2 + 3 + 10 2 + 3 − 10
2 + 3 − 10 2 + 3 − 10
2. n a m is a RF of n a n-m = =
( ) ( )
2 2
and vice - versa. 2+ 3 − 10 2 + 3 + 2 6 − 10

3. a + b is a RF of a - b 2 + 3 − 10 2 + 3 − 10 −5 − 2 6
= = ×
and vice - versa. −5 + 2 6 −5 + 2 6 −5 − 2 6
2 1 1 2
=
( 2 + 3 − 10 −5 − 2 6)( )
4. 3 a + 3 b is a RF of a 3 - a 3 ´ b 3 + b 3
(−5) − (2 6 )
2 2

and vice - versa.


2 1 1 2
−5 2 − 2 12 − 5 3 − 2 18 + 5 10 + 2 60
5. 3 a - 3 b is a RF of a 3 - a 3 ´ b 3 + b 3 =
25 − 24
and vice - versa.
= −5 2 − 2 4 × 3 − 5 3 − 2 9 × 2 + 5 10 + 2 4 ×155
x 2 -k x 2 -k
(
6. a + b ) (
+ a- b ) = −5 2 − 4 3 − 5 3 − 6 2 + 5 10 + 4 15
2
= 2a, a - b = 1Þ x = ± k ± 1. = −11 2 − 9 3 + 5 10 + 4 15

MATH 29
Property Addition Multiplication
1. Closure a + b is a real number a × b is a real number.
2. Commutative a + b = b + a a×b=b×a
3. Associative (a + b) + c = a + (b + c) (a × b) × c = a × (b × c)
4. Identity For each a, a + 0 = 0 + a = For each a, a × 1 = 1 × a = a

1 1
5. Inverse a + (-a) = (-a) + a = 0 a ´ = ´a = 1
a a

a × (b + c) = (a × b) + (a × c) Left distributive property


6. Distributive
(b + c) × a = ab + ac Right distributive property

M I N D M A P

Real Number (R)


The set of real numbers is all the numbers
that have a location on the number line.

Rational numbers (Q) Irrational numbers (P)


All terminating or recurring decimals All non-terminating or non-recurring decimals

Applications of irrational Surds Rationalisation


numbers: Square roots of positive If the product of two
rational numbers that cannot irrational numbers is
be expressed as the squares rational, each is called
To compute of any rational numbers. the rationalizing factor
3 3 3
compound interest Eg: √2, √4, √19 of the other.

( (
t
Amount = p 1 + r Unit surds and multiples of surds Type 1: When the
100 All multiples of surds can be denominator is a
expressed as unit surds as monomial irrational
n n n
k√a = √k ×a number.
To calculate area or
circumference of any Mixed surds Type 2: When the
circular component. If a rational number (not equal to 0) denominator is a
n n n
and √b is a surd, then a + √b, a - √b binomial irrational
number.
Compound surd
For equations in physics,
A surd which is the sum or differnce
like to evaluate gravitational
of two or more surds
force between two objects
and much more. Like surds:
If two surds are different multiples
of the same surd

30 CLASS - IX
NUMBER SYSTEM - IV

Do at Class!
I. RATIONAL NUMBERS (b) 
Find two rational numbers between 0.1 and
0.2.
1. What are the additive inverse of the following?
(c) 
How many rational numbers can you find
4 -7
(a) (b) between two given rational numbers?
9 11
1 9. Find two rational numbers between
(c) 0 (d) -3 4 7 3 and 1 .
8 (i) and , (ii) 1
(e) -2 5 13 4 5
10. Find three rational numbers between 0 and 0.2
2. What are the multiplicative inverse of the following?
11. Find three rational numbers between 3 and 4.
(a) 7 (b) 1
3 1 12. Find the rational number that is one seventh of the
(c) (d) - 3
14 2 way from 1 3 to 4 .
4 8
1 1
(e) 3 (f) a b 13. Find four rational numbers between -1 and - .
8 c 2
12
14. Express as decimal fraction.
3. Name the property illustrated. 125
15. Find a vulgar fraction equivalent to 0.03
(a) 7 ´1 = 7
1 æç 1 ö÷
+ ç- ÷ = 0
(b)
8 8 9 çè 9 ÷ø 16. Express the following rational numbers in the form
8 5 40 2 2 p
(c) ´ = (d) 0 + = , p,q are integers, q ≠ 0.
q
9 11 99 3 3
(i) 6.46 (ii) 0.136
(e) 1 ´ 3 = 3 ´ 1 (f) 5 + 1 = 1 + 5 (iii) 3.146 (iv) -5.12
2 4 4 2 7 2 2 7
(g) æç- 1 ´ 5 ö÷÷´ 2 = - 1 ´æç 5 ´ 2 ö÷÷
17. Determine, without actually dividing, which of the
çç ç following Rational numbers can be named,
è 3 7 ÷ø 9 3 çè 7 9 ÷ø
(a) by a terminating decimal,
æ ö æ
(h) ç 1 + 3 ÷÷ + 1 = 1 + ç 3 + 1 ÷÷
ö
ç ç
çè 5 4 ÷ø 2 5 çè 4 2 ÷ø (b) by a repeating decimal.
2 7 1
(i) (ii) (iii)
(i)
2 æç 7 4 ö÷ 2 7 2 4 3 20 2
´ç + ÷ = ´ + ´
5 çè 8 9 ÷ø 5 8 5 9 1 1 1
(iv) (v) (vi)
4. What two properties are involved if we wish to 6 8 11

1 1 49
show that 5 ´ a = a? (vii) (viii)
5 12 80
5. 
Suppose 'a' is a rational number. What is the 47 11
(ix) 3 (x)
reciprocal of the reciprocal of 'a'? 160 907
6. 
Suppose 'a' is a rational number. What is the 18. 
Write the terminating decimal numeral for the
opposite of the opposite of 'a'? given Rational number.
7. 
Give five equivalent numbers for the rational (i)
7 (ii) 29
3 4 50
number .
8 17
8. (a) Find a rational number between
1
and
3 (iii) (iv) 123
2 4 32 4000

MATH 31
19. 
Write the repeating decimal for each of the 26. T
ell whether each decimal numeral represent a
following, and use a bar to show the repetend. rational or an irrational number.
1 -4 (a) 0.578 (b) 0.573 333….
(i) (ii)
9 3 (c) 0.688 434 445 4… (d) 0.727 374 75….
11 (e) 0.638 754 71….. (f) 0.471 7171….
(iii) 1 (iv)
6 12 (g) 283 (h) 289.387 000….
7 (i) 5.93 (j) 2 309 87
(v) (k) 0.585 885 888…..
13
20. In each of the following, write each mixed numeral 27. Find an irrational number between
as a fraction and a decimal numeral. 2 and 3 4 and 7
(i) (ii)
7 8 5 7 5 13
(i) 3 (ii) -3
16 15 (iii) 1 and 2
8 28. Show that (i) 3 (ii) 5 are not rational numbers.
(iii) 27
33
29. Determine, on the number line, the point which
21. Find the repeating decimal that is equivalent to the represents the irrational number 3 .
1
fraction . 30. Simplify:
11
 Use the example to determine the repeating (a) 8 (b) 63
decimal that is equivalent to each of the following 1
(c) 32 (d) 9800
fractions: 2
2 3 4 5 6
, , , , 31. Using 2 =1.414 or 5 =2.236, evaluate to the
11 11 11 11 11
nearest tenth.
22. Show that 0.234 is a rational number.
2 2
(a) 50 (b) 45
23. 
Write down a fraction which is equivalent to 5 3
0.033636363…..
1
(c) 162
24. Given the following real numbers: 9
32. Combine:
3
-5,0, 3, , - 9 , 8 , 6.37, p, 4,
-2
, 0.03 (a) 2 2 + 3 2 (b) 5 7 - 2 7
5 7
(a) Which are rational? (c) 3 11 + 2 7 - 11

(b) Which are irrational? 33. Simplify and combine:


(c) Which are negative integers? (a) 3 + 48 (b) 15 48 - 2 75
(d) Which number is neither positive nor negative? 34. Multiply:
25. Write True or False to describe each sentence. (a) 5 2 ´ 2 3 (b) 2 33 ´ 5 6
(a) All rational numbers are real numbers. (c) 2 3 ´ 3
(b) All real numbers are rational numbers. 35. Divide:
(c) Some real numbers are rational numbers.
12 90 18 2
(d) All integers are rational numbers. (a) (b)
3 10 3 8
(e) No rational number is also an irrational number.
(f) There exists a whole number that is not a natural
(c)
8 2 -4 8
number. 32

32 CLASS - IX
NUMBER SYSTEM - IV

36. Which is greater?


46. Taking 2 =1.414 and 3 =1.732, find without
(a) 20 or 27 (b) 8
10 or 8
23 using tables or long division, the value of
3
(c) 3 or 4 4 1 2
(a) (b)
37. Which is the largest? 3- 2 3- 2
(i)
4
3 , 6 7 , 12 48 (ii)
6
4 , 9 10 , 3 3
1 2
47. Prove that + = 2+ 3
38. Evaluate without using tables: 2 -1 3 +1

(4 )
2
(a) 3 + 12 48. Simlify:

6 4 3 2 6
1 1 (i) - +
(b) - 2 3- 6 6+ 2 2+ 3
3 7 -5 3 7 + 5
49. If x = 2 + 3 , find the value of x 2 + 1
7- 3 x 2
x
39. If = , find the value of x.
x 7+ 3 5 - 21
50. If x = , find the value of
40. Express 1+ 2 1- 2 in the form 2
+
5+ 3 5- 3 1 1
(i) x + and (ii) x 2 +
a 5+b 6 x x2

II. SURDS III. Choose the correct answer:


Simplify: 1
1. Which of the following fractions lie between
1 5 46 1 5
41. (a) (b) (c) 1 and ?
3 12 75 4
7 4 13 7
224 (1) (2) (3) (4)
42 112 - 63 + 33 11 57 17
28 (a) 1, 2 (b) 1, 3
(c) 2, 3, 4 (d) 1, 2, 4
43. Rationalise the denominators of
1 2. Express 0.34 + 0.34 as a single decimal.
(a) (b)
2
4- 3 5+ 3 (a) 0.6788 (b) 0.689

1 3+ 2 (c) 0.6878 (d) 0.687


(c) (d)
2 5- 3 3- 2 n
3. If 64
5n = 125, then 5 = .......................
44. Find the values of a and b if 1
(a) 25 (b)
3+ 2 125
(i) =a+b 2
3- 2 (c) 625 (d) 1
5
7 -1 7 +1 2 2
4. If x 4 + 1 = 1297 and y - 1 = 2400 , then y - x
4
(ii) - =a+b 7
7 +1 7 -1
=...............
1
45. Rationalise the denominator of . (a) 10 (b) 25
2+ 3+ 5 (c) 13 (d) 43

MATH 33
2 x +3 x +3 14. Which of the following is the greatest?
5. What is the value of 42 x-2 , if (16) = (64) ? 3
(a) 64 (b) 256 (a) 72 (b) (49)2
-1
(c) 32 (d) 512 æ 1 ö÷ 3 -1
(c) çç (d) (2401) 4
6. 
Which of the following pairs having two equal çè 343 ÷÷ø
values? (where x ∈ R)
x
(a) 9 2 , 24 3
x
(b) (256) ,(4
4
x )
4
3 x
15. ( 5)( 2 )( 3 )( 6 ) =........................
6 3 12

(a) 12
1749600 (b) 3
2 ´ 12 109350
x x 2 2
(c) (343) ,(7 3
4 12
) (d) (36 ) ,(6 )
2 7 3 7 (c) 12
177960 (d) Both (a) & (b)
q− p 2 q− p
16. If p = 3 and q = 2, then (3 p − 4q) ÷ (4 p − 3q)
7. The expression ( 5- 3 )( 7- 2 ) when
= .................
simplified becomes a ...............
(a) 1 (b) 6
(a) simple surd (b) mixed surd 1 2
(c) compound surd (d) binomial surd (c) (d)
6 3
8. If m and n are positive integers, then for a positive é (32) + (81)0.2 ù 0.25

17. êê ú = ........................
{ ( a )}
mn
number a, n 0.5 0.5 ú
m
êë ( )
256 - ( ) úû
121
(a) amn (b) a (a) 2 (b) 5
(c) am/n (d) 1 (c) 1 (d) 11
1 éê m 12 æç 1 ö÷ ù
-1
1 1
9. If 2− m × =
2+ m 4
, then
14 êê
(4 ) + ççè 5m ÷÷ø ú = .......
ú
18. The smallest among the surds 10 - 5, 19 - 14 ,
ë úû 22 - 17 and 8 - 3 is .............
1 10 - 5 19 - 14
(a) (b) 2 (a) (b)
2 -1 22 - 17 8- 3
(c) 4 (d) (c) (d)
4
19. If m = a + c and m , a and c are three
10. The surds 2 3
3 and 5
5 , in their descending surds, then
order are .............. (a) m is dissimilar to a and c
3 5
(a) 3 5
3, 5 , 2 (b) 2 , 3, 5 (b) a and c are similar to m
(c) 2, 5 5, 3 3 (d) 3
3, 2 , 5 5 (c) only a is similar to m
-2 -1 (d) None of these
11. 2 éê(16 - 15) + 25(13 - 8) úù
-1 
+ (1024) = ........
ë û
20. The surd obtained after rationalising the numerator
(a) 2 (b) 3
(c) 1 (d) 5 4 - 25 - a
of is equal to .....................
x -y a -9
æxö æyö
y-x
12. If x = 2 and y = 4 then çç ÷÷ + çç ÷÷÷ .............. a-b 1
çè y ÷÷ø èç x ø (a) (b)
4 - 25 - a 4 - 25 - a
(a) 4 (b) 8
(c) 12 (d) 2 1 1
(c) (d)
13. In which of the following pairs of surds are the (a - 9)(4 + 25 - a ) 4 + 25 - a
given two surds similar?
(a) 5 ,7 5 (b) 3
7,2 7 21. If 13 - x 10 = 8 + 5 , then what is the value
(c) 7 , 28 (d) Both (a) & (c) of x?

34 CLASS - IX
NUMBER SYSTEM - IV

(a) –5 (b) –6 1
é -1 ù 4
(c) –4 (d) –2 êïìæ 1 ö-2 ïü 3 ú
30. If êï ç ÷ ï ú m
If the surds 4 4 , 6 5 , 8 6 and 12 8 are arranged in
22.  êíïççè 72 ÷÷ø ýï ú = 7 , then m = ....................
ascending order from left to right, then the third
êïî ïþ ú
ë û
surd from the left is ................... 1
-1
12
8 4
4 (a) (b)
(a) (b) 3 4
8 6
(c) 6 (d) 5 (c) –3 (d) 2
1
23. 11 11 11.....4terms = ................... é a +b ù
êïìïæ 1 öa-b ïüï ú
2
1 (a +b)
16
113 16
11 31. êï ç ÷ ï ú = ............................
êíïççè x a2 -b2 ÷÷ø ýï ú
(a) (b)
16 êïïî ïï ú
(c) 16
1114 (d) 1115 êë þ úû
5- 3 1
24. If = x + y 3 , then (x,y) is ................... (a) x2 (b)
2+ 3 x
(a) (13, –7) (b) (–13, 7) 1
(c) (–13, –7) (d) (13, 7) (c) 73 (d) 2
x
(a 2m+n-p )
2
25. The simplified form of 125 + 125 - 845 is
2m+n 1
.................. 32. If = 16 and a = 2 10
then -1

(a) 15 (b) 2 5
2n-m (am-2n+2 p )
(c) - 5 (d) -2 5 =...................
26. Which of the following statements is true? 1
(a) 2 (b)
4
p : If x is a conjugate surd of y, then x can be RF of y. 1
(c) 9 (d)
q : If x is a RF of y, then x need not be conjugate of y. 8
(a) Only p (b) Only q  −1  
( p + q−1 )( p−1 − q−1 ) ÷  1 − 1 
(c) Both p & q (d) Neither p nor q   p−1 q−1 
  ( pq )2
33. Simplify:  
3-2 5 
 1 1  
27. If = a + b 5 where a and b are rational  −1 + −1  
6- 5  p q  
numbers, then what are the values of a and b?
8 -9 8 -9 (a) (pq)2 (b) –1
(a) , (b) , (c) –(pq)–2 (d) 1
35 35 31 31
-8 9 -8 9 2 2
(c) , (d) , 34. If x= ,y= , then
31 31 35 35 10 - 8 10 + 2 2
2
35x ´(81) ´ 6561
2 (x - y ) = .......................
28. If = 37 , then x = ..................
3 2x (a) 4 2 (b) 32
(a) 3 (b) –3 (c) 8 2 (d) 64
1 -1
(c) (d)
3 3 35. If a = 6 - 3 , b = 3 - 2 and c = 2 - 6 ,
3 3 3
æn ö
2çç -4÷÷÷ then find the value of a + b + c - 2abc .
ç
29. If 2n = 1024, then 3 è 4 ø
= ..........................
(a) 3 2 - 5 3 - 6 (b) 3 2 - 5 3 - 6
(a) 3 (b) 9
(c) 27 (d) 81 (c) 3 2 - 4 3 + 6 (d) 3 2 + 4 3 + 6
MATH 35
41. Simplify:
81 81 81 81 1 1 2
36. ....¥ = ..................... - + = ..........
64 64 64 64 19 - 360 21 - 440 20 + 396
81 9 (a) 1 (b) 2
(a) (b)
64 8 (c) 0 (d) None of these
3 3
(c) (d) 42. If a = 17 - 16 and b = 16 - 15 then
2 2 2
(a) a < b (b) a > b
37. If a p = b q = c r = abc , then pqr = ...................... (c) a = b (d) None of these
(b) pq + qr + pr
2 2
(a) p q + q r
(c)
2
( pq + qr + rp ) (d) pq (qr + rp )
43. ( 6
15 - 2 56 × )( 3
)
7 + 2 2 = ...................
(a) 0 (b) 1
2
é 2 ù 1 (c) –1 (d) 2
38. The value of ê(23 + 2 )
2 3
+ (140 - 29) ú is ..........
2
ê ú
ë û 44. 63 + 56 = ..................
(a) 196 (b) 289
(c) 324 (d) 400 (a) 4
7( 3+ 5 ) (b)
4
7( 3 +1 )
1
39. If x = 6 + 5, then x 2 + - 2 = ................. (c) 4
7( 3- 5) (d) 4
7( 2 + 1)
x2
(a) 2 6 (b) 2 5
(c) 24 (d) 20 7 +2 3 c + p + q + r
45. If = ( p < q < r ), where
2 7- 5 23
40. 6 + 6 + 6 + .......¥ is equal to .................. p, q, r are rational numbers, then q+ r – p = ...............
(a) –3 (b) 3 (a) 361 (b) 302
(c) 6 (d) 2 (c) 418 (d) 426

Do at Home!
I. RATIONAL NUMBERS 3. Name the property illustrated.

1. What are the additive inverse of the following? 3 3 1 æç 1 ö÷


(a) ´1 = (b) + ç- ÷ = 0
7 7 5 çè 5 ÷ø
2 -3
(a) (b) 4 3 3 4 6 1 1 6
3 5 (c) ´ = ´ (d) + = +
5 2 2 5 8 4 4 8
1 æ 1 1ö 1 1 æ 1 1ö
(c) 0 (d) -2 (e) çç + ÷÷÷ + = + çç + ÷÷÷
4 çè 3 2 ø 6 3 çè 2 6 ø
(e) -3
4. What two properties are involved if we wish to
2. What are the multiplicative inverse of the following?
1
show that 3 ´ a = a ?
(a) 3 (b) 2 3
5. 
Suppose b is a rational number. What is the
2 1
(c) (d) - reciprocal of the reciprocal of b?
12 3
1 6. 
Suppose p is a rational number. What is the
(e) 2 opposite of the opposite of p?
3

36 CLASS - IX
NUMBER SYSTEM - IV

7. 
Give five equivalent numbers for the rational 19. 
Write the repeating decimal for each of the
5 following, and use a bar to show the repetend.
number .
6
1 5 2 5
8. (a) Find a rational number between and (i) (ii) -
3 6 3 11
(b) Find two rational numbers between 0.3 and
1 9
0.5. (iii) (iv)
3 11
(c) 
How many rational numbers can you find 1
between 1 and 2 given rational numbers? (v)
12
9. Find two rational numbers between
20. In each of the following, write each mixed numeral
3 5 2 2
(i) 5 and 8 , (ii) and . as a fraction and a decimal numeral.
3 7
3 6
10. Find three rational numbers between 1 and 3 (i) 2 (ii) -4
5 5
11. Find three rational numbers between 4 and 5. 3 4
(iii) 15 (iv) 21
12. Find the rational number that is one seventh of the 6 10
7 35 5
way from to . (v) 6
4 8 20
3
13. Find four rational numbers between -2 and - .
2 21. Find the repeating decimal that is equivalent to the
14 1
14. Express as decimal fraction. fraction .
140 11
15. Find a vulgar fraction equivalent to 0.04  Use the example to determine the repeating
decimal that is equivalent to each of the following
16. Express the following rational numbers in the form fractions:
p
, p,q are integers, q ≠ 0. 7 8 9 10
q , , ,
11 11 11 11
(i) 3.42 (ii) 0.123 22. Show that 0.1364 is a rational number.
(iii) 4.268 (iv) -3.11
(v) 1.29 23. 
Write down a fraction which is equivalent to
0.189189189189189...........
17. Determine, without actually dividing, which of the
following Rational numbers can be named, 24. Given the following real numbers:
(a) by a terminating decimal, 4 22 -3
-3, 2 , 0, , - 16 , 20 , 9.23, , 6, , 0.01
7 7 4
(b) by a repeating decimal.
(a) Which are rational?
3 4 1
(i) (ii) (iii) (b) Which are irrational?
7 6 5
(c) Which are negative integers?
2 14
(iv) (v) (d) Which number is neither positive nor negative?
9 25
25. Write True or False to describe each sentence.
18. 
Write the terminating decimal numeral for the
(a) 2 is a rational number?
given Rational number.
18 12 16 (b) 0 is a neither positive nor negative integer?
(i) (ii) iii)
15 48 50 (c) 25 is a irrational number?

11 136 (d) All real numbers are rational numbers?


(iv) (v)
22 2000 (e) 2 is a prime number?

MATH 37
26. T
ell whether each decimal numeral represent a 37. Which is greater?
rational or an irrational number. (a) 3
15 (or) 4 7 (b) 3 (or) 4 7
(a) 1.73205 (b) 0.285714285714
(c) 4
6 (or) 8 12
(c) 048 (d) 0.06
(e) 3
38. Which is the largest?
27. Find an rational number between (i) 3 (ii)
4 , 4 3 , 12 50 5
4 , 10 6 , 20
18
3 5 2 6
(i) and (ii) and
5 7 9 18
39. Evaluate
1 4
(iii) 3 and 4 (iv) and æ 1 1 ö÷
2
2 3 (a) çç + ÷
(v)
6
and
12 çè 5 - 2 5 + 2 ÷ø
7 9
æ 1 1 ö÷æç 1 1 ö÷
(b) ççç - ÷÷ç + ÷
28. Show that (i) 2 , (ii) 7 , (iii) 11 , (iv) 13 and è 5 -2 5 + 2 øçè 2 + 3 2 - 3 ÷ø
(v) 3 are not rational numbers. 40. The sides AB,BC of a traingle, right-angled at B, are

29. 
Show that (i) 3 + 5 is an irrational number
(3 ) (
3 + 5 cm, and 3 3 - 5 cm respectively. )
(ii) ( )
2 + 5 is an irrational number. Find (a) the area of the traingle
(b) the length of the hypotenuse.
30. Is 100 + 36 the same as 100 + 36 ? Give
reasons. 5+ 3
41. Simplify:
80 + 48 - 45 - 27
31. Prove that 7 is not the cube of a rational number? II. SURDS
32. Simplify: 42. Simplify:
(a) 18 (b) 75 1 3
(a) (b)
5 4 16
(c) 405 (d) 999
3 23
(c) 1
33. Combine: 49

(a) 3 3 + 5 3 (b) 10 6 - 8 6 4 18 8 75 9 2
43. - +
(c) 4 5 + 3 6 - 2 5 12 32 3

44. Rationalise the denominators of


34. Simplify and combine:
4+ 5 4- 5 3 2
(a) 2 18 - 32 (b) 28 - 175 + 112 (i) + (ii) +
4- 5 4+ 5 5- 3 5+ 3
35. Multiply:
45. Find the values of a and b if
(a) 2 7 ´ 3 5 (b) 8 ´ 12 ´ 10 5+2 3 4+ 3
(i) =a+b 3 (ii) =a+b 3
(c) 11a ´ 11a 7+4 3 4- 3

36. Divide: 1
46. Rationalise the denominator of .
p pq 6+ 7+ 8
140 + 210
(a) (b) 3-5 5
pq q 7 47. Express
3+2 5
(
in the form a 5 - b where a and )
2 169 b are simple fractions.
(c)
26
38 CLASS - IX
NUMBER SYSTEM - IV

( 3)
4
6 2 4 3 2 6 20. 3
= ......................
48. Simplify: - +
3+ 6 6+ 2 2+ 3
1 -1

7 3 2 5 3 2 21. Rationalizing factor of 5 3 + 5 3 is ......................


49. Simplify: - -
10 + 3 6+ 5 15 + 3 2 3
81 .
22. Express the following in the simplest form:
2 1 3
3
50. If x = 2 + 1 , find the value of x + .
x2 3
23. Express the following in the simplest form: 625 .
III. Fill ins. 23
24. Express the following as a pure surd: 16 .
1. 5 ´ 125 = ..................... 3
20 25. 3
3 + 2 is a surd. (True/False)
2. = ......................
320 3
5
26. Express the surd with rational denominators.
3. 3
6 ´ 6 ´ 6 = ......................
3 3 3
6
4. 3 ÷ 4 12 = ...................... 27. If p = 2 + 3 and pq is a rational number, then q
5. The sum/difference of a ration and an irrational is a unique surd. (True/False).
number is ...................... 28. Divide: 6
144 by 6
4 .
7
6. , in rational denominator is ...................... 29. 
Express the following in the simplest form:
3 3
7. Two mixed quadratic surds, a + b and a - b
3
15625 .
,whose sum and product are rational are called 30. Which is smaller, 2 - 1 or 3- 2 ?
..................... surds.
8. 10 3 and 11 3 are ..................... surds. (similar/ IV. HOTS.
dissimilar) x 2 -1
1 æç 1 ö÷ =
a -1
.
1. If x = ç a + ÷÷ , then show that
9. x + y is a pure surd, if x = ...................... (zero/ 2 çè aø x - x -1 2
2

one) 1 2
3 2
10. Conjugate surd of 2. If x = 3+33 +3 3 , then show that x -9x +18x -12 = 0.
5 - 3 is ......................
1 1
11. If x + 5 = 4 + y , then x + y = ......................
(where x and y are rational).
3. If (
x = 4 + 15 ) (
3
+ 4- 15 3 , ) then show that
3
x - 3x - 8 = 0.
12. If the product of two surds is a rational number,
then each of the two is a ..................... of the 4. If x - 12 = 4 - x , then find x.
other.
1 1 1
3
13.  6 - 7 is the conjugate surd of 6+ 7. 5. If a 3 + b 3 +c 3 =0, then show that (a + b+c) = 27 abc.
(True/False)
1 2 3
3 6. Show that - + = 0.
14. Express the surd with rational denominators. 2+ 3 5- 3 5- 2
11
15. Find the smallest rationalizing factor of 28 . æ ö÷
2
ç 2+ 3 2- 3 ÷÷ = 2
7. Prove that çç + ÷
çç
16. Multiply: 5 by 4 2 . 2 - 2 - 3 ÷÷ø
3
è 2 + 2+ 3
17. Which is greater, 2 or 3 3 ?
A B C D
8. Let = = = . Prove that Aa + Bb + Cc + Dd
18. Express the surd 2 3 as a pure surd. a b c d
= (a + b+c+d )( A + B+C+ D) .
19. Multiply: 14 by 8.
MATH 39
Chapter 2 Learning Checklist
Recapitulate

ALGEBRA - IV Division of a polynomial by a polynomial


Remainder Theorem
Factor Theorem
POLYNOMIALS Factorisation
HCF and LCM of polynomials
Square roots of algebraic expressions

Recapitulate
Definition of polynomial:
An algebraic expression in the form of a0 + a1x + a2x2 + a3x3 + a4x4 + .... + an-1xn-1 + anxn, where a0,a1,a2,a3,a4,.......an , a
are real number an≠0 and n is non negative integer is known as polynomial of degree n.

Terms used in polynomials:


Constant: Component which never change its value or magnitude is known as constant . For example all real
numbers are always constant as they never changes its values
Variable: Component of any term or expression or equation which varies situation to situation is known as variable.
Term: Term is an element which is combination of 4 things signs, number, variable and power.
l
For example: 2x 3 , 3x, -5y, - 273 , b2 are terms. Each real number is known as constant term or degree of
8
constant term is zero because it does not have any variable. For example 2 can be written as 2 ´1 - 2 ´ x 0 = 2x 0
hence 2 is a constant term. Similarly each real number is known as constant term.
Like terms: Two or more terms having same type of variable and same power on them, then they are said to be like term
7 8 2 2 10 2
for example 3x, - x, x, 2x are all like term sand 7y , 9y , y ..... are also like terms.
2 9 3
19
Unlike terms: Term if they are not like then they are known as unlike terms for example 7a, 8b, c, 2x, 0.9x 3 , 5y 2 are
3
all unlike terms.
Coefficient of a term: They constant number used in writing the term is known as coefficient of term. For example
+3 is the coefficient of term 3x 2 y 3 ,- 10 is coefficient of term - 10 x 3 .
Algebraic expression: When two or more unlike terms are written together and separated by either plus sign or minus
sign the such combination of term is known as algebraic expression.

Terms in each polynomial. Terms in each polynomial Coefficient in each term.


Polynomial Expression Terms Term Coefficient
x+3 x and 3 x 1
3x2 3
3x2 - 2x + 5 3x2, -2x, and 5
-2x -2
-7 -7 2a3b2 2
2a3b2 - 3b2 + 2a - 1 2a3b2, -3b2, 2a, and -1 2 2
- x -
1 2 2 3 1 2 2 3 3 3
x - x+ x , - x, and
2 3 4 2 3 4 -7 -7

40 CLASS - IX
ALGEBRA -IV POLYNOMIALS

Identifying Polynomials

Non-Polynomial Expression Reason it is not a polynomial


Polynomials cannot contain variable
2x + x1/2 exponents. They also cannot contain non-
integer exponents.
x In general, polynomials can contain
+ 2y fractions. However, they cannot contain
y
variables in a denominator.
Polynomials cannot have negative
6x-2 + 2x - 3
exponents on variables.
Polynomials cannot contain non-
cos(x2 - 1) polynomial functions including
trigonometric functions like cosine.
Types of Polynomials:
(i) 
Monomial: A polynomial having one term is called a monomial. (Mono means one)
Example. 2x, 4x2

(ii) Binomial: A polynomial having two terms is called a binomial. (Bi means two)
Example. 2x + 3, 4x3 + x

(iii) Trinomial: A polynomial having three terms is called a trinomial. (tri means three)
Example. 2x4 + x2 + 1, x2 + 2x + 3

(iv) Multinomial. If number of term is more than 3 than it is known as Multinomial for example 6x5 - 10x4 + 7x3 -
2x2 + 6x - 12, 1 + x + x2 + x3.

(v) Zero polynomial. Number itself is known as zero polynomial. Zero polynomial is neither monomial, nor binomial,
nor trinomial, and nor Multinomial.

Degree of a polynomial
The highest power of a polynomial is known as its degree.
(i) If the degree of a polynomial is zero then it is known as constant polynomial. Each real number is known as
constant polynomial because it degree is zero and value is constant.Constant polynomial is always monomial.

(ii) Linear polynomial: A polynomial of the form: ax+b, a≠0 is known as linear polynomial its degree is always zero it
may be monomial or binomial. For example each of polynomial 2x, -3x, 5x is linear polynomial as well as monomial
1
and linear polynomials.-3x + 7, 8x - 9, 7x + 8, x - 9 are linear polynomials as well as binomials.
3
(iii)Quadratic Polynomial: An algebric expression of type ax2 + bx + c, a ≠ 0 is known as quadratic polynomial, Or we
can say that polynomial of degree 2 is known as quadratic polynomial. Quadratic polynomial can be a Monomial,
7
binomial or trinomial. For example 3x 2 , - x 3 , 5x 2 each is quadratic polynomial as well as Monomial. Polynomials
2
31x2 - 10, 5x2 + 12, -x2 - 1 each is quadratic as well as binomial and polynomials x2 + x + 1, 5x2 - 7x + 10 are quadratic
as well as trinomials.

(iv)Cubic Polynomials: A polynomial of the form ax3 + bx2 - cx + d, a ≠ 0 is known as a cubic polynomial. , or we
can say that polynomial of degree 3. A cubic polynomial may be Monomial, binomial, trinomial or Multinomial.

MATH 41
1 3
For example 50 x 3 , - x , 12x 3 are cubic polynomials as well monomials. 6x3 + 7x, 16x3 - 8x2, 3x3 + 8 are Cubic
2
polynomials as well as binomials.

(v)Biquadratic or quadric Polynomial: A polynomial of degree 4 is called a biquadratic polynomial.


For Example. x4 + 2x3 + 4x2 + x + 2, ax4 + bx + cx + dx + e, where a,b,c,d,e are real number and a ≠ 0.
A biquadratic polynomial is also known as a quadratic polynomial.

Value of a polynomial
The value of a polynomial p(x) at x = a is p(a). Obtained on replacing x by a.

For Example: Consider p(x) = x2 + 2x + 3 . The value of p(x) at x = 2 is p(2) = (2)2 + 2(2) + 3 = 4 + 4 + 3 = 11

Zeroes of a Polynomial
Consider the polynomial
p(x) = 4x3 - 3x2 - 6x -32
p(-2) =
 4(-2)3 -3(-2)2 - 6(-2) + 32
= -32 - 12 + 12 + 32 = 0
The value of p(x) at x = -2 is 0.
We can say that (-2) is a zero of polynomial p(x)
In general, we say that (a) is a zero of polynomial p(x) is a such that p(a) = 0.
We can say p(x) = 0 is a polynomial equation and a is the root of polynomial equation p(x) = 0. So, we can say a is the
zero of the polynomial p(x) or a root of the polynomial equation p(x) = 0. By convention every real number is a zero
of zero polynomial.

Example 01:
Find p(0), p(1) and p(-1) for each following given polynomials:
(i) x2 + 2x + 1 (ii) x2 - 1 (iii) x+1 (iv) x100 - 1

Solution.
(i) p(0) = (0)2 + 2(0) + 1 = 1 (ii) p(0) = (0)2 - 1 = -1 (iii) p(0) = 0 + 1 = 1 (iv) p(0) = 0100 - 1 = -1
p(1) = 12 + 2(1) + 1 = 4 p(1) = (1)2 - 1 = 0 p(1) = 1 + 1 = 2 p(1) = 1100 - 1 = 0
p(-1)= (-1)2 + 2(-1) + 1 = 0 p(-1) = (-1)2 - 1 =0 p(-1) = -1 + 1 = 0 p(-1)=(-1)100 - 1 = 0

Polynomial operations
1.Addition and subtraction:
Adding and subtracting polynomials is the same as the procedure used in combining like terms.When adding polynomials,
simply drop the parenthesis and combine like terms. When subtracting polynomials, distribute the negative first, then
combine like terms.
Addition:
(2x2 + 3x - 7) + (3x2 - 4x - 10) = 2x2 + 3x2 + 3x - 4x - 7 - 10 = 5x2 - x - 17
Subtraction:
(5x2 - 12x + 1) - (2x2 + 3x - 7) = 5x2 - 12x + 1 - 2x2 - 3x + 7 = 3x2 - 15x + 8

42 CLASS - IX
ALGEBRA -IV POLYNOMIALS

2. Multiplication:
a. Monomial times Monomial: To multiply a monomial times a monomial, just multiply the numbers then multiply the
variables using the rules for exponents.
(-2x2y)(5xy7) = -2 × 5x2 × x × y × y7 = -10x3y8

b. Monomial times Polynomial: Simply use the distributive property to multiply a monomial times a polynomial.
5x2(-2x4 + 3y - 6) = 5x2(-2x4) + 5x2(3y) + 5x2(-6) = -10x6 + 15x2y - 30x2

c. Binomial times a Binomial: To multiply two binomials, use the FOIL method (First times first, Outside times outside,
Inside times inside, and Last times last).
(x+2)(x-3) = x(x) + x(-3) + 2(x) + 2(-3) = x2 - 3x + 2x - 6 = x2 - x - 6

d. Polynomial times polynomial: To multiply two polynomials where at least one has more than two terms, distribute
each term in the first polynomial to each term in the second.
(2x - 3)(4x2 - 5x + 1)
= 2x(4x2) + 2x(-5x) + 2x(1) - 3(4x2) - 3(-5x) - 3(1)
= 8x3 - 10x2 + 2x - 12x2 + 15x - 3
= 8x3 + 22x2 + 17x - 3

3.Division:
1. Division by Monomial: Each term of the polynomial is divided by the monomial and it is simplified
as individual fractions.
10 y 2 - 25y + 20 10 y 2 25y 20 4
= - + =- 2y + 5 -
-5 y -5 y -5 y -5 y y

Algebraic expressions and identities


(i) (x + a)(x + b) = x2 + (a + b)x + ab
(ii) (a + b)2 = a2 + 2ab + b2 Perfect squares
(iii)(a - b)2 = a2 - 2ab + b2 Perfect squares
(iv) a2 + b2 = (a + b)2 - ab = (a - b)2 + 2ab
2 2
1 æç 1ö æ 1ö
(v) x 2 + = ç x + ÷÷÷ - 2 = çç x - ÷÷÷ + 2
x 2 çè xø çè xø
(vi) (a + b)2- (a - b)2 = 4ab
(vii) (a + b)(a - b) = a2 - b2
(viii) (a + b + c)2 = a2 + b2 + c2 + 2(ab+ bc + ca)
(ix) (x + y)3 = x3 + y3 + 3x2y + 3xy2 = x3 + y3 + 3xy (x + y)
(x) (x - y)3 = x3 - y3 - 3x2y + 3xy2 = x3 - y3 - 3xy (x - y)
(xi) (x + y)(x2 - xy + y2) = x3 + y3
(xii) (x - y)(x2 + xy + y2) = x3 - y3

MATH 43
RECAP WORKOUTS
1. Identify the terms, their coefficients for each of the following expressions.
(i) 5xyz2 – 3zy (ii) 1 + x + x2 (iii) 4x2y2 – 4x2y2z2 + z2
x y
(iv) 3 – pq + qr – rp (v) - - xy (vi) 0.3a – 0.6ab + 0.5b
2 2
2. Classify the following polynomials as monomials, binomials, trinomials. Which polynomials do not fit in any
of these three categories?
x + y, 1000, x + x2 + x3 + x4, 7 + y + 5x, 2y – 3y2, 2y – 3y2 + 4y3, 5x – 4y + 3xy. 4z – 15z2, ab + bc + cd + da,
pqr, p2q + pq2, 2p + 2q

3. Add the following.


(i) ab – bc, bc – ca, ca – ab (ii) a – b + ab, b – c + bc, c – a + ac
(iii) 2p2q2 – 3pq + 4, 5 + 7pq – 3p2q2 (iv) l2 + m2, m2 + n2, n2 + l2, 2lm + 2mn + 2nl

4. Subtract the following


(a) 4a – 7ab + 3b + 12 from 12a – 9ab + 5b – 3
(b) 3xy + 5yz – 7zx from 5xy – 2yz – 2zx + 10xyz
(c) 4p2q – 3pq + 5pq2 – 8p + 7q – 10 from 18 – 3p – 11q + 5pq – 2pq2 + 5p2q
5. Verify whether the following are zeros of the polynomial, indicated against them.
1 4
(i) p(x) = 3x + 1, x =- (ii) p(x) = 5x - p, x =
3 5
(iii) p(x) = x2 - 1, 1 = 1, -1 (iv) p(x) = (x + 1)(x - 2), x = -1, 2
m
(v) p(x) = x2, x = 0 (vi) p(x) = lm + m, x = -
l
1 2 1
(vii) p(x) = 3x2 - 1, x = , (viii) p(x) = 2x + 1, x =
3 3 2
6. Multiply the following polynomials:
(i) (8x3y2)(-3x2y3) (ii) (-9x3y) (-8x2y3)
(iii) j2(k5j3) (iv) a4(b4a6)
(v) 2x3(9x2 + 5y) (vi) 5x3(2x + 4y)
(vii) 5m2(3m3 + 5m2 - 4m + 6) (viii) -4x2y(x2 + 7xy - 6y3)
(ix) (x + 6)(x + 2) (x) (x - 6)(x + 9)
(xi) (4x - 3)(3x - 5) (xii) (x - 8)(x - 7)
(xiii) (6a + 1)(5a + 2) (xiv) (5x + 4y)(2x + 5y)
(xv) (2x + y)(4x - 9y) (xvi) (6r - 5)(6r + 1)
(xvii) (6c + 7)(6c - 7) (xviii) (3x + 5y)­2
(xix) (x - 2)(x2 - x + 3) (xx) (2x - 5)(5x2 + 4x + 7)

7. Divide the following polynomials:


4x - 7
(i) 9x - 6 (ii)
3 2
2
5x 2 - 25x + 2
(iii) x - 3x + 5 (iv)
x -5 x

44 CLASS - IX
ALGEBRA -IV POLYNOMIALS

4 x10 - 5x 9 + 20 x 4
(v) (vi) (-x6 + x5 + 7x2 - 9) ÷ x4
-4 x 2
(vii) (x2 + 2x + 6) ÷ x (viii) (3x2 - 15x + 5) ÷ (-3x)
(ix) (2x11 - 5x7 - 10x6) ÷ 2x3 (x) (-2x6 + 5x5 + 9x2 + 2) ÷ x4

8. Use a suitable identity to get each of the following products.


(i) (x + 3) (x + 3) (ii) (2y + 5) (2y + 5) (iii) (2a – 7) (2a – 7)
æ 1 öæ 1ö
(iv) çç3a - ÷÷÷çç3a - ÷÷÷ (v) (1.1m – 0.4) (1.1m + 0.4) (vi) (a2 + b2) (–a2+ b2)
çè 2 øçè 2ø
æ x 3 y öæ x 3 y ö
(vii) (6x – 7) (6x + 7) (viii) (–a + c) (–a + c) (ix) çç - ÷÷çç - ÷÷
çè 2 4 ÷øçè 2 4 ÷ø
(x) (7a – 9b) (7a – 9b)

9. Use the identity (x + a) (x + b) = x2 + (a + b) x + ab to find the following products.


(i) (x + 3) (x + 7) (ii) (4x + 5) (4x + 1) (iii) (4x – 5) (4x – 1)
(iv) (4x + 5) (4x – 1) (v) (2x + 5y) (2x + 3y) (vi) (2a2 + 9) (2a2 + 5)
(vii) (xyz – 4) (xyz – 2)

10. Find the following squares by using the identities.


(i) (b – 7)2 (ii) (xy + 3z)2 (iii) (6x2– 5y)2
2
æ2 3 ö
(iv) çç m + n÷÷÷ (v) (0.4p – 0.5q)2 (vi) (2xy + 5y)3
çè 3 2 ø

11. Simplify.
(i) (a2– b2)2 (ii) (2x + 5)2 – (2x – 5)2 (iii) (7m – 8n)2+ (7m + 8n)2
(iv) (4m + 5n)2 + (5m + 4n)2 (v) (2.5p – 1.5q)2 – (1.5p – 2.5q)2 (vi) (ab + bc)2– 2ab²c
(vii) (m2 – n2m)2 + 2m3n2

12. Show that.


(i) (3x + 7)2 – 84x = (3x – 7)2 (ii) (9p – 5q)2+ 180pq = (9p + 5q)2
2
æ4 3 ö 16 2 9 2
(iii) ççç m - n÷÷÷ + 2mn = m + n (iv) (4pq + 3q)2– (4pq – 3q)2 = 48pq2
è3 4 ø 9 16
(v) (a – b) (a + b) + (b – c) (b + c) + (c – a) (c + a) = 0

13. Using identities, evaluate.

(i) 71² (ii) 99² (iii) 1022 (iv) 998²


(v) 5.2² (vi) 297 x 303 (vii) 78 x 82 (viii) 8.92
(ix) 10.5 x 9.5

14. Using a2– b2 = (a + b) (a – b), find


(i) 512 – 492 (ii) (1.02)2 – (0.98)2 (iii) 1532– 1472 (iv) 12.12 – 7.92

15. Using (x + a) (x + b) = x2 + (a + b) x + ab, find


(i) 103 x 104 (ii) 5.1 x 5.2 (iii) 103 x 98 (iv) 9.7 x 9.8

MATH 45
46
Brain Power On
.....................................................................................................................................................................................................
Directions: Help polly penguin find her way back to her igloo! Solve the problems and follow the correct path to the igloo! Watch for dead ends!

CLASS - IX
ALGEBRA -IV POLYNOMIALS

Note Division of a polynomial by a polynomial


..............................................
(i) Factor method
The factor method for division of In this method, we factorize the polynomial to be divided so that one of the
polynomials is used only when the factors is equal to the polynomial by which we wish to divide.
remainder is zero.
Example 01:
Divide: 4x2 + 7x - 15 by x + 3

Solution:
4x2 + 7x - 15 = 4x2 + 12x - 5x - 15
The word algebra is
derived from the Arabic word al- 4x(x + 3) - 5(x + 3) = (4x - 5)(x + 3)
jabr.This word is a part of the title
4 x 2 + 7x - 15 (4 x - 5)( x + 3)
of a ninth century work, “Hisâb \ = = 4x -5
al-jabr w’al-muqâbalah,”written x +3 x +3
by Mohammed ibn Músâ al- (ii)Long division method:
Khwârizmî. Step 1 : First arrange the terms of the dividend and the divisor in the descending
The word al-jabr means “a order of their degrees.
restoration,” a reference to the fact Step 2 : Now the first term of the quotient is obtained by dividing the first term
that, if a number is added to one side of the dividend by the first term of the divisor.
of an equation, then it must also be
added to the other side in order to Step 3 : Then multiply all the terms of the divisor by the first term of the
“restore” the equality. The title of the quotients and subtract the result from the dividend.
work, freely translated, is “The Science Step 4 : C
 onsider the remainder as new dividend and proceed as before.
of Reduction and Cancellation.” Of
course, today,algebra has come to Step 5 : Repeat this process till we obtain a remainder which is either 0 or a
mean a great deal more. polynomial of degree less than that of the divisor.
Example 02:
Divide: 2x3 + 9x2 + 4x - 15 by 2x + 5
Solution:
2x +5 2x3 + 9x2 + 4x -15 x2 + 2x - 3
2x3 + 5x2
(-) (-)
4x2 + 4x
4x2 + 10x
(-) (-)
- 6x - 15
- 6x - 15
(+) (+)
0
∴ (2x3 + 9x2 + 4x -15) ÷ (2x +5) = x2 + 2x - 3

Remainder Theorem
Let p(x) be any polynomial of degree greater than or equal to one and ‘a’ be
any real number.
All of this was done using words, no When p(x) is divided by (x-a), the remainder is p(a).
algebraic symbolism. If a polynomial p(x) is divided by (x-a), the remainder is p(a).

MATH 47
Proof: Let p(x) be divided by (x-a), the quotient is q(x) and the remainder is r(x).
Then we have Brain Power On
p(x) = (x-a) q(x) + r(x), ........................(1) .............................................
r(x) = 0 or r(x) ≠ 0 . Fill in the empty spaces to
complete the puzzle. In any row, the
two left spaces should multiply to
Either
equal the right-hand space. In any
Case I. If r(x) = 0 column, the two top spaces should
On putting the value of r(x) = 0 in (1) we get multiply to equal the bottom space.
p(x) = (x-a)q(x) ..............(2)
On substituting x = a in (2), we have 6

p(a) = (a-a) q(a) = 0


x+3
p(a) = 0 = Remainder

18 36x2 + 144x
Case II. If r(x) ≠0 + 108
Degree of r(x) < degree of (x-a)
Since degree (x-a) = 1, degree of r(x) is 0, i.e., constant. Let r(x) = r.
On substituting r(x) = r in (1), we get
P(x) = (x-a) q(x) + r .......................(3)
On substituting x= a in (3), we get
p(a) = (a-a) q(a) + r
p(a) = 0 × q(a) + r
p(a) = 0 + r
p(a) = r
Hence, the remainder is p(a) when p(x) is divided by (x-a).

1
Example 03: Find the remainder when x3 - 4x2 + 12x + 7 is divided by x + .
2
Solution: Let f(x) - x3 - 4x2 + 12x + 7............................(1) Note
..............................................
1 æ 1ö
Divisor = x + = x - çç- ÷÷÷ If a polynomial p(x) is divided by g(x)
2 çè 2 ø
æ 1ö then we get remainder as follows:
By remainder theorem, when f(x) is divided by x - ççç- ÷÷÷ ,
è 2ø
Remainder
æ 1ö Divisor = g(x)
then remainder = f çç- ÷÷÷ =r
çè 2 ø
(x + a) = x - (-a) p(-a)
On Substituting x = - 1 in (1), we get
2 æ bö æ bö
(ax - b ) = a çççx - ÷÷÷ p ççç ÷÷÷
 1  1
3 2
 1  1 1 1 1 è aø èaø
f −  = −  − 4 −  + 12 −  + 7 = − − 4× −12× + 7
 2   2   2   2  8 4 2
æ bö æ bö
1 1 (ax + b) = a ççç x + ÷÷÷ p ççç- ÷÷÷
= − −1 − 6 + 7 = − è aø è aø
8 8
-1 æ bö æ bö
Remainder = b - ax = -a çç x - ÷÷÷ p çç ÷÷÷
8 çè a ø çè a ø

48 CLASS - IX
ALGEBRA -IV POLYNOMIALS

Brain Power On
.....................................................................................................................................................
Synthetic Division Sudoku
2 4
Directions: On a separate sheet of paper, solve 1. Divide:( −4 − 15 + − ) ÷ ( + 2) F-1
each problem and place the remainder in the
3 2
indicated row and column of the puzzle.Then, 2. Divide:( −4 + − 42 ) ÷ ( − 5) C- 7
write out the solution in the space below the
problem. When finished, solve the remaining 3. Divide:( 3
+3 2
− − 54 ) ( + 6) −1 H- 3
Sudoku puzzle. Remember, each row, each
column, and each 3 x 3 square should have the 4. Perform the following operation:
numbers 1 – 9, with no repetition. 3 2
F-8
+4 − +2
A B C D E F G H I −1
4 3 2
5. Divide: ( + 12 + 25 + − 18 ) ÷ ( + 10 ) A - 3
1 5 4 6 7
6. Perform the following operation:
2 6 4 9 5
4
−2 3
− 52 2 + − 28 A-5
3 3 1 8 −8

4 8 2 1 4 5 7 7. Divide:( 4
−8 3
+ − 58 ) ÷ ( − 8) E-8

5 3 2
8. Divide:( 9 − 73 + − 2) ÷ ( − 7) I-8
6 1 5 3 6 8 4
9. Perform the following operation:
7 1 7 6 A-8
3 4
− + − 47 2 − 21 +
8 2 4 3 8 +6

9 6 8 1 2 10. Divide:( 6 3
+ 47 2
+ + 84 ) ( + 8) −1 G-3

Example 04: If the polynomials px3 + 4x2 + 3x - 4 and x3 - 4x + p are divided by (x
– 3) then the remainder in each case is the same. Find the value of p.
Solution: Let A(x) = px3 + 4x2 + 3x - 4 ................... (1)
B(x) = x3 - 4x + p ........................... (2)
g(x) = x - 3
If (x-3) divides (1), then remainder = A(3)
On Substituting x =3 in (1), we get
A(3) = p(3)3 + 4(3)2 + 3(3) - 4
= 27p + 36 + 9 - 4
= 27p + 41
If (x-3) divides (2), then remainder = B(3)
On Substituting x =3 in (2), we get
B(3) = (3)3 - 4(3) + p
= 27 - 12 + p
= 15 + p
MATH 49
Remainder A(3) = B(3)
27p + 41 = 15 + p
Brain Power On
.............................................
27p - p = 15 - 41
26p = -26 ⇒ p = -1 Circling the squares !
Example 05: Let R1 and R2 are the remainder when the polynomials The puzzle is to place a different
f(x) = 4x3 + 3x2 - 12ax - 5 and g(x) = 2x3 + ax2 - 6x + 2 are number in each of the ten squares
divided by (x-1) and (x+2) respectively. If 3R1+R2+28=0, find so that the sum of the squares of any
the value of ‘a’. two adjacent numbers shall be equal
Solution: Let f(x) = 4x3 + 3x2 - 12ax - 5 to the sum of the squares of the two
numbers diametrically opposite to
∴R1 = Remainder when f(x) is divided by (x-1).
them.
R1 = f(1)
The four numbers placed, as
R1 = 4(1)3 + 3(1)2 - 12a(1) - 5
examples, must stand as they are.The
R1 = 4 + 3 - 12a - 5 square of 16 is 256, and the square
R1 = 4 + 3 - 12a - 5 of 2 is 4. Add these together, and the
R1 = 2 - 12a result is 260. Also—the square of
g(x) = 2x3 + ax2 - 6x + 2 14 is 196, and the square of 8 is 64.
R2 = Remainder when g(x) is divided by x+2 These together also make 260. Now,
in precisely the same way, B and C
R2 = g(-2)
should be equal to G and H (the sum
R2 = 2(-2)3 + a(-2)2 - 6(-2) + 2
will not necessarily be 260), A and
R2 = 2 × (-8) + a × 4 - 6 × (-2) + 2 K to F and E, H and I to C and D,
R2 = -16 + 4a + 12 + 2 = -2 + 4a and so on, with any two adjoining
Given that squares in the circle. All you have
3R1 + R2 + 28 = 0 to do is to fill in the remaining six
3(2 - 12a) + (-2 + 4a) + 28 = 0 numbers. Fractions are not allowed,
and I shall show that no number
6 - 36a - 2 + 4a + 28 = 0
need contain more than two figures.
-32a + 32 = 0 ⇒ 32a = 32
A B
a=1
16 2
J C
Factor Theorem
Let p(x) be any polynomial of degree greater than or equal to 1 and ‘a’ be any
real number, then
(i) (x-a) is a factor of p(x) if p(a) =0; and I D
(ii) p(a) = 0 if (x-a) is a factor of p(x)
Proof: Let p(x) be a polynomial of degree and ‘a’ be a real number.
(i) If p(a) = 0 (given) H E
14 8
Let q(x) be the quotient when p(x) be divided by (x-a) G F
By remainder theorem, remainder = p(a)
Polynomial = Divisor × Quotient + Remainder
∴ p(x) = (x-a) q(x) + p(a) ⇒ p(x) = (x-a)q(x) ∵ p(a) = 0
⇒(x-a) is a factor of p(x). Proved.
(ii) If (x-a) is a factor of p(x) (given)
⇒ p(x) divided by (x-a) gives remainder = 0 .....(1)

50 CLASS - IX
ALGEBRA -IV POLYNOMIALS

But, by remainder theorem, when p(x) is divided by (x-a), gives


Brain Power On Remainder = p(a) ….(2)
.............................................
From (1) and (2), remainder p(a) = 0 Proved.
Get Matheholic!
Example 06: Find the values of a and b so that the polynomial x3 - ax2 - 13x +
1. There are two jugs at hand with b has (x-1) and (x+3) as factors.
a capacity of p and q liters and any
amount of water. What quantities m Solution: Let f(x) = x3 - ax2 -13x + b
of water can be measured out? If (x-1) is a factor of (1), then f(1) = 0 .................................. (1)
(a) p = 5, q = 3, m = 4 On Substituting x=1 in (1), we get
(b) p = 5, q = 3, m = 1
f(1) = (1)3 - a(1)2 - 13(1) + b = b - a - 12 ................(2)
(c) p = 4, q = 9, m = 1, 2, . . . , 13
Or
2.How can one measure 15 min If (x+3) is a factor (1), then f(-3) = 0
using a 7 min- and a 11 min sand-
On substituting x = -3 in (1), we get
glass?
f(-3) = (-3)3 - a(-3)2 - 13(-3) + b
0 = -27 - 9a + 39 + b
0 = 12 – 9a + b ..........................................................(3)
From (2) b – a - 12 = 0
From (3) b - 9a + 12 = 0 On solving a = 3, b = 15
Example 07: Without actual division, prove that 2x4 - 6x3 + 3x2 - 2 is exactly
divisible by x2 - 3x + 2.
Solution: Let f(x) = 2x4 + 6x3 + 3x2 + 3x - 2 .................................(1)
Let g(x) = x2 - 3x + 2 = x2 - 2x - x + 2
3.At a 100 m race the first runner
Now, f(x) will be exactly divisible by g(x), if it is exactly divisible by (x-2)
A beats the second B by 10 m, and
as well as (x-1).
the second B beats the third C by
10 m. How many meters is the first If (x-2) divides (1), then f(2) = 0
runner A ahead of the third C when On substituting, x= 2 in (1), we get
crossing the finishing line?
f(2) = 2(2)4 - 6(2)3 + 3(2)2 + 3(2) -2
= 32 – 48 + 12 +6 – 2 = 0
So (1) is exactly divisible by (x-2), by factor theorem.
If (x-1) divides (1), then f(1) = 0
On substituting, x = 1 in (1), we get
f(1) = 2(1)4 - 6(1)3 + 3(1)2 + 3(1) - 2
4. If 5 cats catch 5 mice in 5 minutes
how many cats catch 100 mice in =2–6+3+3–2=0
100 minutes? As f(1) = 0
So by factor theorem (x-1) is the factor of (1).
i.e., (x-2)(x-1) is a factor of (1)
(x2 - 3x + 2) divides (2x4 - 6x3 + 3x2 + 3x - 2)exactly.

MATH 51
Brain Power On
.....................................................................................................................................................
Polynomials Word Search

BINOMIAL COEFFICIENT

DEGREE SUBTRACTION

MONOMIAL QUARTIC

LINEAR HEXIC

GROUPING CONSTANT

VARIABLE QUINTIC

EXPONENT POLYNOMIAL

TERM RATIONAL

ADDITION

Example 08: What must be added to x4+2x3 - 2x2 - 2x - 1 to obtain a polynomial


which is exactly divisible by (x2 + 2x - 3)?
Solution: Let f(x) = x4 + 2x3 - 2x2 - 2x - 1 ..................................... (1)
Let g(x) = x2 + 2x - 3 = x2 + 3x - x - 3
= x(x + 3) - 1 (x + 3) = (x - 1) (x + 3)
Now, f(x) when divided by a quadratic polynomial (degree 2) will give a
remainder of degree 1.
Let (ax+b) be added to f(x) to obtain
p(x) = (x4 + 2x3 - 2x2 - 2x - 1) + (ax + b)
If (1) is divisible by (x2 + 2x -3)or (x-1)(x+3)
p(-3) = 0 and p(1) = 0
On substituting, x = -3 in (1) we get
p(-3) = (-3)4 + (2)(-3)3 - 2(-3)2 - 2(-3) - 1 + a(-3) + b
= 81 – 54 – 18 + 6 – 1 - 3a + b
= 14 – 3a + b, As p(-3) = 0 so 14 – 3a + b = 0 ................. (2)
On substituting, x = 1 in(1) we get
p(1) = (1)4 + 2(1)3 - 2(1)2 - 2(1) - 1 + a(1) + b

52 CLASS - IX
ALGEBRA -IV POLYNOMIALS

=1+2–2–2-1+a+b
Brain Power On =3–5+a+b
.............................................
Factoring Polynomials message = -2 + a + b. As p(1) = 0 so -2 + a + b = 0 ...................... (3)
Decoder On solving (2) and (3), we get
Solve each problem and write the
matching letter on the blank above a = 4, b = -2
the answer.
The polynomial to be added = ax + b = 4x - 2
C 3x2 - 8x + 4 Factorisation
S 5x + 19x + 12
2
Type 01: Factorisation by taking out the common factor:
Y 9x + 66x + 21
2 By observation, if we find some common factor in each term of an expression,
take it out as common factor. Divide each term by this common factor.
O 5x2 - 18x + 9
Example 09: Factorise the following:
I 4x2 - 15x - 25
(i) 3x2 + 6xy (ii) 7xy - 21x2y2
F 4x2 - 17x + 4
(iii) 3x2y2 + 2x3y + 9xy2 (iv) x3y3 + 2x2y2 + x2y4
A 6x2 + 5x - 6
(v) 46x2 + 2xy + 10y3 (vi) ax2 + bx2 +ay2 +by2
U 16x2 + 60x - 100
Solution:
T 6x2 - 7x - 10
(i) 3x2 + 6xy
N 6x2 + 12x - 144
Here 3x is a common factor.
H x2 + 3x - 18
So, 3x2 + 6xy = 3x (x + 2y)
P 6x2 + 66x + 60
(ii) 7xy - 21x2y2
What is the difference between a
fish and a piano? Here 7xy is a common factor.
So, 7xy - 21x2y2 = 7xy(1 - 3xy)
(iii) 3x2y2 + 2x3y + 9xy2
Here xy is a common factor
So, 3x2y2 + 2x3y + 9xy2 = xy(3xy + 2x2 + 9y)
3(3x + 1)(x + 7) (5x - 3)(x - 3)
(iv) x3y3 + 2x2y2 + x2y4
4(x + 5)(4x - 5) Here x2y2is a common factor
x3y3+2x2y2 + x2y4 = x2y2(xy + 2 + y2)
(3x - 2)(x - 2) (2x + 3)(3x - 3) (v) 46x2 + 2xy + 10y3
,
6(m - 4)(m + 6) (x - 2)(6x + 5) Only 2 is a common factor
So, 46x2 + 2xy + 10y3 = 2(23x2 + xy + 5y3)
(x - 2)(6x + 5) 4(x + 5)(4x - 5) (vi) ax2 + bx2 + ay2 + by2
In the first terms, x2 is a common factor and y2 is a common factor in the
6(x - 4)(x + 6) (2x + 3)(3x - 2) last two terms.
So, ax2 + bx2 + ay2 + by2 = x2(a + b) + y2(a + b)
(x - 4)(4x - 1) (x - 5)(4x + 5)
Now (a+b) is a common factor.
(5x + 4)(x + 3) (x - 3)(x + 6) So, ax2 + bx2 + ay2 + by2 = x2(a + b) + y2(a+b) = (a + b)(x2 + y2)

MATH 53
Brain Power On
.....................................................................................................................................................
Find a route through the maze, only passing through squares with true identities.

Type 02: Factorisation by grouping the terms:


Sometimes it is not possible to take out a common factor from all the terms
of a given expression.We group the terms to have a common factor. Common
factors are taken out from each group. This method will be more clear from the
following examples.
Example 10: Factorise the following:
(i) a2 + 2b + ab + 2a (ii) xy + 2bx + ay + 2ab
(iii) 2b2 + 8ab + 4ac + bc (iv) ax - by + bx - ay
(v) axy2 + 3x + 2a2y2 + 6a (vi) 6pm + 9mq + 8pn + 12qn
Solution:
(i) a2 + 2b + ab + 2a = (a2 + ab) + (2a + 2b)
= a(a + b) + 2(a + b)
= (a+b) (a+2)
(ii) xy + 2bx + ay + 2ab = x(y + 2b) + a(y + 2b)
= (y + 2b)(x + a)
54 CLASS - IX
ALGEBRA -IV POLYNOMIALS

(iii) 2b2 + 8ab + 4ac + bc = (2b2 + bc) + (8ab + 4ac)


Brain Power On
= b(2b+c)+ 4a(2b+c)
.............................................
= (2b+c)(b+4a)

(iv) ax - by +bx -ay = (ax - ay) + (bx - by)


= a(x-y) + b(x-y)
= (x-y)(a+b)

(v) axy2+3x+2a2y2+6a = (axy2+3x) +(2a2y2+6a)


= x(ay2+3)+2a(ay2+3)
Spot the math errors!
1. x2 - x2 = x2 - x2 ⇒ (x + x)(x - x) = (ay2 + 3)(x + 2a)
= x(x - x) ⇒ x + x = x ⇒ 2x =
(vi) 6pm + 9mq + 8pn + 12qn = (6pm + 9mq) + (8pn + 12qn)
x⇒2=1
= 3m(2p + 3q) + 4n(2p + 3q)
2. 25 - 45 = 16 - 36
9 9 = (2p + 3q)(3m + 4n)
52 - 2 × 5 × = 42 - 2 × 4 ×
2 2
9 81 9 81 Type 03: Factorisation of the perfect square polynomials:
52 - 2 × 5 × + = 42 - 2 × 4 × +
2 4 2 4
Formulae: (i) a2 + 2ab + b2 = (a + b)2 (ii)a2 - 2ab + b2 = (a - b)2
( ( ( (
2 2
9 = 4- 9
5- (iii)(a + b + c)2 = a2 + b2 + c2 + 2ab + 2bc + 2ca
2 2

5-
9
= 4-
9 (a) Here the first and third terms are perfect squares.
2 2
5=4
(b) The middle term = 2 (Product of square roots of first and third terms).This
method is illustrated by the following examples.
3. x = (π + 3)
Example11: Factorise:
2
2x = (π + 3) (i) 4a2 + 12ab + 9b2 (ii) x2 + 10x + 25
2x(π - 3) = (π + 3)(π - 3) (iii) 9x2 + 24xy + 16y2 (iv) 16a2 - 40ab + 25b2
2πx - 6x = π2 - 9 25
(v) x 2 + 5x + (vi) 49x4 - 168x2y2 + 144y4
9 - 6x = π2 - 2πx 4
x4 4
9 - 6x + x2 = π2 - 2πx + x2 (vii) + 4 +1
4 x
(3 - x)2 = (π - x)2
Solution:
3-x=π-x
π=3 (i) 4a2 + 12ab + 9b2 = (2a)2 + (3b)2 + 2(2a)(3b) = (2a + 3b)2

(ii) x2 + 10x + 25 = (x)2 + (5)2 + 2(x)(5) = (x + 5)2


4. a=b
a2 = ab (iii) 9x2 + 24xy + 16y2 = (3x)2 + (4y)2 + 2(3x)(4y) = (3x + 4y)2
a2 - b2 = ab - b2 (iv) 16a2 - 40ab + 25b2 = (4a)2 + (5b)2 - 2(4a)(5b) = (4a - 5b)2
(a + b)(a - b) = b(a - b) 2 2
25  5  5  5
= ( x ) +   + 2 ( x )  =  x + 
2
a+b=b 2
(v) x + 5x +
4  2   2   2
2b = b
2=1 (vi) 49x4 - 168x2y2 + 144y4 = (7x2)2 + (12y2)2 - 2(7x2)(12y2) = (7x2 - 12y2)2

MATH 55
4 4
(iv) x + 4 + 1 = x + 4 + 2 − 1
4 x4 4 x4
 x 2 2  2 2  x 2  2   x2 2
2

=   +  2  + 2   2  − (1) =  + 2  − (1)
2 2

 2  
 x  
2 x  
  2 x 

 x2 2  x 2 2 
=  + 2 + 1 + 2 − 1
 2 x  2 x 
 2  2  x  2    x2 
 x 2 2
=   +   + 2    −1  + 2 −1
 2   x   2  x    2 x 
 
Type 04: Factorisation by splitting the Middle term
(Factors of quadratic polynomial):
Let the quadratic polynomial be x2 + bx + c Let x+p and x+q be two linear
factors of this polynomial.
So x2 + bx + c = (x + p)(x + q) or x2 + bx + c = x2 + (p + q)x + pq
Comparing the coefficients of like power of ‘x’ on both sides, we get b = p+q,
c = pq
To factorise the given quadratic polynomial, we have to find two numbers p
and q such that p+q=b and pq=c
Verification of the method
x2 + bx + c = x2 + (p + q)x + pq
Or x2 + bx + c = x2 + px + qx + pq
Or x2 + bx + c = x(x + p) + (x + p)q
Or x2 + bx + c = (x + p)(x + q)
Thus we have the following rule for factorisation of x2 + bx + c
I. Convert the given quadratic polynomial in the standard form .
II. Find two numbers p and q such that
(a) Sum of p and q is equal to b.
p+q=b
(b) Product of p and q is equal to c.
pq=c

III. Split the middle term bx as px+qx and e as pq.


x2 + bx + c = x2 + px + qx + pq
IV. By grouping, factorise them
x2 + bx + c = (x2 + px) + (qx + pq)
Or x2 + bx + c = x(x + p) + q(x + p) Note
Or x2 + bx + c = (x + p) (x + q) ..............................................

The following solved examples will make this method more clear. If c is –ve, then out of a p and q, one
number is +ve and the other is –ve.

56 CLASS - IX
ALGEBRA -IV POLYNOMIALS

Example 12. Factorise:


(a) x2 - 21x + 90 (b) x2 - 27x + 176 (c) x2 + 5x - 84
(d) x2 + 19x - 150 (e) x2 - 24x - 180 (f) x2 - 20x - 300
Hint for trial
90 = 2 × 45, 2 + 45 = 47 Solution:
90 = 3 × 30, 3 + 30 = 33 (a) x2 - 21x + 90

90 = 5 × 18, 5 + 18 = 23 (i) Find two numbers p and q such that


90 = 6 × 15, 6 + 15 = 21 p+q=-21 and pq = 90
By trial, -6-15=-21 and (-6)(-15)=90

(ii) Split the middle term -21x as -6x-15x, and 90 as (-15)(-6) so that
x2 - 21x + 90 = x2 - 6x - 15x + (-15)(-6)

(iii) Factorise by grouping


x2 - 21x + 90 = (x2 - 6x) + (-15x +( -15)(-6))
Or x2 - 21x + 90 = x(x - 6) - 15(x - 6)
Hint for trial Or x2 - 21x - 90 = (x - 6)(x - 15)
176 = 2 × 88, 2 + 88 = 90
(b) x2 - 27x + 176
176 = 4 × 44, 4 + 44 = 48
(i) Find two numbers p and q such that
176 = 8 × 22, 8 + 22 = 30
p+q=-27 and pq=176
176 = 16 × 11, 16 + 11 = 27
By trial, -16-11=-27 and pq = (-11)(-16)

(ii) Split the middle term -27x as -16x -11x, and 176 as (-11) (-16) so that
x2 - 27x + 176 = (x2 - 16x) + [-11x + (-11)(-16)]
Or x2 - 27x + 176 = x(x - 16) - 11(x - 16)
Or x2 - 27x + 176 = (x - 16)(x - 11)

(c) x2 + 5x - 84

(i) Find two numbers p and q such that


p+q=5 and pq=-84
Since the product pq is negative, one number is +ve and the other is –ve.
By trial, 12-7=5 and (12)(-7)=-84
Hint for trial
(ii) Split the middle term 5x as 12x-7x and -84 as -7 12 so that
84 = 2 × 42, 42 - 2 = 40
x2 + 5x - 84 = x2 + 12x - 7x + (-7)(12)
84 = 3 × 28, 28 - 3 = 25
84 = 6 × 14, 14 - 6 = 8 (iii) Factorise by grouping

84 = 7 × 12, 12 - 7 = 5 x2 + 5x - 84 = (x2 + 12x) + {(-7x + (-7)(12))}


x2 + 5x - 84 = x(x + 12) -7(x + 12)
x2 + 5x - 84 = (x + 12)(x - 7)

MATH 57
(d) x2 + 19x - 150
(i) Find two numbers p, q such that
p+q = 19 and pq = -150
Hint for trial
S ince the product is negative (-150), one number is +ve and the other
150 = 2 × 75, 75 - 2 = 73
is –ve.
150 = 3 × 50, 50 - 3 = 47
By trial, 25-6 = 19 and (-6)(25) = -150
150 = 5 × 30, 30 - 5 = 25
(ii) Split the middle term 19x as 25x-19x and -150 as (-6) (25)
150 = 6 × 25, 25 - 6 = 19
So that x2 + 19x - 150 = x2 + 25x - 6x + (-6)(25)
(iii) Factorise by grouping.
x2 + 19x - 150 = x2 + 25x - 6x + (-6)(25)
x2 + 19x - 150 = x(x + 25) - 6(x + 25)
x2 + 19x - 150 = (x + 25)(x - 6)

(e) x2 - 24x - 180


(i) Find two numbers p and q such that
p+q = -24 and pq = -180 Hint for trial
Since the product pq is negative (-180), 180 = 2 × 90, -90 + 2 = -88
one number is +ve, the other is –ve. 180 = 3 × 60, -60 + 3 = -57
By trial, -30+6=-24 and (-6) (30)=-180 180 = 4 × 45, -45 + 4 = -41
(ii) Split the middle term -24x as -30x+6x and -180 as (-6) 30 so that
180 = 5 × 36, -36 + 5 = -31
x - 24x - 180 = x - 30x + 6x + (-6)(30)
2 2
180 = 6 × 30, -30 + 6 = -24
(iii) Factorise by the grouping.
x2 - 24x - 180 = (x2 - 30x) + 6x + (-6)(30)
x2 - 24x - 180 = x(x - 30) + 6(x - 30)
x2 - 24x - 180 = (x - 30)(x + 6)

(f) x2 - 20x - 300


(i) Find two numbers p, q such that
p+q=-20 and pq = -300
Since the product pq is negative (-300).
Hint for trial
One number is +ve, the other is –ve.
300 = 2 × 150, -150 + 2 = -148
By trial -30+10 = -20 and 10(-30) = -300
300 = 3 × 100, -100 + 3 = -97
(ii) Split the middle term -20x as -30x + 10x and -300 as 10 (-30) so that
300 = 4 × 75 -75 + 4 = -71
x2 - 20x - 300 = x2 - 30x + 10x + 10(-30)
300 = 5 × 60, -60 + 5 = -55
(iii) Factorise by grouping.
300 = 6 × 50, -50 + 6 = -44
x2 - 20x - 300 = (x2 - 30x) + (10x + (10)(-30))
300 = 10 × 30, -30 + 10 = -20
x2 - 20x - 300 = x(x - 30) + 10(x - 30)
x2 - 20x - 300 = (x - 30)(x + 10)

58 CLASS - IX
ALGEBRA -IV POLYNOMIALS

Type – 5 : Factorising the difference of two squares


If the given polynomial is in the form of the difference of two squares, then its
two factors are
1. Sum of the two square roots.
2. Difference of the two square roots.
a2 - b2 = (a + b)(a - b)
The method will be more clear by the following solved examples.

Brain Power On
....................................................................................................................................................
Factor completely. Then find the letter which matches.
............................. 1. 2x 2 + 8x + 6 A. None of the answers below
............................. 2. 8 x3 – 1 B. x (x 2 + 1) (x– 1) (x + 1)

............................. 3. x3 + 27 C. (x – 3) (x 2 – 3x + 9)
2
............................. 4. (x + 5) x – 9 (x + 5) E. It is prime.

............................. 5. x5 – x H. (x + 5) (x + 3) (x + 3)

............................. 6. 5 x 2 – 20 L. 2x 2 (4x – y + 6w)

............................. 7. 8 x3 – 2 x 2 y + 12 x2 w N. 7x (x – 3) (x + 3)

............................. 8. 3 x2 – x – 10 O. 5 (x – 2) (x + 2)

............................. 9. x 4 – 25 O. (x – 1) (x + 1) (x2 – x + 1) (x2 + x + 1)

............................. 10. x 6 – 1 R. (x2 + 5) (x 2 – 5)

............................. 11. 5 x3 – 40 R. (2x – 1) (4x2 + 2x + 1)


2
............................. 12. 81x 2 + 64 S. (x– 3) (x + 3)

............................. 13. 7 x3 – 63x T. 2 (x + 3) (x + 1)

............................. 14. 3x 2 – 1 U. (x – 2) (3x – 5)

............................. 15. x3 + 3x2 – 9x – 27 V. 5 (x – 2) (x 2 + 2x + 4)

Y. (x + 3)2 (x – 3x + 9)

Unscramble the letters to the first ten answers to get the first word below and unscramble the last five letters to
get the second word. Can you DECODE the message?

THIS IS A _ _ _ _ _ _ _ _ _ _ .
PRACTICE THE FIRST FIVE LETTERS, NOT THE LAST _ _ _ _ _ .

MATH 59
Example 13.Factorise: Note
(i) x - 4y
2 2
(ii) 4a x - 25b y
2 2 2 2 ..............................................
(iii) a2 - x2 - 2xy -y2 (iv) 4a2 - 4b2 + 4a + 1 Type of Method of
Solution: Polynomial factorisation
Any Look for common
(i) x2 - 4y2 = (x)2 - (2y)2 = (x + 2y)(x - 2y)
polynomial monomial factors.
(ii) 4a2x2 - 25b2y2 = (2ax)2 - (5by)2 = (2ax + 5by)(2ax - 5by) (Always do this first)
(iii) a2 - x2 - 2xy - y2 = a2 - (x2 + 2xy + y2) Example:
= a2 - (x + y)2 = (a + x + y)(a - x - y) 6x2 + 9x = 3x(2x
+ 3)
(iv) 4a2 - 4b2 + 4a + 1 = (4a2 + 4a + 1) - 4b2
Binomials of Check for special
= (2a + 1)2 - (2b)2 = (2a + 2b + 1)(2a - 2b + 1) degree 2 or product:
higher Difference of two
HCF of polynomials squares, x2 - a2
For two given polynomials, f(x) and g(x), r(x) can be taken as the highest Difference of two
common factor, if cubes, x3 - a3
Sum of two cubes,
(i) r(x) is a common factor of f(x) and g(x) and x3 + a3
(ii) Every common factor of f(x) and g(x) is also a factor of r(x).
Example:
Method for finding HCF of the given polynomials. x2 - 16 = (x - 4)(x
+ 4)
Step 1 : Express each polynomial as a product of powers of irreducible factors
x3 - 64 = (x - 4)(x2
which also requires the numerical factors to be expressed as the product
+ 4x + 16)
of the powers of primes.
x3 + 27 = (x + 3)(x2
Step 2 : If there is no common factor then HCF is 1 and if there are common - 3x + 9)
irreducible factors, we find the least exponent of these irreducible Trinomials Check for a perfect
factors in the factorized form of the given polynomials. of degree 2 square, (x ± a)2
Step 3 : 
Raise the common irreducible factor to the smallest or the least Factoring x2 + Bx
exponents found in step 2 and take their product of get the HCF. +C
Examples 14: Factoring Ax2 + Bx
(i) Find the HCF of 48x5y2 and 112x5y +C

Let f(x) = 48x5y2 and g(x) = 112x2y Example:


x2 + 8x + 16 = (x
Writing f(x) and g(x) as a product of powers of irreducible factors.
+ 4)2
f(x) = 24 × 3 × x5 × y2 x2 - 10x + 25 = (x
g(x) = 24 × 7 × x3 × y - 5)2
x2 - x - 2 = (x - 2)
The common factors with the least exponents are 24.x3 and y ∴HCF = (x + 1)
16x3y 6x2 + x - 1 = (2x +
(ii) Find the HCF of 51x2(x + 3)3 (x - 2)2 and 34x(x - 1)5 (x - 2)3 1)(3x - 1)
Let f(x) = 51x2(x + 3)3(x - 2)2 and g(x = 34x(x - 1)5 (x - 2)3 Four or Grouping
more terms
Writing f(x) and g(x) as the product of the powers of irreducible factors. Example:
f(x) = 17 × 3 × x2(x + 3)3 ×(x - 2)2 2x3 - 3x2 + 4x - 6 =
(2x - 3)(x2 + 2)
g(x) = 17 × 2 × x(x - 1)5 ×(x - 2)3

60 CLASS - IX
ALGEBRA -IV POLYNOMIALS

The common factors with the least exponents are 17, x and (x - 2)2
∴ The HCF of the given polynomials = 17 × x ×(x - 2)2
= 17x(x - 2)3

LCM of polynomials
Least Common Multiple or the Lowest Common Multiple is the product of
all the factors (taken once) of the polynomials given with their highest exponents
respectively.

Method to Calculate LCM of the given polynomials


Step 1: First express each polynomial as a product of powers of irreducible
factors.
Step 2 : C
 onsider all the irreducible factors (only once) occurring in the given
polynomials. For each of these factors, consider the greatest exponent
in the factorized form of the given polynomials.
Step 3 : Now raise each irreducible factor to the greatest exponent and multiply
them to get the LCM.
Example 15:
(i) Find the LCM of 18x3y2 and 45x5y2z3
Let f(x) = 18x3y2 and g(x) = 45x5y2z3
Writing f(x) and g(x) as a product of powers of irreducible factors.
f(x) = 2 × 32 × x3 × y2
g(x) = 32 × 5 × x5 × y2 × z3
Now all the factors (taken only once with the highest exponents are 2, 32, 5,
x5, y2 and z3.
∴ The LCM of the given polynomials = 2 × 32 × 5 × x5 × y2 and z3 =
90x y z .
5 2 3

(ii) Find the LCM of 51x2(x + 3)3 (x - 2)2 and 34x(x - 1)5 (x - 2)3
Writing f(x) and g(x) as the product of the powers of irreducible factors.
f(x) = 17 × 3 × x2(x + 3)3 × (x - 2)2
g(x) = 17 × 2 × x(x-1)5 × (x - 2)3
Now all the factors (taken only once) with the highest exponents are 2, 3, 17,
x2 (x - 1)5 , (x - 2)3 and (x+3)3.
∴The LCM of the given polynomials = 2 × 3 × 17 × x2 (x - 1)5 × (x - 2)3
× (x + 3)3
= 102 x2 (x - 2)3 (x - 1)5 (x + 3)3.

Relation between the HCF, the LCM and the product of polynomials
If f(x) and g(x) are two polynomials then we have the relation,
(HCF of f(x) and g(x)) × (LCM of f(x) and g(x)) = ± (f(x) × g(x))

MATH 61
Brain Power On
....................................................................................................................................................
GREATEST COMMONFACTOR

Find the GCF of Each pair of algebraic expressions (Some boxes might not be used)

62 CLASS - IX
ALGEBRA -IV POLYNOMIALS

Example 16:
Brain Power On Let f(x) = (x + 5)2(x - 7)(x + 8)and g(x)= (x + 5)(x - 7)2(x - 8) be two polynomials.
.............................................
Applying math ! The common factor with the least exponents are x + 5 and x - 7.
Government of India has ∴ HCF = (x + 5) (x - 7)
appointed a chairperson to lead a
All the factors (taken only once) with the highest exponents are
city beautification project. The first
act is to install statues and fountains ⇒ LCM = (x + 5)2 (x - 7)2 (x - 8)(x + 8)
in one of the city’s parks. The park Now f(x) ×g(x) = (x + 5)2 (x - 7)(x + 5)(x - 7)2 (x - 8)
is a rectangle with an area of 98x2
+ 105x −27 m2, as shown in the = (x + 5)3 (x - 7)3 (x + 8)(x - 8)
figure below. The length and width LCM × HCF = (x + 5) (x - 7)(x + 5)2 (x - 7)2 (x - 8)(x + 8) = (x + 5)3 (x - 7)3
of the park are perfect factors of the (x - 8)(x + 8)
area.
Thus, we say
(LCM of two polynomials) × (HCF of two polynomials) = Product of the two
polynomials.

Concept of square roots:


If x is any variable then x2 is called the square of the variable and for x2, x is
called the square root.
l × w = 98x2 + 105x - 27 Square root of x2 can be denoted as x 2 and –x can both be considered as
the square root of x2 because (x)(x) = x2 and (-x)(-x)=x2 .
1. Factor by grouping to find the
length and width of the park. In this study we restrict x 2 to x, i.e. +ve value of x.
2. A statue is to be placed in the
Square root of monomials
center of the park. The area of the
base of the statue is 4x2 + 12x + 9 m2. The square root of a monomial can be directly calculated by finding the square
Factor the area to find the lengths of roots of the numerical coefficient and that of the literal coefficients and then
the sides of the statue. multiplying them

3. At the northwest corner of the Example 17: Find the square root of 1296b4
park, the city is going to install a Solution:
fountain. The area of the base of the Now the given monomial is 1296b4
fountain is 9x2 − 25 m2. Factor the 1
area to find the lengths of the sides Square root of 1296b =4 1296b4 = (1296 ´ b4 )2 = 1296 ´ b4
of the fountain.
(36) ´ (b2 ) = 36 ´ b2 Þ 1296b4 = 36b2
2 2
=
.......................................................
.......................................................
81b2 a 4
....................................................... Example 18: Find the square root of .
36x 2 y 6
....................................................... Solution:
.......................................................
(ba 2 )
2
81b2 a 4 81b2 a 4 81 ´ b2 a 4 92 ´ 9ba 2 3ba 2
....................................................... = = = = =
=
36x 2 y 6 6xy 3 2xy 3
36x 2 y 6 36 ´ x 2 y 6 (xy3 )
2
....................................................... 62 ´
.......................................................
81b2a 4 3ba 2
....................................................... ⇒ Square root of = .
36x 2 y 6 2xy 3

MATH 63
Methods of finding the square roots of algebraic expressions other
than Monomials
We have three methods to find the square root of an algebraic expression
which is not a monomial. They are
(i) Method of inspection (using algebraic identities).
(ii) Method of factorization.
(iii) Method of division.
(iv) Method of undetermined coefficients.

Method of inspection:
In this method, the square root of the given algebraic expression is found by
using relevant basic algebraic identities after proper inspection.
Example 19:
Find the square root of x2 + 12xy + 36y2
Solution:
x2 + 12xy + 36y2 = (x)2 + 2(x)(6y) + (6y)2
We know that a2 + 2ab + b2 = (a + b)2
Brain Power On
Now,
2
x + 12xy + 36y = ( x ) + 2 ( x )(6y ) + (6y )
2 2 2 .............................................
The Christmas-free Field
2
( x + 6y ) = ( x + 6y ) 1.Every Christmas-tree is decorated
= with its own set of seven bulbs.
∴ x 2 + 12xy + 36y 2 = x + 6 y 2.Every tree is exactly similar to that
shown next to the field.
Method of division
3.All Christmas-trees have different
We discuss the method of division to find the square root of an algebraic sizes and may have different
expression using the following example. orientation.
Example 20: 4.No Christmas-trees overlap or
Find the square root of x - 8x + 81
2 touch each other,even at a corner.

x–9 5. Can you find and show all five


Christmas-trees?
x x2 - 18x + 81
x2
2x - 9 - 18x + 81
-18x + 81
0
\ x 2 - 18x + 81 = 0

Step 1 : F
 irst the given expression is arranged in the descending powers of x.

Step 2: T
 hen the square root of the first term in the expression is calculated. In
the above problem first term is x2 whose square root is x.This is now the
first term of the square root of the expression.

64 CLASS - IX
ALGEBRA -IV POLYNOMIALS

Step 3: Then the square of x i.e., x2 is written below the first term of the
expression and subtracted. The difference is zero. Then the next two
terms in the expression -18x +81 are brought down as the dividend for
the next step. Double the first term of the square root and put it down as
the first term of the next divisor i.e., 2(x) = 2x is to be written as the first
The radical sign, , was first used in term of the next. Now the first term – 18x of the dividend -18x +81 is
print by Christoff Rudolff in 1525. to be divided by the first term 2x (of the new divisor). Here we get -9
It is thought to be the manuscript which is the second term of the square root of the given expression and
form of the letter r (for the Latin the second term of the new divisor.
word rodix = root), although this Step 4:Thus the new divisor becomes 2x - 9. Multiply (2x - 9) by (-9) and the
is not quite conclusively confirmed. product -18x + 81 is to be brought down under the second dividend
It took a long time for to become -18x + 81 and subtracted where we get 0.
the standard symbol for a square Step 5: Thus x - 9 is the square root of the given expression x2 - 18 x
root and much longer to standardize +81.
3
,
4
,
5
and so on. The indexes of the
root were placed in every conceivable Method of undetermined coefficients
position, with
3
The method of undetermined coefficients to find the square root of an
8, algebraic expression is explained in the following examples.
3 8, and 8
3

3 Example 21:
all being variants for 8 . The
notation 16 was popular for (i) Find the square root of x4 + 4x3 + 10x2 + 12x + 9
4
16. By the 1700s, the index had Solution:
settled where we now put it.
The degree of the given expression is 4, its square root will hence be an
expression of degree 2.
Let us assume the square root to be ax2 + bx + c.
Þ x 4 + 4 x 3 + 10 x 2 + 12x + 9 = (ax 2 + bx + c)
2

2
We know that ( p + q + r ) = p2 + q 2 + r 2 + 2pq + 2qr + 2rp
2
Here, p = ax , q = bx, r = c

Þ x 4 + 4 x 3 + 10 x 2 + 12x + 9

= (ax 2 ) + (bx ) + c2 + 2 (ax 2 )(bx ) + 2 (bx )(c) + 2 (c)(ax 2 )


2 2

Þ x 4 + 4 x 3 + 10 x 2 + 12x + 9 = a 2 x 4 + b2 x 2 + 2abx 3 + 2cax 2 + 2bcx + c2

Now equating the like terms on either sides of the equality, we have
x4 = a2 x4
Þ a2 - 1 Þ a = 1
4 x 3 = 2abx 3 Þ 2ab = 4
Þ ab = 2, but a = 1 Þ b = 2
b2 + 2ca = 10 Þ 22 + 2c = 10
Þ 2c = 6 Þ c = 3
∴ The square root of the given expression is ax 2 + bx + c i.e., x2 + 2x + 3

MATH 65
66
Degree of a polynomial Value of a polynomial Zeroes of a Polynomial
The value of a polynomial p(x) at x = a is p(a).

CLASS - IX
highest power of a polynomial (a) is a zero of polynomial p(x) is a such that p(a) = 0.
Obtained on replacing x by a.

By taking out the


common factor

M I N D M A P
By grouping the
terms
POLYNOMIALS Operations Factorisation
perfect square
An algebraic expression in the form of polynomials
a0 + a1x + a2x2 + a3x3 + a4x4 + .... + By splitting the
an-1xn-1 + anxn, where a0,a1,a2,a3,a4,....an, a Middle term
are real number an≠0 and n is non
The difference of
negative integer is known as polynomial
two squares
of degree n.

Addition and subtraction Multiplication Division of polynomial HCF


Division
of of
polynomials
polynomial LCM of polynomials
Combing the like terms FOIL method by polynomial by polynomial
(i) r(x) is a common product of all the
factor of f(x) and g(x) factors (taken once)of
and the polynomials given
(ii) Every common with their highest
factor of f(x) and g(x) is exponents respectively.
also a factor of r(x).
Factor method Long division method Remainder Theorem Factor Theorem
If a polynomial p(x) is
divided by (x-a),
the remainder is p(a).
ALGEBRA -IV POLYNOMIALS

Do at Class!
1. Which of the following expressions are polynomials? 6. Classify the following as linear, quadratic and cubic
polynomicals:
1 1 1
(i) -2
+ -1 + (ii) x2(x - 1) (i) 3x (ii) y2
x x 2
(iii) 4x3 (iv) 1 + t
(iii)
1
( x - 1)( x - 2) (iv)
(x 2 + x + 1)(x + 1) (v) x + x2 + 5 (vi) x - x3
x (1 + x ) (vii) x2 + x
1 2 1
(v)
8
( x + 4 x + 6) (vi) x 2 + 2
x
7. Find the value of the polynomial 4 + x3 - x + 2x2 at
(i) x = 0 (ii) x = 1 (iii) x = -2
(vii)6 + x4 + 2x + 3x2 (viii) 5x 2 + 3 x + 2
8. Find P(0), P(1) and P(2) for each of the following
2. Write the coefficient of x2 in each of the following: polynomials
(i) 4 + 2x2 + 3x (ii) 6 - 2x2 + 3x3 + x4 (i) P(x) = 4x2 + x - 5

(iii) 𝜋x2 - x + 2 (iv) 3x - 4 (ii) P(y) = 9y2 + 2y2 + y + 7


(iii) P(z) = (z + 1)(z - 1)
3. Rewrite the following polynomials in the standard
form: (iv) P(t) = t4 + t + 1
(i) x - 7 + 8x2 + 9x3 9. 
Verify whether the indicated numbers are zeroes
(ii) -5x2 + 6 - 3x3 + 4x of the polynomial corresponding to them in the
following cases:
(iii) -4 + 6 x 3 - x + 7 x 4 - x 2 1
(i) p ( x ) = 2x + 1, x = (ii) p(x) = x2, x = 0
2
(iv) y2 + 5y3 - 11 - 7y + 9y4
10. Find the zero of the polynomial in each of the
4. Which of the following are monomials, binomials or following cases:
trinomials? (i) p(x) = x+3
(i) 1 (ii) 2 (ii) p(x) = (x + 1)(x + 2)
(iii) p(x) = x2 - 3x + 2
(iii) x3 + x - x + 2 (iv) x2 + 2x + x + 3
(iv) p(x) = 2x2 - 5x - 3
x6 x5 x4
(v) x5 + 6x4 (vi)
+ + 11. Verify whether 2 and 0 are zeroes of the polynomial
4 5 4
p(x) = x2 - 2x.
5. Find the degree of each of the following polynomials:
12. In each of the following, use reminder theorem to
(i). 1 - 2x + x6
find the remainder when f(x) is divided by g(x).
x3 + x4 - x6
(ii). (i) f(x) = 4x4 - 3x3 - 2x2 + x - 7; g(x) = x - 1
x2
(iii). x3(x4 + 1) (ii) f(x) = 2x4 + 6x3 + 2x2 - x + 2; g(x) = x + 2
3 2
(iv). 5x 4 - 8x + 4 x + 3x - 5 x2 2 æ 2ö
3 7 2 (iii) f ( x )= 3x 4 + 2x 3 - + ; g( x ) = çç x + ÷÷÷
3 27 çè 3ø
x4 (iv) f(x) = x3 - 6x2 + 2x - 4; g(x) = 1 - 3x
(v). - x 3 + 2 2x + 3
4
13. Find the value of k if the division of kx3 + 9x2 + 4x - 10
(vi). a2x4 + ax2 + x + 1 by x+3 leaves a remainder -22.
1 3
(vii).
x2
(x - x4 + x8 ) 14. If x3 + px2 + qx + 6 leaves reminder 3 when divided
by x-3 and leaves zero when divided by x-2. Find
values of p and q.
MATH 67
15. The polynomial kx4 + 3x3 + 6 when divided by x-2 27. Factorise the following:
leaves a remainder which is double the remainder
(i). a2 + 6b + 3a + 2ab
left by the polynomial 2x3+17x+k when divided by
x-2. Find the value of k. (ii). 3b2 + bc + 15ab + 5ac

16. Let R1 and R2 be the remainders when the (iii). 2axy2 + 10x + 3ay2 + 15
polynomials f(x) = x3 + 2ax2 - 5x - 7 and (iv).4abc2 + 2a2cd + abd2 + 2b2cd
g(x) = x3 + x2 - 12x + 6a are divided by (x+1) and
(x-2) respectively. If 2R1+R2 = 12, find the value (v). (a2 + 5a)2 + 4a2 - 2a2b - 10ab + 20a
of ‘a’. (vi). ab(c2 + 1) + c(a2 + b2)
17. If (x) = x4 - 2x3 + 3x2 - ax + b is a polynomial (vii). x2 - xy + ay - ax + 4x - 4y
such that when it is divided by (x-1) and (x+1) the
(viii). ap - 2ar + 2p + aq - 4r + 2q
remainders are respectively 5 and 19. Determine
the values of a and b. (ix). x(x + 2)(x + 4) + 4x + 8
18. Show that (x-1) is a factor of (x10 - 1) and also of (x). a2 + b2 + c2 + 2ab + 2bc + 2ca
(x11 - 1).
28. Factorise the following:
19. Determine the value of ‘a’ if (x-a) is a factor of the
(i). x2 - 4y2
polynomial x3 - (a2 - 1)x + 2
(ii). x2 - y2 - 2x + 1
20. Use factor theorem to determine the value of k for
which x + 2 is a factor of (x + 1)2 + (2x + k)3 (iii). x2 - y2 + 6y - 9
21. 
Find the value of k if (x – 3) is a factor of (iv).9a2 + 6a + 1 - 36z2
k2x3 - kx2 + 3kx - k (v). 4a2 - (2b - c)2
22. 
Find the values of m and n in the polynomial (vi). 3x3y - 243xy3
2x3 + mx2 + nx - 14 such that (x-1) and (x+2) are
its factors. (vii). (x + 1)4 - (x - 1)4

23. Find the values of ‘a’ and ‘b’ if x2 - 4 is a factor of (viii). 16x2 - 24yz - 9y2 - 16z2
ax2 + 2x3 - 3x2 + bx + 4 (ix). a2 - b2 - 4c2 + 4d2 - 4(ad - bc)
24. Using factor theorem, show that x-y, y-z, z-x are 2 1
(x). x + 2 - 3
the factors of x2(y - z) + y2(z - x) + z2(x - y) x
(xi). x4 + 4x2 + 3
25. Using factor theorem, show that a+b, b+c, c+a are 1
the factors of (a + b + c)3 - (a3 + b3 + c3) (xii). x 4 + 4 + 1
x
26.Factorise the following 29. Evaluate
(i). 4x + 8x y
2 2 (i) [(999)2 - (1)2]
(ii). 4a3b2 + 6a2b2 + 12ab (ii) [(18.875)2 - (1.125)2]

(iii). 36a2 + 48ab + 24b2 I. Choose the correct answer:


(iv). 6x(y + z) - 8y(y + z) 1. If the degree of a polynomial AB is 15 and the
(v). 2x(m - n) + 4y(4m - 4n) - 6z(n - m) degree of polynomial B is 5, then the degree of
polynomial A is ......................
(vi). xy(x + 4y + z)2 - 4xy(x + 4y + z)
(a) 3 (b) 8
(vii). x(x - 2z) + y(x - 2z) + (2z - x)
(c) 4 (d) 10
(viii). (2x-3)2 - 8x + 12 2. The expression 21x2 + 11x - 2 equals to ..................
(ix). x4 - x3y + x2y2 (a) (x-2)(7x+1) (b) (7x+1)(3x-2)
(x). 2a b - 4a b + 8ab - 16b
3 2 2 3 4 5 (c) (7x-1)(3x-2) (d) (7x-1)(3x+2)

68 CLASS - IX
ALGEBRA -IV POLYNOMIALS

3. 
If the LCM and HCF of two polynomials are 2 1
90m5a6b3x2 and m3a5 respectively and also one 12. The square root of y + + 2 is ..................
y2
of the monomial is 18m5a6x2, then the other
monomial is .................. 1 1
(a) y + (b) y -
y y
(a) 5m3a5b3 (b) 15m5a3b2
1 1
(c) 5m5a3b5 (d) 15m3a5b4 (c) y 2 + (d) y 2 -
y2 y2
4. The remainder when x3 - 3x2 + 5x - 1 is divided
13. The product of the polynomials 2x3 - 3x2 + 6 and
by x + 1 is ..................
x2 - x is ............
(a) – 8 (b) – 12
(a) 2x6 - 5x4 + 3x3 + 6x2 - 6x
(c) – 10 (d) – 9
(b) 2x5 - x4 + 3x3 - 6x2 + 6x
5. 
Which of the following is a homogeneous (c) 2x5 - 5x4 + 3x3 + 6x2 - 6x
expression? (d) None of these
(a) 4x2 - 5xy+5x2y+10y2 14. The LCM of x2 - 16 and 2x2 - 9x + 4 is ..............
(b) 5x+10y+100
(c) 14x3 + 15x2y + 16y2x + 24y3 (a) (2x + 1)(x + 4)(x - 4)
(d) x2 + y2 + x + y + 1 (b) (x2 + 16)(2x + 1)
(c) 2(1 - 2x)(x + 4)(x - 4)
6. å x (y 3
- z 3 ) = ..................... (d) (2x - 1)(x + 4)(x - 4)
(a) (x - y)(y - z)(z - x)(x + y + z) 15. If P = 3x3-5x+9, Q = 4x3 + 5x2 - 11 and R = 5x3
(b) (x - y)(y - z)(x - z)(x - y - z) + 4x2 - 3x + 7 , then P – 2Q + R is ...................
(c) (x + y)(y + z)(z + x)(x + y + z)
(a) 2(3x2 + 4x - 19) (b) -6x2 - 5x + 38
(d) (x + y)(y + z)(z + z)(z - y - z)
(c) -2(3x2 + 4x + 19) (d) -2(3x2 + 4x - 19)
7. The remainder when f(x) = 4x3 - 3x2 + 2x - 1 is
divided by 2x + 1 is ................... 16. If g(x) = 3ax + 7a2b - 13ab2 + 9by is a homogeneous
-3 expression in terms of a and b, then the values of
(a) 1 (b) x and y respectively are .............................
4
-13 -7 (a) 2, 2 (b) 2, 1
(c) (d)
4 4 (c) 3, 2 (d) 3, 3
8. The HCF of the polynomials 12a3b4c2,18a4b3c3
17. The polynomial 11a 2 - 12 2a + 2 on factorization
and 24a6b2c4 is ..................
gives
(a) 12a3b2c2 (b) 6a6b4c4
(c) 6a3b2c2 (d) 48a6b4c4 (
(a) 11a + 2 a - 2 )( ) ( )(
(b) a - 2 11a - 2 )
9. Find the value of a, if (x + 2) is a factor of the (
(c) (a + 11) a + 2 ) (d) (11a - 2 )(a + 2 )
polynomial f(x) = x3 + 13x2 + ax + 20
18. If xn + 1 is divisible by x + 1, n must be
(a) –15 (b) 20 ......................
(c) 25 (d) 32
(a) any natural number
10. The polynomial on factorization gives (b) an odd natural number
(a) (x - 4)(x2 - 1) (b) (x - 4)(x2 + 4) (c) an even natural number
(c) (x + 4)(x2 + 1) (d) (x - 4)(x2 + 1) (d) none of these
19. What is the first degree expression to be subtracted
If the expression ax3 + 2x2y - bxy2 - 2y3 is
11. 
from x6 + 8x4 + 2x3 + 16x2 + 4x + 5 in order to
symmetric, then (a, b) = ..................
make it a perfect square?
(a) (2, 2) (b) (–2, 2)
(a) –4x – 4 (b) 4x + 4
(c) (–2, –2) (d) (2, –2)
(c) 4x – 4 (d) –4x + 4
MATH 69
26. If 3x - 1 is a factor of the polynomial 81x3 - 45x2
m n n m a (m + n )
2 2

20. Find the square root of 2 . + 3a - 6 , then a is ...............


(m + n )
(m + n ) -7
8
n2 m2
(a) (b)
m +n
3 3
m +n m n a
2 2 2
(a) (b) 11
m n nm a 2
(m + n )
2
(c) -10 (d)
(m + n ) 2 3 3
m +n
m n nm a 27. If A = 4x3 - 5x + 7, B = 2x3 - x2 + 3 and C = 5x3
(c) (m + n )
(d) None of these
(m + n ) - 8x2 + 10 , then A – 2B – C is .............
(a) 5x3 - 2x2 + x + 4 (b) -5x3 + 10x2 - 5x - 9
21. What is the first degree expression to be added to (c) x3 + 10x2 - 5x + 9 (d) 5x3 - 8x2 + x - 1
16x6 + 8x4 - 2x3 + x2 + 2x + 1 order to make it a
m1 -n 2 2
+2 mn 2
perfect square? 28. The square root of x ´ xn ´ x n is .......
5 15 5 15
(a) x+ (b) - x - (a) x(m +n)2 (b) x(m +n)/2
2 16 2 16 1 2 1
5 15 2 15 (c) 2(m +n) (d) (m + n )
2

(c) - x + (d) + x - x x2
2 16 2 16 29. x831 + y831 is always divisible by...................
1 (a) x – y (b) x2 + y2
22. Factorize the polynomial 8x 3 - . (c) x + y (d) None of these
64
æ 1 öæ 2 x 1ö 30. If (x + 1)(x + 2)(x + 3)(x + k) is a perfect square,
(a) ççç2x - ÷÷÷ççç4 x - + ÷÷÷
è 4 øè 2 16 ø then the value of k is ........
æ 1 öæ x ö (a) 4 (b) 5
(b) çç2x - ÷÷çç4 x 2 + - 16÷÷
çè ÷
8 øèç 2 ÷ø (c) 6 (d) 7
æ 1 öæ 1 xö
(c) çç2x - ÷÷çç4 x 2 + + ÷÷ 31. If A=6x4 + 5x3 - 14x2 + 2x + 2 and B = 3x2 - 2x
çè 4 ÷øçè 16 2 ÷ø - 1 , then the remainder when A ÷ B is ..............
æ 1 öæ x ö (a) x (b) 2x
(d) çç2x - ÷÷÷çç4 x 2 + - 16÷÷÷
çè 4 øçè 2 ø (c) 3x (d) 4x

23. The product of polynomials 3x3 - 4x2 + 7 and x2 + 1 The polynomial x5 - a2x3 - x2y3 + a2y3 on
32. 
is ................ factorization gives ............
(a) 3x5 - 4x4 + 3x3 + 3x2 + 7 (a) (x - y)(x - a)(x + a)(x2 + y2 + xy)
(b) x5 + 4x2 - 2x + 3 (b) (x + a)(x - y)(x - a)(x2 - y2 + xy)
(c) 3x5 - 4x4 - 3x3 + 4x + 8 (c) (x + a)(x + y)(x - a)(x2 + y2 + xy)
(d) 3x5 - 5x4 + 8x2 + 2x + 1 (d) None of these
24. The LCM and HCF of two monomials is 60x4y5a6b6 33. The HCF of the polynomials x4 + 6x2 + 25, x3 -
and 5x2y3 respectively. If one of the two monomials 3x2 + 7x - 5 and x2 + 5 - 2x is ...................
15x4y3a6, then the other monomial i .................
(a) x2 - 2x - 5 (b) x2 - 2x + 5
(a) 12x2y3a6b6 (b) 20x4y5b6 (c) x - 1 (d) 3x + 2
(c) 20x2y5b6 (d) 15x2y5b6
34. The HCF of the polynomials (2x - 1)(5x2 - ax + 3)
25. Which of the following is a factor of the polynomial and (x - 3) (2x2 + x + b) is (2x - 1)(x - 3). Then
f(x) = 2x3 - 5x2 + x + 2? then values of a and b respectively are .............
(a) x + 1 (b) x + 2 (a) 16, –1 (b) –16, 1
(c) 2x +1 (d) 2x - 1 (c) –16, –1 (d) 16, 1

70 CLASS - IX
ALGEBRA -IV POLYNOMIALS

35. The remainder when x45 is divided by x2 – 1 is ...... 43. Find the value of
2 2 2
(a) 2x (b) –x (a + b) ( b + c) (c + a )
+ +
(c) 0 (d) x (b - c)(c - a ) (a + b)(c - a ) (a - b)(b - c)
(a) –1 (b) 0
36. Factorize å a (b
a ,b,c
2 4
- c4 )
(c) 1 (d) 2
(a) (a-b)2(b-c)2(c-a)2 44. Find the square root of the expression
(b) (a - b)(a + b)(b - c)(b + c)(c - a)(c + a) 1 æ 1 1 1ö
(c) (a + b)2(b + c)2(c + a)2 ( x 2 + y 2 + z 2 ) + 2 çç + + ÷÷÷
xyz çè x y z ÷ø
(d) None of these
x+y+z
37. The polynomial 6y4 - 19y3 - 23y2 + 10y + 8 on (a)
xyz
factorization gives
yz zx xy
(a) (y + 1)(y - 4)(3y + 2)(2y + 1) (b) + +
x y z
(b) (y + 1)(y - 4)(3y - 2)(2y - 1)
(c) (y + 1)(y - 4)(3y - 2)(2y + 1) (c) x+ y+ z
(d) (y + 1)(y - 4)(3y + 2)(2y - 1)
x y z
(d) + +
38. If the LCM of the polynomials and (y - 3) (2y+1) d b yz xz xy
(y + 13)7 and (y - 3)4 (2y + 1)9 (y + 13)c is (y - 3)6 4 1
45. Factorize the expression 9x + +2 .
(2y + 1)10 (y + 13)7 , then the least value of a + b x4
+ c is ................ æ 1 öæ 1 ö
(a) çç3x 2 - 2 + 2÷÷÷çç3x 2 + 2 + 2÷÷÷
(a) 23 (b) 3 çè x øèç x ø
(c) 10 (d) 16 æ 1 öæ 1 ö
(b) çç3x 2 - 2 - 2÷÷çç3x 2 + 2 + 2÷÷
çè x ÷
øèç x ÷ø
39. The LCM of the polynomials 195(x + 3)2 (x - 2) (x
+ 1)2 and 221(x + 1)3(x + 3)(x + 4)is .................. æ 1 öæ 1 ö
(c) çç3x 2 - 2 + 2÷÷÷çç3x 2 - 2 + 2÷÷÷
çè x øèç x ø
(a) 221(x + 3)2(x + 1)2(x - 2)(x - 14)
(b) 13(x + 3)(x + 1)2 æ 1 öæ 1 ö
(d) çç3x 2 + 2 + 2÷÷çç3x 2 + 2 - 2÷÷
(c) 3315(x + 3)2(x + 1)3(x - 2)(x + 4) çè x ÷øèç x ÷ø
(d) None of these
II. Application of polynomials:
40. For what value of k the HCF of x2 + x + (5k - 1)
and x2 - 6x + (3k + 11) is (x - 2)? 1. Imagine that we are trying to find the area of a
lawn so that we can determine how much grass
(a) 1 (b) 2 seed to purchase. The lawn is the green portion as
(c) –2 (d) –1 shown in the figure. Express the are of the lawn in
factored form.
41. The HCF of the polynomials 9(x + a)p (x - b)q (x +
c)r and 12(x + a)p + 3 (x - b)q - 3 (x + c)r + 2 is then 4 4
the value of p + q – r is ...................
(a) 21 (b) 9 4 4
(c) 15 (d) 6 6x
42. The remainders obtained when the polynomial x3 4
+ x2 - 9x - 9is divided by x, x + 1 and x + 2
respectively are ................
(a) –9, 0, –15 (b) –9, –16, 5 10x
(c) 0, 0, 5 (d) –9, 0, 5

MATH 71
Do at Home!
1. Which of the following expressions are polynomials? (iv) p(x) = x3 - x2 - x - 1
1 1 (v) p(y) = 3y3 - 3y2 + 3y + 1
(i) x 2 + - 2 (ii) 2x 3 - x 2 + x 2 + 3
x 9. Verify whether the indicated numbers are zeroes of
3 2 2 5 6 -1
(iii) y + y + y + 3 (iv) y + y + y the polynomial
1
(i) p(x) = 2x - 1, x=
2. Write the coefficient of x2 in each of the following: 2
(ii) p(x) = x2 + x + 2x3, x=0
(i) 2 + x2 + 3x (ii) 3x2 + x - 7 (iii) p(x) = 2x3 - x2 + x - 2, x = 1
(iii) (x + 2)(x - 3) (iv) (x2 + 2x + 3)(x - 2) (iv) p(x) = x2 - x - 2, x=2
(v) (x + y)(x2 - x - 2)
10. 
Find the zero of the polynomial in each of the
3. Rewrite the following polynomials in the standard
following cases:
form:
(i) p(x) = x - 2 (ii) p(x) = x2 - 2x + 1
(i) 2 - x - x3 - 2x2 (ii) x3 - x4 + 2x2 + 7
(iii) p(x) = (x + 3)(x - 2) (iv) p(x) = 3x2 - 5x + 2
(iii) 3 - 2x + x - 7x (iv) x - 7x2 + 5x3 + 10
2 3
(v) p(t) = (t - 2)(2t - 3)
(v) 5x3 - x5 + 2x2 - x4 + 2
11. Verify whether 3 and 0 are zeroes of the polynomial
4. Which of the following are monomials, binomials or
p(x) = x2 - 2x - 3.
trinomials?
(i) x2 + x - 2 (ii) x3 - 2x2 - 1 12. In each of the following, use reminder theorem to
x3 find the remainder when f(x) is divided by g(x).
(iii) x - 2 (iv)
2x (i) f(x) = x3 + 4x2 - 3x + 10; g(x) = x + 4
(v) x + 2x + 2
6 2
æ 1ö
(ii) f ( x ) = 4 x 3 - 12x 2 + 11x - 3; g( x )= çç x + ÷÷÷
çè 2ø
5. Find the degree of each of the following polynomials: (iii) f(x) = 9x3 - 3x2 + x - 5; g(x) = 3x - 2
(i) x - 2 + 3x 2
(ii) 5x - 3x + 2x + 3x
2 3 4
13. The polynomial p(x) = kx3 + 15x2 + 9x - 40 when
(iii) x 4 + 2 x 2 + 3x (iv) x3 - 2x + 3 divided by (x+4) leaves a remainder -28, find the
value of k.
x10 - x12
(v) (vi) x3 - xy + x2y2 + 3x 14. The polynomials ax3 + 3x2 - 13 and 2x3 - 5x + a
x2
are divided by x-2. The remainder in each case is
6. Classify the following as linear, quadratic, cubic and the same. Find the value of a.
Bi quadratic polynomicals:
15. Let R1 and R2 be the remainder when
(i) 2x3 + 3 (ii) x - 3x2 + x3
the polynomials x3 + 2x2 - 5ax + 7 and
(iii) x + 3 (iv) x2 - x + 2
x3 + ax2 - 12x + 6 are divided by (x+1) and (x-2)
(v) x - 2x + x + x - 3
4 3 2
respectively. If R1-R2 = 20, find the value of a.
7. Find the value of the polynomial p(x) = x3 - 2x2 + 3x - 1
at 16. The polynomials ax3 + 3x2 - 3 and 2x3 - 5x + a
when divided by (x-4) leave the remainders R1 and
(i) x = 1, (ii) x = -1 (iii) x = 2 R2 respectively. Find the value of a if
8. Find p(0), p(1) and p(-1) for each of the following (i) R1=R2 (ii) 2R1-R2=0
polynomials
17. Determine whether g(x) is a factor of f(x)
(i) p(x) = x5 - 2x2 + x - 1
(i) f(x) = x3- 3x2 + 4x - 4, g(x) = x – 2
(ii) p(y) = y - y3 + y2 - 10
(iii) p(t) = t4 - 3t3 + 2t2 - t - 1 (ii) f ( x )= 7x 2 - 2 8x - 6, g( x ) = x - 2

72 CLASS - IX
ALGEBRA -IV POLYNOMIALS

18. S how that x-2, x+3 and x-7 are factors of 28.Factorise the following
x 3 - 6x 2 - 13x + 42
(i). 100 - 9z2
19. Use factor theorem to verify that (x+a) is a factor (ii). x3 - x
of xn + an. (iii). 25x2 - 60xz + 36z2
20. Without actual division, prove that x4 + 2x3 + 2x - 3 (iv).x2 - 1 - 2a + a2
is exactly divisible by x2 + 2x - 3 (v). 18a2x2 - 32
(vi). 16x4 - 1
21. Find the value of ‘a’ so that (x + 1) may be a factor
of 2x3 - ax - (2a - 3)x + 2 (vii). x2 + 4y2 - z2 - 4xy
(viii). 4x2 - z2 + 9y2 - 4p2 + 4pz - 12xy
22. If ax3 + bx2 + x - 6 has (x+2) as a factor and
(ix). x3 - 5x2 - x + 5
leaves a remainder 4 when divided by (x-2), find
the values of a and b. (x). x4 + 5x2 + 9
(xi). x8 - y8
23. 
What must be subtracted from
4
x - 6x - 15x + 80 so that the result is exactly
3 2
(xii). x 4 +
divisible by x2 + x - 12? x4

24. Find α and β, if (x+1) and (x+2) are factors of I. Very short answer type questions:
x3 + 3x2 - 2αx + β
1. 11x2 - 88x3 + 14x4 is called a .............. polynomial.
25. Using factor theorem, show that x-y, y-z, z-x are
the factors of x(y2 - x2) + y2(z2 - x2) + z(x2 - y2) 2. The degree of the polynomial 7x3y10z2 is ...............

26.Factorise the following 3. The expression is a polynomial. (True/False)

(i). 5xy - 15x3y3 4. If A = 3x2 + 5x - 3 and B = 5x2 - 7 then 2A – B


(ii). m4n4 + 3m3n3 - m2n4 is ...............
(iii). x3(p + q) + y4(p + q) 5. If a + b + c = 0, then a3 + b3 + c3 = ...............
(iv). (2x + 3y) - 5(2x + 3y)
2
6. Factors of x6 - y6 is ...............
(v). ab(a - b + c) + bc(2a - 2b + 2c) + ca(3a - 3b + 3c)
(vi). x(x - y)4 + 3xy(x - y)2 7. The LCM of 2x 3 , 8x 7 y 2 is ...............
(vii). x2(x - 2y) + 3y2(x - 2y) + z(2y - x) 8. The HCF of 44a3 and 66bpa4 is 22a3, then p can be
(viii). (3a - 1) - 6a + 2
2
...............
(ix). ax2y2 - axyz + a2xz2
9. Is .............. (where x > 0)
(x). (a - b)4 + (b - a)3
10. The quotient of when divided by x + 2 is ...............
27.Factorise the following
(i). xy + 3bx + 2ay + 6ab 11. The remainder obtained when 80x3 + 55x2 + 20x
+ 172 is divided by x + 2 is ...............
(ii). 4ax + 2ay + 2bx + by
(iii). 3pm + 4qm + 3pn + 4qn 12. Factorize. 6x2 + x - 2
(iv).2a2px + 2qz - 4apy + 4pz - 2aqy + a2qx 13. 
Find the LCM and HCF of the polynomials
(v). 2a2c + 2b2c - ab + 4abc2 15x2y3z,3x3yz2 .
(vi). l + x + y - z + xy - yz - zx - xyz
14. Find the remainder when x15 is divided by x - 2.
(vii). ax - ay - 3az + 2bx - 2by - 6bz
(viii). ax - 2bx + 2cx + 6a - 12b + 12c 15. Find the remainder if x5 - 3x3 + 5x + 1 is divided
by 2x - 1 .
(ix). (x + 2)(x2 + 25) + 10x2 + 20x
(x). a3x - a2b(x - y) - ab2(y - z) - b3z 16. a + b - 2 ab is .............. where a> b .

MATH 73
17. The product of two symmetric expressions is a/an 37. The LCM of the polynomials (x2 + x - 2)(x2 + x - a)
.............. expression. and (x2 + x - b)(x2 + 5x + a) is (x + 1)(x + 2)2
2 2 (x + 3) , then find the values of a and b.
18. The square root of a m × bn is ...............
38. Find the remainder when x23 is divided by x2 - 3x + 2.
19. The value of a if x3 - 8x2 + 2x + a is divisible by
x - 2 is ............... 39. If lmx2 + mnx + ln is a perfect square then prove
that, 4l3 = mn .
20. Factorize : a5b - ab5.
40. Find the value of
21. 
The degree of a polynomial A is 7 and that of
(a + b + c) + (a + b - c) + 2 (c2 - a 2 - b2 - 2ab) .
2 2
polynomial AB is 56, then find the degree of
polynomial B.

22. If A = x3, B = 4x2 + x - 1 then find AB. 125a 6 b4 c2 x2


41. If 4 2
= x, then find .
5a b abc
23. Factorize : m + m .
7 4

1 4 42. Find the square root of (x2 + 6x + 8)(x2 + 5x + 6)


24. Factorize: a 2 - a + .
6 3 (x2 + 7x + 12) .
25. If 3x + 8ax + 3 is a perfect square, then find the
2

value of a. III. Verbal math:


26. The factors of a3 + b3 + c3 are ............... 1. If the terms of a polynomial do not have a GCF,
27. The HCF of (a2 + 1)(a + 11) and (a2 + 1)2(a + 11)2 does that mean it is not factorable? Explain.
is ...............
2. A polynomial is factorable, but it is not a perfect
II. Short answer type questions: square trinomial or a difference of two squares.
Can you factor the polynomial without finding the
28. The HCF of (a - 1)(a3 + m) and (a + 1)(a3 - n) and GCF?
(a + 1)(a2 - n) is a2 - 1 , then the values of m and
n are ............... 3. How do you factor by grouping?

29. Expand: Õa
a , b, c
2
( b + c) . 4. Is there a formula to factor the sum of squares?
The factors of (a - b)3 + (b - c)3 + (c - a)3 is
30.  5. In multiplication of polynomials,does the order of
............... the factors matter?
31. Expand å c (a 2 2
- b2 ) .
6. Can every trinomial be factored as a product of
32. If A = 4x - 8x , B = 7x - 5x + 3 and C = 3x +
3 2 3 3 binomials?
x - 11 then find 2A - 3B + 4C.
7. 
Explain why the degree of the product of two
33. If A = x3, B = 4x2 + x - 1, C = x + 1 then find nonzero polynomials equals the sum of their
(A – B) (A – C). degrees.
34. Find the quotient and remainder when x4 + 4x3 - 8. Can negative exponents be factored ? Discuss.
31x2 + 94x + 120 is divided by x2 + 3x - 4 .
3ax x 3 1 9. When dividing a polynomial by x - c do you prefer
35. Factorize : a 3 + + - . to use long division or synthetic division? Does the
8 64 8
value of c make a difference to you in choosing?
36. 
Find the LCM and HCF of the following
Give reasons.
polynomials.

36(x + 2)2 (x - 1)3 (x + 3)5, 45(x + 2)5 (x - 1)2 10. Find out where do we apply polynomials in a real
(x + 3)5 and 63(x - 1)5 (x + 2)5 (x + 3)4. life?

74 CLASS - IX
ALGEBRA
BASICS OF-IV
TRIGONOMETRY
POLYNOMIALS

Chapter 3 Learning Checklist


Right angle Triangle
Trigonometric ratios
BASICS OF Angle of elevation and depression
Standard angles
TRIGONOMETRY Trigonometric ratios of complementary
angles

What is new about trigonometry?


The word “trigonometry” is a Greek word. It means “measurement of
a triangle”. Therefore trigonometry is that branch of mathematics concerned
with the measurement of sides and angle of a plane triangle and the investigations
of the various relations which exist among them.
Today the subject of trigonometry also includes another distinct branch which
concerns itself with properties relations between and behaviour of trigonometric
functions. The importance of trigonometry will be immediately realized when
its applications in solving problem of mensuration, mechanics physics, surveying
and astronomy are encountered.

d=?
𝛼
40˚ 𝛽
79.9˚ 25˚ A
87 feet B
100ft

Triangles
A triangle is a geometric shape with three sides and three angles. Triangle
types are defined by angle and length properties. It is possible to calculate the
missing angles of a triangle easily if you know what type it is.
Type of Triangles
By Side By Angle

Equilateral triangle Acute triangle has


has three equal sides three angles < 90°

Isosceles Triangle Right triangle has


has two equal sides one angle = 90°

Scalene Triangle has Obtuse triangle has


no equal sides one angle > 90°

MATH 75
Labelling a right-angled triangle
Brain Power On
Labelling a triangle In order to help define the trigonometric ratios it is
.............................................
important that you understand how to correctly identify the sides of a triangle in
relation to its angles. Let’s start by drawing a right-angled triangle. How many triangles are there in
this big triangle?
A right triangle has two shorter sides, or legs, and the longest side, opposite
the right angle, which is always called the hypotenuse. The two shorter
sides have some other special names, too, based on which acute angle of the
triangle you happen to be working with at a particular time.
In reference to acute angle θ, the leg on the other side of the triangle from
θ is called the opposite side. That opposite side is never along one of the rays
making up the angle.
𝜃
hypotenuse hypotenuse
adjacent opposite

𝜃
opposite adjacent

The other leg in the right triangle is then called the adjacent side. Adjacent
means “next to,” and in the case of right triangles, the adjacent side helps form
the acute angle along with the hypotenuse because it lies along one of the angle’s
rays.
Example 01: For the triangle shown, state which sides are: (a) the hypotenuse
(b) the adjacent (c) the opposite.
C

Solution:
(a) The hypotenuse is the longest side, which for this triangle is CB.
(b) The adjacent is the side that is next to the angle θ , which for this triangle
is AB.
(c) The opposite side is the side that is opposite the angle θ , which for this
triangle is AC. Note
..............................................
Pythagoras' Theorem
The following Greek letters are
If the length of the hypotenuse of a right-angled triangle is c and the lengths
used generally in trigonometry to
of the other sides are a and b:
indicate in a general way the number
of degrees in angles.
c
a (1) (theta) (2) (phi)
(3) (alpha) (4) (beta)
b (5) (gamma) (6) (delta)
(7) (lamda) (8) (omega),etc
c = a + b and also a = c - b and b = c2 - a2
2 2 2 2 2 2 2

76 CLASS - IX
ALGEBRA
BASICS OF-IV
TRIGONOMETRY
POLYNOMIALS

To find the hypotenuse, add the squares of the other sides, then take the
Brain Power On square root.
............................................. To find a shorter side, subtract the squares of the other sides, then take the
The last straw square root.
It you take three drinking straws, Example 02: Find the length of the side of the triangle marked x in the diagram.
you can easily arrange them into one
triangle that has three equal sides.
Now, suppose you have six drinking 6 cm
x
straws. Can you arrange them into
four triangles, all with equal sides?

3 cm

Solution:
As this is a right angled triangle, Pythagoras' Theorem can be used. Here the
length of the hypotenuse is 6 cm, so a = 6 cm, c = 3 cm and b = x
So a2 = b2 + c2
62 = x2 + 32
36 = x2 + 9
36 - 9 = x2
27 = x2
27 = x
x = 5.2 cm
Example 03: A ladder 13 m long is placed on the ground in such a way that it
touches the top of a vertical wall 12 m high. Find the distance of the foot of the
ladder from the bottom of the wall.

12 m 13 m
wall ladder

ground
Solution:
Let the required distance be x meters. Here, the ladder, the wall and the ground
from a right-angled triangle. The ladder is the hypotenuse of that triangle.
Note
.............................................. According to Pythagorean Theorem,
Pythagorean Triple x2 + 122 = 132
x2 = 132 – 122
A Pythagorean triple consists of
three positive integers a, b, and c, x2 = (13 + 12)(13 – 12)(algebraic identity)
such that a2 + b2 = c2. Such a triple x2 = (25)(1)
is commonly written (a, b, c), and a x2 = 25
well-known example is (3, 4, 5). If (a, x= 25
b, c) is a Pythagorean triple, then so is
x = 5 cm
(ka, kb, kc) for any positive integer k.
Therefore, distance of the foot of the ladder from the bottom of the wall = 5 meters

MATH 77
Example 04: The sketch shows part of a rollercoaster ride between two points,
A and B at the same horizontal level. The highest point in this section, C, is 20
metres above AB, the length of the ramp is 55 metres and the length of the drop
is 30 metres. Find the horizontal distance between A and B.
C

ramp
drop
55 m
30 m
20 m

A D B

Solution:
Using Pythagoras in triangle ACD gives
AD2 = 552 - 202 (AD is one of the short sides of the triangle, so subtract)
AD = 2625 = 51.23 m
Using Pythagoras in triangle BCD gives
DB2 = 302 - 202 (BD is one of the short sides of the triangle)
DB = 500 = 22.36 m
AB = AD + DB = 73.5 m
The horizontal distance between A and B is 74 metres

Trigonometric Ratios

hypotenuse
y
Using trigonometry, the
British geographers calculated the
x
altitude of Mount Everest to be
Sine
29,002 feet. In 1999, scientists used
In a right-angled triangle the sine of an angle is defined as:
a GPS transmitter to measure the
length of side opposite to the angle
sine of an angle = height of the peak and found that the
length of the hypotenuse
altitude was actually 29,035 feet.
As the hypotenuse in a triangle is always the longest side, the value for the That means that the measurement of
sine of an angle is never bigger than 1. In mathematical writing, the word
“sine” is shortened to “sin” and the angle of interest is written after for example the first geographers was only 33 feet
sin . There is no multiplication here and the text “ sin ” implies the sine of the off!
angle . Applied to the triangle above:
y x
sin q = sin f=
hypotenuse hypotenuse
Cosine
In a right-angled triangle the cosine of an angle is defined as:
length of side adjacent to the angle
cosine of an angle =
length of the hypotenuse

78 CLASS - IX
ALGEBRA
BASICS OF-IV
TRIGONOMETRY
POLYNOMIALS

The value of the cosine of an angle is never bigger than 1, for the same
Brain Power On reason as the sine.When writing mathematically the word “cosine” is shortened to
............................................. “cos” and, as with the sine, the angle of interest is written after, for example cos .
Missing squares Again there is no multiplication here and the text “ cos ” implies the cosine of
the angle . Applied to the triangle above:
The four coloured pieces can be
x y
put together in two different ways cos q = cos f=
to make these shapes with base 13 hypotenuse hypotenuse
units and height 5 units.Why is there
one square missing in the second Tangent
arrangement? In a right-angled triangle the tangent of an angle is defined as:
length of side opposite to the angle
tangent of an angle =
length of side adjacent to the angle
You may notice that the tangent represents the gradient of a line (usually
defined as the change in y divided by the change in x). The word “tangent” is
shortened to “tan” with tan denoting the tangent of the angle , again there is
no multiplication involved. Applied to the triangle above:
y x
tan q = tan f =
x y

Interestingly you can use the definitions sine and cosine above to show that:
sin q sin f
tan q = tan f =
cos q cos f

Fundamental Identities
Reciprocal Identities
1 1 1
Note csc q = sec q = cot q =
.............................................. sin q cos q tan q

1. The student is warned against Quotient Identities


regarding sin as sin x .‘sin ’ means sin q cos q
‘sine of angle ’. It would be wrong tan q = cot q =
cos q sin q
to write
sin x + sin y = sin (x + y)
Example 05: Write down the values of sin , cos , tan and for the given
sin + cos = (sin + cos) etc.
triangle.
2. (sin )2 is written as sin2 and is
read as ‘sin square ’. (sin )3 written
as sin3 and is read as ‘sin cube ’. 10
8
Similarly, for powers of other t-ratios,
such as
(tan )2 = tan2 , (cos )3 = cos3 ,
(tan )3 = tan3 , (cos )2 = cos2 , 6
(cos )5 = cos5 , (tan )4 = tan4 , etc. Solution:
There is only one exception.(sin x) –1
Opposite = 8
is not written as sin–1 . It has a Adjacent = 6
different meaning altogether. Hypotenuse = 10

MATH 79
opposite adjacent opposite Note
sin q = cos q = tan q = ..............................................
hypotenuse hypotenuse adjacent
8 6 8 When writing the values of the
= = = trigonometric functions, do not
10 10 6
forget the argument of the function.
= 0.8 = 0.6 4
=
3 4
sin q =
5 correct
3
Example 06: If cos q = , then find the values of tan , cosec
5 4
sin = incorrect
Solution: R 5
3
Given, cos q =
5
Let PQR be the right triangle such that ∠QPR = θ
Assume that PQ = 3 and PR = 5

P Q
Then, QR = PR - PQ = 2 2
25 - 9 = 4
opposite side QR 4
So, tan q = = =
adjacent side PQ 3
Brain Power On
hypotenuse PR 5 .............................................
and cosec q = = =
oppositeside QR 4 Trigonometry riddles

3 1. What did the triangle say to the


1
Example 07: If sin A = , prove that tan A + = 2 ,if A is an acute angle. circle?
5 cos A
Solution: 2. What do you call a teapot of
Draw right-angled triangle ABC such that ∠C is a right angle. Then, boiling water on top of the Mount
B
BC 3 Everest?
sin A = =
AB 5 3. Why did the identity sin(2r) =
Let BC = 3 and AB = 5. By Pythagoras theorem, 2sin(r) get turned down for a loan?
5 3
AC2 = AB2 - BC2 = 52 - 32 = 16 ⇒ AC = 16 = 4 4. Why do you rarely find
BC 3 AC 4 mathematicians spending time at the
tan A = = and cos A = = beach?
AC 4 AB 5
A 4 C
1 3 5 5. What does trigonometry have in
\ tan A + = + = 2. common with a beach?
cos A 4 4
6. What do you call an insect that's
Pythagorean Identities not feeling well?
The right triangle in figure. The Pythagorean theorem states, 7. Can an English major learn
Math?
b2 + a2 = c2
c
Dividing each side by c2, we get b 8. What is the opposite of a stop
2 2 sign?
b2 a 2 æ bö æa ö
+ 2 = 1 or çç ÷÷÷ + çç ÷÷÷ = 1
c 2
c çè c ø çè c ø a 9. What is the square root of ab?
10.Why didn't Sin and Tan go to the
In terms of trigonometric functions of the angle θ, this equation states that
party?
sin2 θ + cos2 θ = 1
80 CLASS - IX
ALGEBRA
BASICS OF-IV
TRIGONOMETRY
POLYNOMIALS

Another identity can be obtained from equation by dividing each side by cos2θ.
Brain Power On
............................................. sin 2 q 1
2
+1 =
Geometry in 3 dimensions
cos q cos2 q
Now use formulas to get
Find the length of d , the diagonal
tan2 θ + 1 = sec2 θ
of a cube with the following given
sides. Similarly, by dividing each side of equation by sin2 θ, we get 1 + cos2 θ = csc2 θ,
a. 5 units which we write as
cot2 θ + 1 = csc2 θ
b. 7 units
c. sec x = 2 units Collectively, the identities in equations are referred to as the Pythagorean
Identities.
Example 08: Prove: (1 - cos² θ) csc² θ = 1
Solution:
Let A = (1 - cos² θ) csc² θ and B = 1
Since , sin² θ + cos² θ = 1, we have sin² θ = 1 - cos² θ
d
A = sin² θ x csc² θ
We know that csc² θ = 1/sin² θ
A = sin² θ x 1/sin² θ
A = 1
A = B
Hence Proved
Example 09: Prove : tan⁴ θ + tan² θ = sec⁴ θ - sec² θ
Solution :
Let A = tan⁴ θ + tan² θ and B = sec⁴ θ - sec² θ
A = tan² θ (tan² θ + 1)
A = (sec² θ - 1) (tan² θ + 1) [since, tan² θ = sec² θ – 1]
A = (sec² θ - 1) sec² θ
A = sec⁴ θ - sec² θ
A = B
Hence Proved

ìï (sec q – 1) üï
Example 10: Prove : ïí ïý = cosec θ - cot θ.
ïï(sec q + 1)ïï
î þ
ìï (sec q – 1) üï
Solution : Let A =
ïí ïý and B = cosec θ - cot θ
ïï(sec q + 1)ïï
î þ
ìï (sec q – 1) üï
A = ïí ïý
ïï(sec q + 1)ïï
î þ
é {(sec q - 1)(sec q - 1)} ù
A = ê ú
ê {(sec q + 1)(sec q - 1)} ú
êë úû
MATH 81
(Multiplying numerator and denominator by (sec θ - l) under radical sign)
ïìï(sec q – 1)2 ïüï Brain Power On
A = í ý .............................................
ïï (sec2 q - 1) ïï Consider the situation below.
ïî ïþ
A boy who is on the 2nd floor of
ìï(sec q - 1) üï
= ïí ïý their house watches his dog lying on
ïïî tan q ïïþ the ground. The angle between his
eye level and his line of sight is 32°.
Since, sec² θ = 1 + tan² θ θ sec² θ - 1 = tan² θ
a. Which angle is identified in the
= (sec θ – 1)/tan θ
problem: angle of elevation or angle
= (sec θ/tan θ) – (1/tan θ) of depression? Justify your answer.
= {(1/cos θ)/(sin θ/cos θ)} - cot θ
b. If the boy is 3 meters above the
= {(1/cos θ) × (cos θ/sin θ)} - cot θ ground, approximately how far is the
= (1/sin θ) - cot θ dog from the house?
A = cosec θ - cot θ c. If the dog is 7 meters from the
A = B Proved house, approximately how high is
the boy above the ground?
Angle of elevation and depression
When you look up at something, such as an aeroplane, the angle between your
line of sight and the horizontal is called the angle of elevation.
Similarly, if you look down at something, then the angle between your line of
sight and the horizontal is called the angle of depression.

Angle of depression

Angle of elevation Cliffs

Example 11: The angle of elevation of the top of the building at a distance of
50m from its foot on a horizontal plane is found to be 60 degree. Find the height
of the building.
Solution: First let us draw the rough diagram for the above question.
A
Now we need to find the length of the side AB.
tan θ = Opposite side/Adjacent side
tan 60° = AB/BC
Substitute, the value of tan 60° from the trigonometric ratio table,
1.732 = AB/50
AB = 50 (1.732) = 86.6 m 60°
B 50 m C

82 CLASS - IX
ALGEBRA
BASICS OF-IV
TRIGONOMETRY
POLYNOMIALS

Table of Trigonometric Ratios

Angle Sine Cosine Tangent Angle Sine Cosine Tangent

1 .0175 .9998 .0175 46 .7193 .6947 1.0355


2 .0349 .9994 .0349 47 .7314 .6820 1.0724
3 .0523 .9986 .0524 48 .7431 .6691 1.1106
4 .0698 .9976 .0699 49 .7547 .6561 1.1504
5 .0872 .9962 .0875 50 .7660 .6428 1.1918
6 .1045 .9945 .1051 51 .7771 .6293 1.2349
7 .1219 .9925 .1228 52 .7880 .6157 1.2799
8 .1392 .9903 .1405 53 .7986 .6018 1.3270
9 .1564 .9877 .1584 54 .8090 .5878 1.3764
10 .1736 .9848 .1763 55 .8192 .5736 1.4281
11 .1908 .9816 .1944 56 .8290 .5592 1.4826
12 .2079 .9781 .2126 57 .8387 .5446 1.5399
13 .2250 .9744 .2309 58 .8480 .5299 1.6003
14 .2419 .9703 .2493 59 .8572 .5150 1.6643
15 .2588 .9659 .2679 60 .8660 .5000 1.7321
16 .2756 .9613 .2867 61 .8746 .4848 1.8040
17 .2924 .9563 .3057 62 .8829 .4695 1.8807
18 .3090 .9511 .3249 63 .8910 .4540 1.9626
19 .3256 .9455 .3443 64 .8988 .4384 2.0503
20 .3420 .9397 .3640 65 .9063 .4226 2.1445
21 .3584 .9336 .3839 66 .9135 .4067 2.2460
22 .3746 .9272 .4040 67 .9205 .3907 2.3559
23 .3907 .9205 .4245 68 .9272 .3746 2.4751
24 .4067 .9135 .4452 69 .9336 .3584 2.6051
25 .4226 .9063 .4663 70 .9397 .3420 2.7475
26 .4384 .8988 .4877 71 .9455 .3256 2.9042
27 .4540 .8910 .5095 72 .9511 .3090 3.0777
28 .4695 .8829 .5317 73 .9563 .2924 3.2709
29 .4848 .8746 .5543 74 .9613 .2756 3.4874
30 .5000 .8660 .5774 75 .9659 .2588 3.7321
31 .5150 .8572 .6009 76 .9703 .2419 4.0108
32 .5299 .8480 .6249 77 .9744 .2250 4.3315
33 .5446 .8387 .6494 78 .9781 .2079 4.7046
34 .5592 .8290 .6745 79 .9816 .1908 5.1446
35 .5736 .8192 .7002 80 .9848 .1736 5.6713
36 .5878 .8090 .7265 81 .9877 .1564 6.3138
37 .6018 .7986 .7536 82 .9903 .1392 7.1154
38 .6157 .7880 .7813 83 .9925 .1219 8.1443
39 .6293 .7771 .8098 84 .9945 .1045 9.5144
40 .6428 .7660 .8391 85 .9962 .0872 11.4301
41 .6561 .7547 .8693 86 .9976 .0698 14.3007
42 .6691 .7431 .9004 87 .9986 .0523 19.0811
43 .6820 .7314 .9325 88 .9994 .0349 28.6363
44 .6947 .7193 .9657 89 .9998 .0175 57.2900
45 .7071 .7071 1.0000 90 1 0 undefined

MATH 83
Example 12: A nursery plants a new tree and attaches a guy wire to help support
the tree while its roots take hold. An eight foot wire is attached to the tree and to
a stake in the ground. From the stake in the ground the angle of elevation of the
connection with the tree is 42º. Find to the nearest tenth of a foot, the height of
the connection point on the tree.
Solution:
The "angle of elevation" is from the ground up. It is assumed that the tree
is vertical, making it perpendicular with the ground. This problem deals with 8’
X
"opposite" and "hypotenuse" making it a sine problem.
Substitute, the value of sin 42° from the trigonometric ratio table,
x x
sin 42° = ; 0.669 = ; x » 5.4 ’
8 8
Example 13: A radio station tower was built in two sections. From a point
87 feet from the base of the tower, the angle of elevation of the top of the first
section is 25°, and the angle of elevation of the top of the second section is 40°.
To the nearest foot, what is the height of the top section of the tower ?
Solution:
Think of this problem as working with two separate triangles: x
(1) The larger triangle with the 40° angle and a vertical side that represents the
b
entire height, b, of the tower.
a
(2) The smaller triangle with the 25° angle and a vertical side, a, that represents 40˚
the height of the first (bottom) section of the tower. 25˚
87 feet
Solve for the vertical heights (b and a) in the two separate triangles.
The needed height, x, of the second (top) section of the tower will be the
difference between the entire height, b, and the height of the first (bottom)
section, a.You will need to subtract.
In both triangles, the solution deals with "opposite" and "adjacent" making it
a tangent problem.
b b
Larger triangle with height b: tan 40° = ; 0.83 = ; b » 73.0 ’
87 87
a a
Smaller triangle with height a: tan 25° = ; 0.46 = ; a » 40.5 ’
87 87
Difference (b - a): 73.0 - 40.5= 32.4 feet

Example 14: From the top of the tower 30 m height a man is observing the
base of a tree at an angle of depression measuring 30 degree. Find the distance
between the tree and the tower.
Solution: Now we need to find the distance between foot of the tower and the A
foot of the tree (BC).
tan θ = Opposite side/Adjacent side
tan 30° = AB/BC 30 m

0.577 = 30/BC
BC = 51.96 m
B C

84 CLASS - IX
ALGEBRA
BASICS OF-IV
TRIGONOMETRY
POLYNOMIALS

Brain Power On
....................................................................................................................................................
Pythagorean mosaic
Find the missing side in each triangle and shade according to the answers given in the square.

Brain Power On
....................................................................................................................................................
1. Create a short story based on the Pythagorean mosaic
scene pictured below.
2. Incorporate the concepts you have
learned in the story by creating at
least 2 problems. Present the solutions
to the problem.
3. Present your work to the class.

MATH 85
Example 15: The angles of depression of the top and the bottom of a 12 m
high building from the top of the tower are 45 degree and 60 degree respectively.
Calculate the height of the tower
Solution: First let us draw the rough diagram for the above question.

x C
B
12 m

D E
Here AD represents height of the tower, DC represents the distance between
tower and the building.
In the right triangle ADE the side which is opposite to angle is known as
opposite side(AD),the side which is opposite to 90 degree is called hypotenuse
side(AE) and remaining side is called adjacent side (DE).
Now we need to find the height of tower (AD)
tan θ = Opposite side/Adjacent side
Let AB = h
In ∆ ABC
tan 45° = AB/BC
1 = x/BC
BC = x ...................................................... (1)
In ∆ ADE Brain Power On
................................................
tan 60° = AD/DE
How long is this side?
1.732= (AB+BD)/DE 40
2.24 cm
6.9 cm
1.732 = (12 + x)/DE 220
3.2 cm

2.73 cm
cm 1.1
DE = (12 + x)/1.732 .............................................. (2)
Length of BC and DE are equal (1) = (2) 1.7 cm 460

x = (12 + x)/1.732
1.9

1 cm
190 300
cm

m
1.732 x = 12 + x 4.8c

1.732 x - x = 12 cm
3.2 1 c 3.2 cm
1 cm m
x(1.732 - 1) = 12 2 cm 430 cm
3.8
110 2.9 cm
On solving for x 3.6 c
m
2.5
x = 16.392 cm
280
Height of the tower = 12 + 16.392 = 28.392 m 8 cm

86 CLASS - IX
ALGEBRA
BASICS OF-IV
TRIGONOMETRY
POLYNOMIALS

Classroom Activity
Make your own Clinometer
Objective: To make a clinometer and use it to measure the height of an object.
Materials required: Stiff card, small pipe or drinking straw, thread, a weight.
Procedure:
(A) To make a clinometer:
1. Attach a protractor on a cardboard and fix a viewing tube (straw or pipe) along the diameter.
2. Punch a hole (o) at the center of the semicircle.
3. Suspend a weight {w} from a small nail fixed to the center.
4. E nsure that the weight at the end of the string hangs below the protractor.
5. Mark degrees (in sexagecimal scale with 00 at the lowest and 10 to 900 proceeding both clockwise and
counterclockwise).
Tape
Straw
Tape
80

80
Tape
60

(on back)

60
40

20 40
0 20

Protractor

String

Weight

(B) To determine the height of an object:


6. First measure the distance of the object from you. Let the distance be d.
7. L ook through the straw or pipe at the top of the object. Make sure you can clearly see the top of the object.
8. Hold the clinometer steady and let your partner record the angle the string makes on the scale of the clinometer.
Let this angle be θ.
9. Determine the height of the object.
10. Write your observation.You may change the distance of the object (by either moving the object or by changing
their position) and note how the angle of elevation varies.

Trigonometric ratios of standard angles


Trigonometric ratios of 0° and 90°
Consider the figure given below which shows a circle of radius 1 unit centred
at the origin. Let P be a point on the circle in the first quadrant with coordinates
(x, y). Y

B(0, 1)
P(x, y)
1
y
θ
O x Q A(1, 0) X

MATH 87
We drop a perpendicular PQ from P to the x-axis in order to form the right
triangle OPQ.
Let < POQ = θ, then
sin θ = PQ / OP = y/1 = y (y coordinate of P)
cos θ = OQ / OP = x/1 = x (x coordinate of P) Thales and the Great
tan θ = PQ / OQ = y/x Pyramid
If OP coincides with OA, then angle θ = 0°.Since, the coordinates of A are (1, Thales was arguably the
0), we have founder of the great ancient Greek
sin 0° = 0 (y coordinate of A) cosec 0° is not defined tradition of mathematics and a
cos 0° = 1 (x coordinate of A) sec 0° = 1 teacher of Pythagoras. Thales asked
sin 0° 0 the Egyptian priests about the height
tan 0° = = = 0 cot 0° is not defined
cos 0° 1 of the Great Pyramid of Cheops, and
If OP coincides with OB, then angle θ = 90°.Since, the coordinates of B are they did not tell him, so he set about
(0, 1), we have
measuring it himself.
sin 90° = 1 (y coordinate of B) cosec 90° = 1
cos 90° = 0 (x coordinate of B) sec 90° is not defined Thales hit upon an idea!
sin 90° 1 What if he measures the length of
tan 90° = = is not defined cot 90° = 0
cos 90° 0 the shadow of the pyramid? Then
Trigonometric ratio of 45° measure the length of a stick and its
If an acute angle of a right triangle is 45°, then the other acute angle is shadow to create a proportion.
also 45°. Thus the triangle is isosceles. Let us consider the triangle ABC with
∠B = 90°, ∠A = ∠C = 45°
Then AB = BC. Let AB = BC = a. C

By Pythagorean Theorem, Length


of base
AC² = AB² + BC²
a Thales Stick
AC² = a² + a²
AC² = 2a² Stick
Shadow
AC = 2 x a A a B

Hence, we can find the trigonometric ratios of angle 45° from the right
triangle ABC.
1/2 Length Shadow
BC a 1 1 of base
sin 45° = = = cos ec 45° = = 2
AC a 2 2 sin 45° Height of pyramid Height of Stick
= Shadow of stick
1/2 length of base + shadow of pyramid
AB a 1 1
cos 45° = = = s ec 45° = = 2
AC a 2 2 cos 45°
BC a 1
tan 45° = = =1 cot 45° = =1
AB a tan 45°

Trigonometric ratios of 30° and 60°


Let ABC be an equilateral triangle whose sides have length a (refer to the
figure). Draw AD perpendicular to BC, then D bisects the side BC.
So, BD = DC = a/2 and ∠BAD = ∠DAC = 30°.

88 CLASS - IX
ALGEBRA
BASICS OF-IV
TRIGONOMETRY
POLYNOMIALS

A
Brain Power On
............................................
Move any four match sticks to get
three equilateral triangles
a a
2

1 10 3 60°
11 60°
12 9 A a D a B
2 2
Now, in right triangle ADB, ∠BAD = 30° and BD = a/2.
6 7 8 4 In right triangle ADB, by Pythagorean Theorem,
AB² = AD² + BD²
5 a² = AD² + (a/2)²
AD² = a² - (a²/4)
AD² = 3a²/4
AD = (3a / 4)
AD = 3 x (a/2)
Hence, we can find the trigonometric ratios of angle 30° from the right triangle
ADB.
a
BD 2 1 1
sin 30° = = = cos ec 30° = =2
AB a 2 sin 30°
3
a 1 2
AD 2 3 s ec 30° = =
cos 30° = = =
AB a 2 cos 30° 3
a
BD 1 1
tan 30° = = 2 = cot 30° = = 3
AD 3 3 tan 30°
a
2
In right triangle ADB, ∠ABD = 60°. So, we can determine the trigonometric
ratios of angle 60°.
3
a 1 2
AD 2 3 cos ec 60° = =
sin 60° = = =
AB a 2 sin 60° 3
a
BD 2 1 1
cos 60° = = = s ec 60° = =2
AB a 2 cos 60°

3
a 1 1
AD cot 60° = =
tan 60° = = 2 = 3 tan 60° 3
BD a

2
MATH 89
Values of trigonometric ratios for specific angles
How do I remember it all?
angleθ ............................................
0° 30° 45° 60° 90°
ratio Write numbers 1 to 4 for sin
1 1 3 angles and 4 to 1 for cos angles.
sin θ 0 1
2 2 2 Take square root and divide every
values by 2. The other standard
3 1 1
cos θ 1 0 angles can be computed easily from
2 2 2 sin and cos values.
1 00 300 450 600 900
tan θ 0 1 3 not defined sin 0 1 2 3 4
3
cos 4 3 2 1 0
2 2
cosec θ not defined 2 2 1
3

2
sec θ 1 2 2 not defined
3

1
cot θ not defined 3 1 0
3
Brain Power On
Example 16: If β = 30°, prove that 3 sin β - 4 sin3 β = sin 3β. ............................................
Solution: Which is top view?

L.H.S = 3 sin β - 4 sin3 β


= 3 sin 30° – 4. sin3 30°
= 3 ∙ (1/2) - 4 x (1/2)3
= 3/2 – (4 x 1/8)
= 1
R.H.S. = sin 3A
= sin 3 x 30°
= sin 90°
=1

Therefore, L.H.S. = R.H.S. (Proved)
Example 17 : Find the value of 4/3 tan2 60° + 3 cos2 30° - 2 sec2 30° - 3/4
cot2 60°
Solution:
The given expression A B
2 2 2
4 æ 3ö æ 2 3 ö÷ 3 æ 3 ö÷
( 3) + 3 × ççç ÷÷÷ - 2 × ççç ÷÷ - × çç ÷÷
2
×
3 ÷
çè 2 ø çè 3 ÷ø 4 ççè 3 ÷ø
4 3 12 3 3
= ×3 + 3× - 2× - ×
3 4 9 4 9
9 8 1 10 1
=4+ - - = =3
4 3 4 3 3 C D
90 CLASS - IX
ALGEBRA
BASICS OF-IV
TRIGONOMETRY
POLYNOMIALS

Brain Power On
....................................................................................................................................................
TRIGONOMETRIC IDENTITIES
Connect the equivalents. Then find the isosceles, right triangle formed.
Note: Discuss with your teacher to solve the trigonometric identities that are not discussed in this chapter.

tan2 x + 1 cos2 x - sin2 x

tan2 x
1 + cot x 2

-sin x
1 - sin x2

4cscx
sin2 x
cos2 x cos(2x)
4 2sinx cosx cos(-x)
3 sec x sin x
sin(x-y)
sin(x+y) sec2x
5sin x csc x cos(x+y) cos2 x

csc2 x cos x cos y - sin x sin y

tan(-x)

-tan x
sin x cos y - cos x sin y
7tan x cot x 4 cot x
sin(-x) sin x cos y + cos x sin y
4cos x
sin x 3 7
cos x 5
cos x sin(2x)

MATH 91
Example 18: The escalator at the Metro Rail Station rises 76 feet at a 30°
angle as shown in the diagram below. Find the distance d a person travels on the
escalator stairs.

d
76 ft

Solution: The trigonometric ratio that involves opposite side and hypotenuse is
sine.
sin 30° = opposite / hypotenuse
Substitute, sin 30° = 76 / d
d × sin 30° = 76
Substitute 0.5 for sin 30°.
d × 0.5 = 76
d = 152
Hence, a person travels 152 feet on the escalator stairs.

Trigonometric ratios of complementary angles :


Two acute angles are complementary to each other if their sum is
equal to 90°. In a right triangle the sum of the two acute angles is equal to 90°.
So, the two acute angles of a right triangle are always complementary to each
other.
Let ABC be a right triangle, right angled at B.
Brain Power On
C ............................................
Find the missing angles

S
U
V
T
710

A B
If <ACB = θ, then <BAC = 90° - θ and hence the angles <BAC and <ACB
W

are complementary
N
M
L

For the angle θ, we have,


J H
K

R
P

üï
I

AB AC
sin q = cos ecq = ïï
AC AB ïï
ïï
D

AC
A
F

BC sec q = ïý (1)
230

cos q =
C

E
B

AC BC ïï
ïï
AB BC ïï
680
390

tan q = cot q =
BC AB ïïþ

92 CLASS - IX
ALGEBRA
BASICS OF-IV
TRIGONOMETRY
POLYNOMIALS

Similarly, for the angle (90° - θ), we have


BC AC ïüï
sin (90° - q) = cos ec (90° - q) = ïï
AC BC ïï
AB AC ïï
cos (90° - q) = sec (90° - q) = ý ( 2)
AC AB ï
ï
ï
ï
BC AB ï
tan (90° - q) = cot (90° - q) = ï
AB BC ïþï
Comparing the equations in (1) and (2) we get,
AB AC
sin q = = cos (90° - q) cos ec q = = sec (90° - q)
AC AB
BC AC
cos q = = sin (90° - q) sec q = = cos ec (90° - q)
AC BC
AB BC
tan q = = cot (90° - q) cot q = = tan (900° - q)
BC AB

Example 19: Evaluate: cos 56° / sin 34°


Solution: The angles 56° and 34° are complementary.
So, using trigonometric ratios of complementary angles, we have
cos 56° = cos (90° - 56°) = sin 34°
cos 56° / sin 34° = sin 34° / sin 34° = 1
Hence the value of cos 56° / sin 34° is 1.

Example 20: Evaluate : (cos 65° sin 18° cos 58°) / (cos 72° sin 25° sin 32°)
Solution : Using trigonometric ratios of complementary-angles, we have
cos 65° = cos (90° - 25°) = sin 25°
sin 18° = sin (90° - 72°) = cos 72°
Brain Power On
............................................ cos 58° = cos (90° - 32°) = sin 32°
1. Given that sin (x + 10)º = cos (cos 65° sin 18° cos 58°) / (cos 72° sin 25° sin 32°) is
(3x + 20)º, find the number of = (sin 25° cos 72° sin 32°) / (cos 72° sin 25° sin 32°)
degrees in the acute angles of the
corresponding right triangle. = 1

2. In right ΔABC, m∠C = 90º, cos A Example 21: If sin A = cos 33°, find A
= 1/5. What is sin B ? Solution : Using trigonometric ratios of complementary-angles, we have
3. In right ΔABC, m∠C = 90º. sin A = cos (90° - A)
Simplify the following expression
Therefore,
sin A - cos B
2 sin A = cos 33° → cos (90° - A) = cos 33°

4. In right ΔABC, m∠C = 90º and 90° - A = 33°


m∠A does not equal the m∠B. If sin 90° - 33° = A
A = m and cos A = k, find cos B +
Hence, A is 57°.
sin B.

MATH 93
94
Trigonometric ratios of standard angles Trigonometric ratios
angleθ of complementary angles Angle of elevation and depression
0° 30° 45° 60° 90°
ratio sin q = cos (90° - q )

CLASS - IX
1 1 3 cos q = sin (90° - q ) Angle of depression
sin θ 0 1
2 2 2 tan q = cot (90° - q )
Angle of elevation Cliffs
3 1 1
cos θ 1 0 cos ec q = sec (90° - q)
2 2 2
1 sec q = cos ec (90° - q)
tan θ 0 1 3
3 cot q = tan (900° - q)
2
cosec θ 2 2 1
3
2 length of side opposite to the angle
sec θ 1 2 2 M I N D M A P sine of an angle =
3 length of the hypotenuse
1 TRIGONOMETRY
cot θ 3 1 0 length of side adjacent to the angle
3 cosine of an angle =
measurement of a triangle. length of the hypotenuse
Triangles
length of side opposite to the angle
Type of Triangles tangent of an angle =
Trigonometric Ratios length of side adjacent to the angle
Classified based on sides and angles.

q = 90°
Fundamental Identities Pythagorean Identities
Right-angled triangle sin2 θ + cos2 θ = 1
𝜃 tan2 θ + 1 = sec2 θ
hypotenuse
adjacent cot2 θ + 1 = csc2 θ
Reciprocal Identities Quotient Identities
opposite 1 sin q
csc q = tan q =
sin q cos q
Pythagoras' Theorem 1
If the length of the hypotenuse sec q = cos q
cos q cot q =
of a right-angled triangle is c sin q
and the lengths of the other 1
cot q =
sides are a and b tan q
c2 = a2 + b2
BASICS OF-IV
ALGEBRA TRIGONOMETRY
POLYNOMIALS

Do at Class!
1. Do the following lengths form a right triangle? V U B
e
6 8 K y
5 12 A x C
f
(i) 9 (ii) 13 (iii) S T (iv)
D
L K S R
q a
b
8 10 P
r c
(v) H I J (vi) P T Q
3 4

4. In figure, ∠OMP, ∠ORQ, ∠OQM are right angles.


(iii) 6 (iv) 5 Write the values of the following t-ratios:
P
v) a = 6.4, b = 12, c = 12.2
Q
vi) a = 2.1, b = 7.2, c = 7.5

2. 
In each figure, use the letters to indicate the M
O R
perpendicular, hypotenuse, and the base (in that
order) with respect to the angled marked x. (i) sin RQM (ii) sin QMP
(iii) sin OQR (iv) cos QMP
(v) tan RQM (vi) cot MP
c b (vii) sec ROQ
r
q
5. From figure, find the value of tan θ, tan2θ, tan3θ,
x x
(i) a (ii) p sin3θ and cos3θ. A
f

p g 5
m 4
e
x
x
(iii) n (iv) θ
h B 3 C
x
υ 6. 
Use trigonometric ratio tables to solve the
u
d following problems.
i
x
w
a. • sin 36° • cos 92°
(v) (vi)
• tan 87° • sin 49°

3. 
Write down the tangent ratio of each angled b.Find the missing side. Round to the nearest tenth.
marked with a small letter. (In each figure) 1) 2) 3)

C L 8
y y x
7

270 390 460

M x
10 x 10
A x B N
(i) (ii) T
MATH 95
7. Complete the following:
B
C B 25 A
5 9 12
3 15
20
B 15 A C
A 4 C

tan A =
tan B =
cot A =
cot B =
sin A =
sin B =
cosec A =
cosec B =
cos A =
cos B =
sec A =
sec B =
8. From figure, find the value of sin , cos θ, tan θ, 11. In a ΔABC, right angled at B, AC = 15 cm and ,
sin2θ, cos2θ and tan2θ. calculate the measure of AB and BC.
C
C A
θ
13
12 15 15

A 5 B
B C
9. In figure, ΔABC is right angled at B, ΔBSC is right- 18
A B
angled at S, and ΔBRS is right-angled at R, AB =
18 cm, BC = 7.5 cm, RS = 5 cm, ∠BSR = x° and 12. Answer the following :
∠SAB = y°. Find (i) tan x°, (ii) sin y°, (iii) cos y°.
(Leave your answers as fractions).
C 600 c
S b
o 7.5 cm
x 300
5 cm
a
yo
A 18 cm
R B (i) If c = 2, find the exact values of a and b.

10. 
ABC is a right-angled A (ii) If b = 4 find the exact values of a and c.
triangle, right-angled at B.
(iii) If , find the exact values of b and c.
Given that ∠ACB = θ, side
AB = 2 units and side BC 2 (iv) If c = 5, find the exact values of a and b.
= 1 unit, find the value of
sin2θ + tan2θ. (v) What is the exact value of tan 30° ?
q
(vi) What is the exact value of tan 60° ?
C 1 B

96 CLASS - IX
BASICS OF-IV
ALGEBRA TRIGONOMETRY
POLYNOMIALS

II. Choose the correct answer: 6. A ship’s guidance system measures that the ship is
380 m from the top of a lighthouse. The top of the
1. Hari drove about 150 km east from station A. She
lighthouse is 88 m above sea level. How far is the
drove about another 75 km North to station B.
ship from the lighthouse to the nearest tenth of a
What is the approximate air distance from station
metre?
A to B?
(a) 182.9 m (b) 369.7 m
(a) 160 km (b) 175 km
(c) 234.0 m (d) 390.1 m
(c) 168 km (d) 225 km
7. A 1.5m tall boy stands at a distance of 2m from
2. What is the measure of the missing length?
lamp post and casts a shadow of 4.5m on the
? ground. Find the height of the lamp post.
19 m
(a) 3 m (b) 2.5 m
(c) 5 m (d) none of these
52 m
8. If the angle of elevation of a cloud from a point
(a) 55 m (b) 57 m 200m above a lake is 30° and the angle of depression
(c) 63 m (d) 71 m of its reflection in the lake is 60° , then the height
3. What is the area of square C? of the cloud above the lake is .................
(a) 200 m (b) 500 m
35 cm (c) 300 m (d) 400 m
C
A
9. 
In the below figure what are the angles of
depression from the observing positions D and E
of the object A?
B 28 cm
D E
(a) 90 cm 2
(b) 1960 cm 2

(c) 2009 cm2 (d) 3969 cm2


600

4. What is the measure of the missing length?

20 m 450
12 m ? C B A
(a) 30°, 45° (b) 60°, 45°
25 m (c) 45°, 60° (d) none of these

(a) 13 m (b) 14 m 10. Value of θ, for sin 2θ = 1, where 0° < θ < 90° is
(c) 15 m (d) 16 m ......................
(a) 30° (b) 60°
5. 
What is the measure of the hypotenuse to the
(c) 45° (d) 135°
nearest tenth of a metre?
11. Value of sec2 26° – cot2 64° is ....................

A= ? (a) 1 (b) –1
256 m2 (c) 0 (d) 2

12. 
Product tan 1°.tan 2°.tan 3° ………. tan 89° is
32 m .......................
(a) 35.8 m (b) 45.3 m (a) 1 (b) –1
(c) 64.0 m (d) 90.5 m (c) 0 (d) 90

MATH 97
13. If A + B = 90°, cot B = 3/4 then tan A is equal to 3. An airplane ascends with a 30° angle of inclination.
..................... If the airplane is flying at a rate of 5 km/min, what
is the altitude after 5 min?
(a) 3/4 (b) 4/3
(c) 1/4 (d) 1/3

14. The value of 2 sin230° – 3 cos2 45° + tan2 60° + 3


sin2 90° is ..........................
ory
ject
Tra
(a) 1 (b) 5 Altitude

(c) 0 (d) none of these


Angle of inclination

15. If cos θ = 1/2 , sin φ = 1/2 then the value of θ +


φ is .......................
(a) 30° (b) 60° 4. The plane shown is ascending at a 10° angle. If
(c) 90° (d) 120° the plane is 40 ft high, how far is the end of the
runway?
III. Application of trigonometry
Note: Use t-ratio table wherever necessary

1. A tree casts a shadow 32 ft long. A hiker estimates


40 ft
the angle between the Sun’s rays and the ground 10º
to be 72° (see figure). Estimate the height of the
tree to the nearest whole number.
5. The angle of elevation of a jet fighter from a point
A on the ground is 60°. After a flight of 15 seconds,
the angle of elevation changes to 30°. If the jet is
flying at a speed of 720 km/hr, find the constant
height at which the jet is flying. (Use ).
A 15 sec E
(3000 m)

720 F
32 ft

2. The Washington Monument is 555 ft tall. If the


Sun’s rays and the monument form an angle of 27° B C D
(see figure), how long is the shadow to the nearest
whole number? 6. Given that the property line is halfway between the
two houses on the plan below, what is the distance
between the two houses to the nearest tenth of a
metre? How far is the corner of house B from the
survey post to the nearest tenth of a metre?
270 Property line

555ft House A House B


19.4 m

720
Survey post
Road

98 CLASS - IX
BASICS OF-IV
ALGEBRA TRIGONOMETRY
POLYNOMIALS

7. An eagle is sitting on a tree 14 m above the ground. IV. Verify the following:
The eagle sights a mouse on the ground at an angle
of 58° with the tree. How far is the eagle from the 2 tan 30° 3
1. sin 60° = 2
=
mouse? 1 + tan 30° 2

2.
1 - tan 2 30° 1
cos 60° = =
580 1 + tan 2 30° 2
3. cos 60° = cos2 30° – sin2 30°
14 m 4. cos 60° = 1 – 2 sin2 30° = 2 cos2 30° – 1

5. cos2 30° + sin2 30° + tan2 45° = 2

6. 4(sin4 30° + cos4 60°) – 3(cos2 45° – sin2 90°) = 2


8. Joan stands 20 m from the base of a large tree. Her
line of sight to the top of the tree makes an angle 7. If A = B = 60°, verify that cos (A – B) = cos A cos
of 54° with the horizontal. Joan is 1.6 m tall. How B + sin A sin B.
tall is the tree?
8. If A = 30° and B = 60°, verify that sin (A + B) =
sin A cos B + cos A sin B.
cos q sin q
9. + =2
sin(90° - q ) cos(90° - q )
10. sin (60° – θ) = cos (30° + θ)
540
11. (1 - cos θ)(1 + cos θ)(1 + cot² θ) = 1

12. cot θ + tan θ = sec θ csc θ

20 m 13. √{(sec θ – 1)/(sec θ + 1)} = cosec θ - cot θ.

9. A roof rises 1.2 m for every 2 m run. Find the 14. (1 - sin A)/(1 + sin A) = (sec A - tan A)²
pitch of the roof in degrees (angle x). 15. (tan θ + sec θ - 1)/(tan θ - sec θ + 1) = (1 + sin
θ)/cos θ

16. tan⁴ θ + tan² θ = sec⁴ θ - sec² θ


1.2 m
x 17. cos θ/(1 - tan θ) + sin θ/(1 - cot θ) = sin θ + cos θ
2m
18. sec θ √(1 - sin ² θ) = 1
10. Grace wants to know how far it is to the other
side of the river. She makes the measurements as V. Solve the following:
shown. What is the distance from A to B across the 1. Prove : tan 35° tan 60° tan 55° tan 30° = 1
river?
2. Evaluate : tan 25° / cot 65°

3. If sec 5θ = csc (θ - 36°), where 5θ is an acute angle,


find the value of θ.

4. 
Using trigonometrical ratios of complementary
A angles prove that tan 1° tan 2° tan 3° ......... tan
89° = 1
350 5. 
Without using trigonometric tables, evaluate sin
C 20 m B 35° sin 55° - cos 35° cos 55°

MATH 99
Do at Home!
Note: Use t-ratio table wherever necessary 6. 
Find the sine, cosine and tangent of the given
triangle.
1. A right triangle has side lengths 3 and 4, as shown
B
in the figure. Find the length c of the hypotenuse.
c
5
c
3 C 12 A

7. The diagram shows a quadrilateral ABCD.


4 B C
400
2. The length of one side of a right triangle is 24, and
the length of the hypotenuse is 26, as shown in the 16 cm
figure. Find the length a of the other side.

26
a A 12 cm D

AB = 16 cm
24 AD = 12 cm
Directions : For problems 3 – 4 refer to the right Angle BCD = 40°
triangle below. Angle ADB = angle CBD = 90°
A Calculate the length of CD.

c 8. ABCD is a triangle.
b B

a 7cm 12cm
3. If a = 4, b = 3, and c = 5, find
(i) cos A (ii) sin A (iii) tan A. 650
Express your answers as reduced fractions. A D C
4. If a = 12, b = 5, and c = 13, find ADC is a straight line with BD perpendicular to AC.
(i) cos A (ii) sin A (iii) tan A. AB = 7 cm,
Express your answers as reduced fractions. BC = 12 cm

5. The length of one side of a right triangle is 40 ft, Angle BAD = 65°
as shown in the figure. 9. A shed has a gable roof as drawn below. Calculate
(i) Find the length of side a. the length of sheets of roofing iron requreid in its
(ii) F
 ind the length of hypoteuse c. Round your construction.
answers to the nearest whole number. Let the length of the sheets be l.
l
c 1.6 m
a
200 4.8 m
40 ft
100 CLASS - IX
BASICS OF-IV
ALGEBRA TRIGONOMETRY
POLYNOMIALS

10. 
A guy (support) wire is 3. Given the figure on the right, which referes to the
attached 3.2 m up a pole angle of depression?
and at a point 2.1 m from L
object
the pole. The ground and

Pole
the pole are perpendicular

Gu
(at right angles). What is

y
eye K M

wi
object

re
the length of the guy wire?

N
Ground
(a) ∠MKN (b) ∠MKL
II. Match the following using figures: (c) ∠LKN (d) none of these
B P q Q
4. In the triangle PQR, what is the length of PQ?
c P
a p
r
13 cm
C b A R

Column A Column B Q 12 cm R

b (a) 1 cm (b) 5 cm
(1) tan A (a)
c (c) 9 cm (d) 12 cm

p 5. In the triangle DEF, what is m∠E to the nearest


(2) cos A (b) degree?
q E

q 17
(3) sin P (c)
r
r D 15 F
(4) cot P (d)
p
(a) 16° (b) 28°
a (c) 41° (d) 62°
(5) sec P (e)
b 6. The expression 2 (sin 30°) – tan 45° is equal to
....................
III. Choose the correct answer:
(a) 0 (b) 1
1. With respect to the given angle, what is the ratio
(c) 2 (d) –1
of the hypotenuse to the opposite side?
(a) sine (b) cosine 7. A kite held by 125 m of string makes an angle of
(c) tangent (d) cosecant elevation with the ground of 45°. About how high
is the kite above the ground?
2. Which of the following statements is correct?
(a) 62.8 m (b) 75.1 m
(c) 88.4 m (d) 113.6 m
x 8. From the top of a barn 7.62 m high, you see a
y cat on the ground. The angle of depression of the
600 cat is 40°. How many meters must the cat walk to
4 reach the barn?
(a) x = 8 (b) sin 30° = 1/x (a) 9.08 m (b) 9.80 m
(c) sin 60° = y/4 (d) cos 60° = 4/y (c) 9.81 m (d) 9.18 m
MATH 101
9. ABCD is a parallelogram. If AB is 8 cm long, BC is 13. (sec A + tan A) (1 – sin A) = ....................
5 cm and their included angle measures 100°, how
(a) sec A (b) sin A
long is diagonal AC?
(c) cosec A (d) cos A
(a) 12.95 m (b) 12.59 m
(c) 10.40 m (d) 10.14 m 1 + tan 2 A
14. = .....................
10. In right trinagle PQR, PQ = 12 cm and QR = 5 1 + cot 2 A
cm. What is cos R? (a) sec2A (b) –1
(a) 12/13 (b) 5/13 (c) cot2A (d) tan2A
(c) 5/12 (d) 12/5
15. If the angle of elevation of a tower from a distance
11. If sin (A + B) = 1 = cos (A ¬– B) then ................ of 100 m from its foot is 60°, then the height of the
(a) A = B = 90° (b) A = B = 0° tower is .....................
(c) A = B = 45° (d) A = 2B 200
(a) 100 3m (b) m
12. (1 + tan A + sec A) (1 + cot A – cosec A) = .......... 3
(a) 0 (b) 1 100
(c) 50 3m (d) m
(c) 2 (d) –1 3

IV. Where Have I Gone Wrong?


Directions:
1. Read carefully the problems given below.
2. Analyse the suggested solutions and find out what is wrong with them.
3. Write your solutions in the 2nd column of the table given below.
4. On the 3rd column of the table, write an explanation why your solution is correct.
1. An airplane is flying at constant altitude of 100 m above the ground. At that instant, it was recorded that the
angle of depression of the airport is 40°. Find the horizontal distance between the airplane and the airport.

Suggest Solution My Solution My Explanation


Let A represent the airplane and B the
airport. A
400
1000 m

C x B

opposite
tan 40° =
adjacent
x
tan 40° =
1000
x = 1000 (tan 40°)
= 1000 (0.8391)
x = 839.1
∴ The horizontal distance between the
airplane and the airport is 839.1 m.

102 CLASS - IX
BASICS OF-IV
ALGEBRA TRIGONOMETRY
POLYNOMIALS

2. Adrian noticed that one of the lampposts installed in their garden is about to collapse. As a precautionary
measure, he attached a 2-meter wire to the lamppost to support it. One end of the wire is attached one meter
from the base of the lamppost and the other end is attached to the base of a nearby tree. Determine the angle
the wire makes with the ground.

Suggest Solution My Solution My Explanation


M

2m
1m

G A
adjacent
sin G =
hypotenuse

GA = GM 2 - MA 2

GA = 22 - 12
GA = 3
3
sin G =
2
æ 3ö
G = sin-1 ççç ÷÷÷
èç 2 ÷ø

By Pythagorean Theorem,

G = sin–1 (0.8660)

∴ m ∠G = 60°

Space for Rough Work

MATH 103
Scope
Whether you want to explore space, time, matter or the
many other intriguing elements of the physical world, a physics
degree can do wonders for your career path.Physics graduates
go on to work within research roles, these are spread across
many different industries – including education, automotive and
aerospace industries, defense, the public sector, healthcare,
energy, materials, technology, computing and IT.
Chapter 1
Learning Checklist
Some basic aspects of motion

MOTION IN A Speed, velocity and acceleration.


Graphical representation of motion

STRAIGHT LINE Equations of uniformly accelerated motion


Uniform circular motion

Motion
Motion of a body refers to the change in its position with respect to time in
a given frame of reference. Surprisingly, everything in this world is constantly
moving and nothing is stationary. The apparent state of rest, as we shall learn, is a
notional experience confined to a particular system of reference. A building, for
example, is at rest in Earth's reference, but it is a moving body for other moving
systems like train, motor, airplane, moon, sun etc.
In our daily life, we see many objects moving. Some objects move in a straight
line. For example, a ball rolling on a horizontal surface, a stone falling from
a building and a runner on a 100 m race track. In all these examples, objects
change their positions with time along a straight line. This type of motion is
called rectilinear motion. In this chapter we shall confine ourselves to the study
of rectilinear motion.
In our discussions, we shall treat the objects in motion as point objects. This
approximation is valid so far as the size of the object is much smaller than the
distance it moves in a reasonable duration of time. In a good number of situations
in real-life, the size of objects can be neglected and they can be considered as
point-like objects without much error.

Frame of reference and observer


Motion has no meaning without a reference system. Frame of reference is
a mathematical construct to specify position or location of a point object in
space. Basically, frame of reference is a coordinate system. There are plenty of
coordinate systems in use, but the Cartesian coordinate system, comprising of
three mutually perpendicular axes, is most common. A point in three dimensional
space is defined by three values of coordinates i.e. x, y and z in the Cartesian
system as 3 shown in the figure below.

y
A(x,y,z)

x
o
z
A point in three dimensional space is defined by three values of coordinates The position of plane with respect to
the earth keeps changing with time.
106 CLASS - IX
MOTION IN A STRAIGHT LINE

If one or more coordinates of an object change with time, we say that the
Brain Power On object is in motion. Otherwise, the object is said to be at rest with respect to this
.......................................... frame of reference. The choice of a set of axes in a frame of reference depends
upon the situation. For example, for describing motion in one dimension, we
1. Each day you see the Sun rise in need only one axis. To describe motion in two/three dimensions, we need a set
the east, travel across the sky, and set of two/ three axes. Description of an event depends on the frame of reference
in the west. chosen for the description. For example, when you say that a car is moving on a
road, you are describing the car with respect to a frame of reference attached to
a) Explain this observation in terms
of your frame of reference. you or to the ground. But with respect to a frame of reference attached with a
person sitting in the car, the car is at rest.
b) Compare the observation to the To describe motion along a straight line, we can choose an axis, say X-axis, so
actual motions of the Sun and Earth. that it coincides with the path of the object. We then measure the position of the
object with reference to a conveniently chosen origin, say O, as shown in Figure.
Positions to the right of O are taken as positive and to the left of O, as
negative. Following this convention, the position coordinates of point P and Q
are +360 m and +240 m. Similarly, the position coordinate of point R is –120 m.
R O Q P
m
-160 -120 -80 -40 0 40 80 120 160 200 240 280 320 360 400
-x +x

Scalar and vector


Each of the physical quantities you encounter in this chapter can be categorized
as either a scalar or a vector quantity. A scalar is a quantity that can be completely
specified by its magnitude with appropr iate units; i.e., a scalar has only
magnitude and no direction. A vector is a physical quantity that requires
the specification of both magnitude and direction. Mass is an example of a scalar
quantity. If someone tells you that mass of an object is 2 kg, that information
completely specifies the mass of the object; no direction is required. Other
examples of scalar quantities are, temperature, time interval, the number of
students in a class, the volume of water in a bucket and the number of pages in
this book.
An example of a vector quantity is force. If your friend tells you that he is
going to exert a force of 5 N on an object, this is not enough information to let
you know what will happen to the object. The effect of a force of 5 N exerted
horizontally is different from the effect of a force of 5 N exerted vertically upward
or downward. In other words, you need to know the direction of the force as
well as its magnitude. Velocity is a vector quantity. If you wish to describe the
velocity of a moving vehicle, you must specify both its magnitude (say, 30 m/s)
and the direction in which the vehicle is moving (say, northeast). Other examples
of vector quantities include displacement and acceleration, which are defined in
this chapter. We use different symbols to represent scalar and vector quantities. A
scalar quantity is represented by an ordinary letter (such as a) or number (such as
5) with appropriate unit. 3 cm, 6 L, and 12 kg represent scalar quantities.
A vector quantity is represented by a symbol printed in lowercase / uppercase,
such as a or A. Since in handwriting, this representation is not practical, a common
notation is to indicate a vector quantity by an arrow over its symbol, a or A
. When we are interested only in the magnitude of a vector quantity, such as a,
we write it as a scalar (i.e., a) indicating that its direction is not being considered.
PHYSICS 107
Graphically, a vector is represented by an arrow. The length of the arrow is
proportional to the magnitude of the vector and the arrow points in the Brain Power On
direction of the vector. Figure (a) shows vector A and vector –A, both has the ..........................................
same magnitude but are in opposite directions.
The Race Track
(a) (b)
A A Players: Two or more
The players take turns in plotting
the position of their racing car
around a track.
A B
Description
(a) Vectors in same direction (b) Vectors in different directions
One of the players draws a track
Figure (b) shows vector A and another vector B whose magnitude is same on a piece of squared paper, with a
as that of A but direction is different. Scalars can be added and subtracted like line representing the start/finish line
ordinary numbers. Vectors follow different laws. However, vectors having same and two dots on the start/finish line
direction can be added easily. For example, sum of vector A and a vector C is a representing the cars; for example:
vector D whose magnitude is the sum of the magnitudes of vector A and C and
direction is the same as that of A.

A D= A+C

C
The players then take turns in
Addition of vectors moving their car along the track
Subtraction of vector C from vector A can be seen as addition of vector C to according to the following rules:
vector A as shown in Figure.The resultant has a magnitude equal to the difference • Each car is initially stationary.
of the magnitude of A and the magnitude of B. It points in the direction of A • The car moves the same direction
(the bigger of the two vectors).You will learn about other operations of vectors and distance as it did in its
in higher classes. previous move, or can accelerate
A A or decelerate by one square in any
direction.
B (-C)
C For example, suppose in the previous
B = A - C = A + (-C) move Blue moved three squares right
-C and two squares up:

Subtraction of vectors

Distance and Displacement


Displacement is a measurement of change in position of the particle in
motion. Its magnitude and direction are measured by the length and direction
The next move can be three squares
of the straight line joining initial and final positions of the particle. Obviously,
right and two squares up again, or
the length of the straight line between the positions is the shortest distance
any of the eight positions around
between the points. Displacement is a vector quantity. In SI units, it is measured
this:
in metres (m).
Let x1 and x2 be the positions of an object at time t1 and t2.Then its displacement,
denoted by ∆x (We use the Greek letter delta (∆) to denote a change in a
quantity.) in time ∆t = (t2 - t1), is given by the difference between the final
and initial positions :

∆x = x2 – x1; For example, this would be one


If x2 > x1 , ∆x is positive; and if x2 < x1 , ∆x is negative possible move:

108 CLASS - IX
MOTION IN A STRAIGHT LINE

Presently, we are dealing with motion along a straight line only. In one-
dimensional motion, there are only two directions (backward and forward,
upward and downward) in which an object can move, and these two directions
can easily be specified by + and – signs.
Consider the motion of a car along a straight line. We choose the x-axis such
that it coincides with the path of the car’s motion and origin of the axis as the
point from where the car started moving, i.e. the car was at x = 0 at t = 0 (refer
figure below). Let P, Q and R represent the positions of the car at different
At each move the starting point,
instants of time. The displacement of the car in moving from O to P is :
end point, and line segment joining
them, must stay within the bounds ∆x = x2 – x1 = (+360 m) – 0 m = +360 m
of the track. So, as in motor racing, a
The displacement has a magnitude of 360 m and is directed in the positive x
player must judge their acceleration
carefully to win the race without direction as indicated by the + sign. Similarly, the displacement of the car from P
going off the track. to Q is 240 m – 360 m = –120 m.
R O Q P
Also, a car may not move onto a
m
grid point currently occupied by the -160 -120 -80 -40 0 40 80 120 160 200 240 280 320 360 400
other car.This allows one car to force -x +x
the other car off the ideal route. x-axis, orgin and postions of a car at different times
The first player to complete a lap The negative sign indicates the direction of displacement. Thus, it is
and reach the finish line without not necessary to use vector notation for discussing motion of objects in one-
crashing into the sides of the track dimension.
wins.
Example The distance is the ‘path length’ travelled. Distance is a scalar quantity. The
magnitude of displacement may or may not be equal to the distance
The following diagram shows a traversed by an object. For example, for motion of the car from O to P, the
finished game in which the players distance is +360 m and the displacement is +360 m. In this case, the magnitude
raced clockwise around a circuit, and of displacement (360 m) is equal to the distance (360 m).
Red, the second player, won by one
move: But consider the motion of the car from O to P and back to Q. In this case,
the path length = (+360 m) + (+120 m) = + 480 m. However, the displacement
= (+240 m) – (0 m) = + 240 m. Thus, the magnitude of displacement (240 m) is
not equal to the path length (480 m).

The magnitude of the displacement for a course of motion may be zero but
the corresponding distance is not zero. For example, if the car starts from
Note: The track can be any desired O, goes to P and then returns to O, the final position coincides with the initial
shape. position and the displacement is zero. However, the distance of this journey is OP
+ PO = 360 m + 360 m = 720 m.
Mark distance and Example 01:
displacement in this maze
What is the distance covered and displacement of a car, (take data from
Figure)

S R O P Q

- X (m)- 100 - 80 - 60 - 40 - 20 0 20 40 60 80 100 +X(m)

a) If the car moves from O to P


b) If the car moves from O to P and then back to R

PHYSICS 109
Solution :
a) Distance covered = Length of path OP = 60 m
Displacement = Final position – Initial position
= + 60 m – (0 m) = + 60m
60 m is the magnitude of the displacement and + sign indicates that it is How about the distance between the
directed towards right or towards P. Earth and the Moon?
Note that in this case magnitude of displacement is equal to the distance.
This is so because the object does not change its direction during the course of
motion.

b) Distance covered = Length of path OP + Length of path PR


= 60 m + (60 m + 40 m) = 160 m
Displacement = Final position – Initial position
= (–40 m ) – (0 m ) = – 40 m
The minus sign shows that the direction of displacement is towards left or
towards – X direction.
Note that in this case, the magnitude of displacement (i.e. 40 m) is not equal
In most pictures, the Earth and Moon
to the distance (160 m).
are shown very close to each other.
Example 02: But, that’s really not the case. They’re
actually really far apart. The moon
A particle moves along a straight line so that its postion at time t
is an average of 384,400 km away.
seconds is x(t) meters, relative to the orgin. Assume that x(0)=0, x(3)
So let’s think about that distance
=2 and x(6)=-5 and that the particle only changes direction when t=3.
compared to the Sun. You could fit
Find the distance travelled by the particle from time t=0 to time t=6.
the distance between the Earth and
Solution : Sun with the distance of the Earth
The distance travelled is 2+7=9 meters. and Moon over 395 times! If it took
you one day to travel to the Moon, it
Speed and Velocity would take over 1 year to travel to the
Sun at that same rate!
An object in motion travels a given distance in a certain time interval. How
fast is the object moving? This is indicated by a quantity called speed. The speed
of an object is defined as the length of the path travelled per unit time.
Brain Power On
Path length or distance covered
Speed = ..........................................
Time taken
Since distance is a scalar quantity, speed is also a scalar quantity. Its SI unit
is m/s. It is also expressed in km h-1. For example, if a car covers a distance of 61
km in 2 h, its speed is 61 km / 2h = 30.5 km h-1.
The velocity of an object is defined as the displacement divided by the time
interval during which the displacement occurred:
Here’s a riddle for you: Two birds
Displacement leave the same tree at the same time.
Velocity = They both fly at 10 km/h for 1 hour,
Time taken
15 km/h for 30 minutes, and 5 km/h
Since displacement is a vector quantity, velocity is also a vector quantity. Its for 1 hour.Why don’t they end up at
unit is the same as that of speed, i.e., ms-1 or kmh-1. the same destination?

110 CLASS - IX
MOTION IN A STRAIGHT LINE

Example 03:
Consider a car moving towards + X - axis. It moves from O to A position +
900 m in 1 minute.
Then its displacement = + 900 m – (0 m ) = + 900 m.
The Global Positioning System Therefore, + 900 m
Velocity = = +15 ms–1
(GPS) 60 s
The magnitude of velocity is 15 m/s and it’s direction (as indicated by + sign)
is towards right or toward X-axis. Suppose the car travels back to origin O in 90 s.
Distance covered
Then, speed =
Time taken
1800 m
= 12 ms-1
150 s
Displacement
and,Velocity = Time taken
0m
= = 0 ms-1
150 s
1800 m
= = 12 ms-1
150 s
GPS was first used by US for
military purposes. Their need of a (Displacement is zero because final position coincides with the initial
global navigation system during the position).
cold war triggered the introduction
of GPS.In the beginning, GPS was
Types of Speed
called Navstar. The first Navstar Uniform speed
satellite was launched in 1978. Out of An object is said to be moving with uniform speed if it covers equal distances
the ten satellites launched, only nine in equal intervals of time. The figure shows the distance covered by a ball after
made it to the orbit. The first fully every 2 seconds.
developed GPS satellite was launched 10 m 10 m 10 m 10 m
in 1989.
2 sec 2 sec 2 sec 2 sec
In 1983, a Korean Airlines
Uniform speed
flight was shot down by Soviet
Union as the flight happened to enter The ball covers 10 m in every 2 seconds. The speed with which the ball is
the Soviet airspace. This incident moving at any point between A and E is 5 m/s.That is, the object is moving with
made President Ronald Reagan let uniform speed. An object is said to be moving with uniform speed if it covers
the free availability of GPS to the equal distances in equal intervals of time.
public. Though availability of GPS Variable speed or Non-uniform speed
was made public, high quality GPS For example, a rubber ball dropped from a height, on reaching the ground
signals were confined to military use bounces off to a height less than the initial height and slowly the height decreases.
until recently. However, on May 1, The distance covered by the ball in unit time decreases. That means the speed of
2000, this practice was taken off. At the ball varies from point to point. Such a speed is called a variable speed. An
present, the GPS for civilian purposes object is said to be moving with variable speed or non-uniform speed if it covers
is more accurate. equal distances in unequal intervals of time or vice-versa.
With 31 active satellites in
the orbit, GPS has become a part of
h
every gadget we use, ranging from
mobiles phones to cars.

h1 > h2 > h3 > h4

PHYSICS 111
Average speed and Instantaneous speed
When we travel in a vehicle the speed of the vehicle changes from time to
time depending upon the conditions existing on the road. In such a situation, the
speed is calculated by taking the ratio of the total distance traveled by the vehicle
to the total time taken for the journey. This is called the average speed. If an
object covers a distance s1 in time t1, distance s2 in time t2 and distance sn in time
tn then the average speed is given by,
S1 + S2 + S3 + ................... + Sn total distance traveled
Average speed = =
t1 + t2 + t3 + ................... + tn total time taken
When we say that the car is traveling at an average speed of 60 km/h it does
not mean that the car will be moving with the speed of 60 km/h throughout the
journey. The actual speed of the car may be less than or greater than the average
speed at a particular instant of time. Hence the speed that the body possesses at a
particular instant of time is called instantaneous speed.

Types of Velocity
Uniform velocity
When direction and magnitude of a moving object remain same then its
velocity is called uniform velocity. An object having uniform speed, may
not have uniform velocity. An object having uniform speed, may not have
uniform velocity.
V
V
F

F F

F
V
V

Non-Uniform Velocity
Non-uniform velocity
When both direction and magnitude or any one of them change with time
then its velocity is called non-uniform speed. Example: Motion of an object in
circular path with uniform speed is non-uniform velocity.
Average velocity
Average velocity is the total amount of distance from the starting point divided
The space shuttle is always
by the amount of time passed to get to the final destination.
launched in the same direction
Total displacement that the Earth rotates, thus taking
Average velocity =
Total time advantage of the Earth’s rotational
where x2 and x1 are the positions of the object at time t2 and t1, respectively. velocity (over 1,500 km/h east). This
Here the bar over the symbol for velocity is a standard notation used to indicate allows the shuttle to use less fuel to
an average quantity. reach space than if it had to achieve
x –x x
v = t2 – t 1 = such a great velocity on its own.
2 1 t
112 CLASS - IX
MOTION IN A STRAIGHT LINE

Example 04: A man walked from point A to F following the route in the grid
Brain Power On below in 3250 seconds. Determine the following
..........................................
1. As you sit in class reading this line, E F
you are traveling at a constant speed D C
as the Earth rotates on its axis. Your
speed depends on where you are. If
you are at the Equator, your speed A B
is 1670 km/h. At 42° latitude, your
speed is 1300 km/h. 0.5km

a) The average speed, in m/s, for the whole journey.


b) The magnitude of the displacement.
c) The magnitude of the average velocity, in m/s, for the whole journey.
Solutions:
24,394 miles
E F
D C

A H
• Do you feel the rotational motion B
of the Earth? Why or why not?
• What evidence do you have that a) 0.5km
you are moving?
distance
Average speed =
2. A person walking forward on the time
train says that he is moving at 2 km
per hour. A person on the platform 3 km + 1 km + 1.5 km + 0.5 km +0.5 km
=
says that the man in the train is 3250 s
moving at 72 km per hour.
6.5 km 6500 m
= =
3250 s 3250 s

= 2 m/s

b) The magnitude of the displacement is the distance between the starting point
A and the final point F and is the hypotenuse AF of the right triangle AFH and
is calculated using pythagora's theorm as follows
a) Which person is correct?
AF2 = FH2 + HA2 = 32 + 42 = 9 +16 = 25
b) How could you get the two men
DA = 25 = 5 km
to agree?
Total displacement 5 km 5000 m
c) average velocity = = = = 1.5 m / s
Total time 3250 s 32250 s

Relative Motion Velocity


The relative motion velocity refers to an object which is relative to some
other object that might be stationary, moving with the same velocity, or moving
slowly, moving with higher velocity or moving in the opposite direction.

PHYSICS 113
P Q Observer
Typical velocity magnitudes in m/s
3 ms-1 2 ms-1 A snail’s pace 10-3
V P/Q = 1 ms-1 A brisk walk 2

To the man Q, the girl P appears to move at a speed of 1 ms-1 towards Q. Fastest human 11
Relative to Q (observer), P is moving at a speed of 1 ms-1 due east as shown.
Freeway speeds 30

Acceleration Fastest car 341


You might have observed while pushing a bus it suddenly starts. Lift gives Random motion of
500
upward push when it starts. This is what acceleration is! Here velocity changes. air molecules
Hence the body accelerates. The rate of change of velocity of an object is Fastest airplane 1000
known as Acceleration. Thus,
Orbiting
Change in velocity v communications 3000
Acceleration = =
Change in time t satellite
Electron orbiting
Where Δv is the change in velocity and Δt is the change in the time. In 2 × 106
in a hydrogen atom
physics acceleration not only has a magnitude, but also has a direction. This Light traveling in
makes acceleration a vector. The SI unit and CGS unit for acceleration is m/s2 3 × 108
vacuum
and cm/s2, respectively.
You can calculate this from the above formula where velocity is meters per
second and time is in seconds.A positive sign of the magnitude of acceleration
shows increase in velocity and a negative sign shows decrease in velocity.
If there is decrease in acceleration, it is called Retardation.
When you apply brake to stop your car, you are decelerating.When a ball rolls
on the ground and slows down to a stop, the ball is decelerating. Deceleration
is the term used for acceleration that causes an object to slow down, while
acceleration is motion where the speed of an object changes every second.
However, in everyday usage, acceleration is usually taken to mean speeding up.
Suppose the velocity of a car changes from + 10 m/s to + 30 m/s in a time
interval of 2.0 s. Note that both velocities are towards the right, as indicated by +
signs. Therefore,
30 m/s − 10 m/s
Acceleration = = + 10 ms-2
2.0 s
The acceleration in the present case is +10 ms-2. This means that the car Acceleration
accelerates in the + x direction and its velocity increases at a rate of 10 ms-1 every
second. If the acceleration of an object during its motion is constant, we say that
the object is moving with uniform acceleration.
Example 05:
A neurotic tiger shark starts from rest and speeds up uniformly to 12 meters
per second in a time of 3 seconds. What was the magnitude of the average
acceleration of the tiger shark? Deceleration
Solution :
Start with the definition of acceleration.

114 CLASS - IX
MOTION IN A STRAIGHT LINE

V f – Vi
Class Room Activity-01 a=
t
The ball that rolls uphill m
The “Uphill Roller” is a beautiful
12 s - 0
a=
physics demonstration first reported by 3s
English mathematician William Leybourn m
a = 4 s2
in 1694. In the original version, a double
cone placed on two divergent inclined
ramps appears to roll “uphill”, apparently Example of Acceleration How Velocity Changes
violating the laws of physics.
A plane taking off Increase in speed

A car stopping at a stop sign Decrease in speed

Jogging on a winding trail Change in direction

Driving around a corner Change in direction

Standing at Earth’s equator Change in direction

Graphical Representation of Motion


A graph is a pictorial representation of the relation between two sets of data
For this amusing activity you need of which one set is of dependent variables and the other set is of independent
2 wooden pencils, a deck of cards and a
variables. To describe the motion of an object, we can use line graphs.
table tennis ball. Divide the deck in half
,placing the halves side by side as shown in In this case, line graphs show dependence of one physical quantity, such as
the figure a, Place the pencils on the half distance, velocity, or acceleration on another quantity, such as time. When a body
decks,the eraser ends touching and other
moves along a straight line path, its motion is said to be linear or rectilinear. The
ends on the table about 1.25 inches apart.
Put the ball at the center of the V to show, linear motion of a body can be studied with the help of the following graphs.
how it rolls downhill and off the pencils to 1. Distance–time graph
the table. Move the half-decks so they are
1.25 inches apart as shown in the figure . 2. Velocity–time graph
Rearrange the pencils so their erasers 3. Acceleration–time graph
touch on the table ,and their others ends
are 1.25 inches apart, each resting on the Distance or displacement time graph
inside edge of a half-deck. Place a weight
(a book) on the eraser ends to hold them Time is always plotted on the X-axis (bottom of the graph). The further to
firmly in the place. the right on the axis, the longer the time from the start. Distance or displacement
is plotted on the Y-axis (side of the graph). The higher up the graph, the further
A
A
A
AA
A
AA
A
A

AAA
AA

from the start.


AA
A
A
A
AA
A
A
A

AA
AAA
A
A
A
AA
A
AA
A
A

AAA
AA
AA
A
A
A
AA
A
A

If an object is not moving, a horizontal line is shown on a distance-time graph.


A

AA
AAA

Figure a Figure b Time is increasing to the right, but its distance does not change. It is not moving.
Place the ball in the middle of the We say it is at Rest.
pencil as shown. Which way will it roll?
Because the cards are raising the ends of
Distance

the pencils a fourth of an inch, you are


likely to guess that the back,as before,will
roll forward the pencil`s lower ends.
Surprisingly, it doesn't it seems to roll
uphill toward the pencil`s higher ends!
Actually of course, the ball`s center of Time
gravity lowers as it moves toward the open
end of the V, so it really rolling downhill. If an object is moving at a constant speed, it means it has the same increase in

PHYSICS 115
distance in a given time. Constant speed is shown by straight lines on a graph.
A steeper line indicates a larger distance moved in a given time. In other words, Brain Power On
higher speed. Both lines are straight, so both speeds are constant ..........................................
The distance–time graph for a girl
on a cycle ride is shown in graph.
Distance

Distance
60 F

50
E

distance / km
D
40
Time Time
30 B C
The line on this graph is curving upwards. This shows an increase in speed,
20 A
since the line is getting steeper: In other words, in a given time, the distance the

time of day
object moves is change (getting larger). It is accelerating. 10
0

m
m
m
m
m
m
Distance

1p
2p
3p
4p
5p
6p
a) How far did she travel?
.......................................................
.......................................................
Time
b) How long did she take?
Instantaneous velocities ......................................................
.......................................................
Calculating the gradient of a graph that does not have a convenient straight line
portion requires a tangent to be drawn to the curve. Consider the displacement- c) What was her average speed in
time graph below, which shows a constantly changing gradient indicating that km/h?
the velocity of the moving object is constantly changing. .......................................................
.......................................................
tangent
d) How many stops did she make?
Displacement

.......................................................
P .......................................................
rise
e) 
How long did she stop for
run altogether?
......................................................
Time .......................................................
slope or gradient of the tangent = Instantaneous velocity
f) W
 hat was her average speed
The instantaneous velocity of the moving object at point P will be given by excluding stops?
the gradient of the curve at this point. Calculating the gradient of the curve at this ......................................................
point is done by drawing a tangent to the curve. The tangent is the straight line .......................................................
that just touches the curve of the graph and has the same gradient as the graph at g) H
 ow can you tell from the shape
this point. The gradient of the tangent can then be calculated, by forming a large of the graph when she traveled
right angled triangle and reading the ‘rise’ and ‘run’ of the triangle. fastest? Over which stage did this
Example 06: happen?
.......................................................
The tables of results below were taken for an object being dropped and falling .......................................................
under gravity.
116 CLASS - IX
MOTION IN A STRAIGHT LINE

Distance fallen (m) 0.0 2.0 4.0 6.0 8.0 10.0


Brain Power On Time taken (s) 0.00 0.63 0.99 1.08 1.25 1.40
..........................................
(i) Plot a graph of distance fallen (on the vertical axis) against time taken (on
The approximate velocity–time
the horizontal axis).
graph for a car on a 5-hour journey
is shown in graph. (ii) Explain why your graph is not a straight line.
C D (iii)Calculate the velocity of the object after 1.00 second.
100
A B
speed (km/h)

80 Solution :
60
40
(i) 10
9
20 o E 8

distance fallen (m)


0 1 2 3 4 5 7
time/hours 6
5
a) State in which of the regions OA, 4
AB, BC, CD, DE the car is 3
2
(i) accelerating, 1
(ii) decelerating, 0 0.2 0.4 0.6 0.8 1.0 1.2 1.4
time (s)
(iii) traveling with uniform velocity.
....................................................... Use a graph sheet
....................................................... (i) The increasing gradient of the graph shows an increasing velocity, in other
....................................................... words acceleration.
.......................................................
(ii) The acceleration is caused by the gravitational force acting on the object.
b) Calculate the value of the (iii)The graph shown as the answer to part (i) has a tangent drawn at a time of
acceleration, deceleration or constant
1.00 second.
velocity in each region.
....................................................... Velocity = gradient of tangent
....................................................... rise 9.2 – 0
....................................................... run = 1.4 – 0.5 = 9.5 m/s
.......................................................
Speed or velocity-time graphs
c) What is the distance traveled over
each region? Speed-Time graphs look much like distance-time graphs. Be sure to read the
....................................................... labels. Time is plotted on the X-axis. Speed or velocity is plotted on the Y-axis.
....................................................... A straight horizontal line on a speed-time graph means that speed is constant.
....................................................... It is not changing over time. A straight line does not mean that the object is
....................................................... not moving. The second graph shows increasing speed. The moving object is
accelerating. The third graph shows decreasing speed. The moving object is
d) What is the total distance traveled?
decelerating.
.......................................................
.......................................................
.......................................................
.......................................................
Speed

Speed

Speed

e) Calculate the average velocity for


the whole journey.
.......................................................
....................................................... Time Time Time
....................................................... Both the dashed and solid line show increasing speed. Both lines reach the
.......................................................
same top speed, but the solid one takes longer. The dashed line shows a greater
acceleration.

PHYSICS 117
Note
..........................................

Speed
Acceleration is determined by the
slope of time-velocity graph.
Changein velocity
tan q=
Time Change in time

Acceleration (i) If the time velocity graph is a


straight line, acceleration remains
The gradient of a velocity-time graph is equal to acceleration. The height or constants.
‘rise’ and length or ‘run’ of a part of the graph is measured.
(ii) If the slope of the straight line is
Change in velocity rise positive, positive acceleration occurs.
Acceleration = = Slope of the tangent =
time run
(iii) If the slope of the straight line
constant velocity is negative, negative acceleration or
tangent
retardation occurs.
(iv) Larger the slope (tan θ) longer
Velocity

will be the straight line.


rise (v) If the time velocity graph is a
curve, then the acceleration changes
large acceleration continuously.
run Time

Area under the curve


Constant velocity
The sketch shows the velocity-time graph for a car that is travelling along a
motorway at a steady 70 km/h. The area under this graph is rectangular in shape.
The shaded area = 2× 70 = 140
This is the distance in kilometer travelled in 2 hours when the speed is 70 km/h.

Area under a velocity – time graph = distance travelled.

v km/h

70

Note
..........................................
0 2 t hours
1 When calculating the area from
Constant acceleration the graph, the unit of time must be
the same on both axes.
Now consider the case when a car is accelerating steadily from 0 to 45 km/h
in 10 seconds. The graph shows this situation, but note that the velocity is in 2 This rule for finding distances
kilometers per hour whilst the time is in seconds.The units need to be converted traveled is true even if the acceleration
in order to find the distance travelled. is not uniform.

118 CLASS - IX
MOTION IN A STRAIGHT LINE

Note Km/h
..........................................
To convert Km/hr into m/s, multiply 45
the number by 5 and then divide it
by 18.
To convert m/s to Km/hr, multiply
the number by 18 and then divide it 0 10 t seconds
by 5.
45 Km/h = 12.5 m/s
The distance travelled is given by the area under the graph:
10 × 12.5
Distance travelled = = 62.5 m.
2
Variable velocity and acceleration
The table and graph give the velocity of a car as it travels between 2 sets of
traffic lights. An estimate for the area under this graph can be found by splitting it
into strips as shown. The strips at each end are approximately triangular in shape
and each strip between them is approximately in the shape of a trapezium.
t(s) 0 2 4 6 8 10 12
v(ms ) 0 5 8 9
-1
8 5 0
v ms–1

C C
B B
A A
0 2 4 6 8 10 12
t seconds
2×5
Area of A = 2 =5
2(5+8)
Area of B = = 13
2
2(8+9)
Area of C = = 17
2
Total area = 5 + 13 + 17 + 13 + 5 = 70
This is an estimate of the distance travelled by the car (in meters) between the
2 sets of traffic lights. This is a simplified model of this situation. In practice the
change in velocity is unlikely to be so smooth and symmetrical. Note that better
estimates can be found by using more data and narrower strips.
Example 07:
A car is waiting at traffic lights. When the lights turn green, the car accelerates
uniformly from rest to a speed of 10 m/s in 20 s. This speed is then maintained
until the car passes a road sign 50 s after leaving the traffic lights.
1.Sketch a speed–time graph to illustrate the motion of the car.
2.Find the distance between the traffic lights and the road sign.

PHYSICS 119
Solution :
Model the car as a particle moving in a straight line
1. v(ms-1)
10

Want to lose weight fast? No


O 20 50 t(s) need to adjust your diet – just move
2. The distance travelled is given by to higher ground. This weight change
1 is the result of fluctuations in Earth’s
s= (a + b)h gravity, which a new high-resolution
2
map shows are greater than thought.
1
= (30 + 50)10 Gravity is often assumed to
2 be the same everywhere on Earth,
= 40 × 10 = 400 but it varies because the planet is not
The distance between the traffic lights and the road sign is 400 m perfectly spherical or uniformly dense.
In addition, gravity is weaker at
Acceleration due to gravity the equator due to centrifugal forces
We know that the earth attracts everybody towards us centre and applies a produced by the planet’s rotation. It’s
pull force on the body known as gravitational force. The acceleration in the also weaker at higher altitudes, further
falling body due to gravitational pull is called acceleration due to gravity from Earth’s centre, such as at the
and is denoted by g. You must be surprised to know that all bodies irrespective summit of Mount Everest.
of their size and mass fall towards earth with same acceleration g = 9.8 m s–2.
Similarly when these bodies are thrown up they move with the same negative
acceleration (retardation) g = - 9.8 m s–2.
t=0 Initial v = 0
t=1s v = 9.8 ms-1

t=2s v = 19.6 ms-1

Motion due to gravity


It means that velocity of a falling body increased by 9.8 m/s in every second
of its motion.
Starting from the rest A gravity model of the Earth computed
by University of Texas Center for Space
After t = 1 s, the velocity of the particle = 9.8 × 1 = 9.8 m s–2. Research and NASA.
After t = 2 s, the velocity = 9.8 × 2 = 19.6 m s–1.
According to the data available
After t = 3 s, the velocity = 9.8 × 3 = 29.4 m s–1 and so on. in this map,Mount Nevado Huascarán
In uniform acceleration magnitude of acceleration remains constant. in Peru has the lowest gravitational
For example, the magnitude of acceleration due to gravity is always 9.8 m s–2. acceleration, at 9.7639 m/s2, while the
However, in non-uniform acceleration magnitude of acceleration varies. highest is at the surface of the Arctic
For example auto vehicles move with variable acceleration on crowded road. Ocean, at 9.8337 m/s2.

Equations of uniformly accelerated motion


Relation among velocity, distance, time, and acceleration is called
equations of motion. There are three equations of motion. Consider a body
of mass m having initial velocity u. Let after time t its final velocity becomes v
due to uniform acceleration a and distance travelled in time t is s.
120 CLASS - IX
MOTION IN A STRAIGHT LINE

First equation of Motion (Equation for position-velocity relation)


We know that:
Final velocity - Initial velocity
Acceleration =
Time - tacken
Testing Humans at High v –u
a=
Accelerations t
at = v – u
v = u + at
This is the first equation of motion.

Example 08:
A particle is moving in a straight line from A to B with constant acceleration 3 m/s2.
Its speed at A is 2 m/s and it takes 8 seconds to move from A to B. Find
1. the speed of the particle at B,
2. the distance from A to B.
Solution
2ms-1
In experiments carried out
by the U.S. Air Force in the 1940s A B
and 1950s, humans riding a rocket a = 3, u = 2, t = 8, v = ? s =?
sled could withstand accelerations as 1. v = u + at
great as 440 m/s2. = 2 + (3 × 8)
The first three photos in this sequence = 2 + (3 × 8)
show Air Force physician John Stapp = 26
speeding up from rest to 188 ms The speed of the particle at B is 26 ms-1
(678 km/h ) in just 5 s. Photos 4–6  u+v 
show the even greater magnitude of 2. S =  t
 2 
acceleration as the rocket sled braked
2 + 26 
to a halt. =  × 8
 2 
= 112
The distance from A to B is 112 m.

Second equation of Motion (Equation for position-time relation)


We know that,
Initial velocity + Final velocity
Average velocity =
2
u+v
Average velocity =
2
Also, Distance = Average velocity × Time
u×v ×t
s=
2
PHYSICS 121
After substituting the value of “v” from first equation of motion we get,
Brain Power On
u + (u + at)
s= ×t ..........................................
2
1. You have 25 horses, you want to
2u + at × t pick the fastest 3 horses out of those
s=
2 25. In each race, only 5 horses can
1 run at the same time. What is the
s = ut + at2
2 minimum number of races required
The above equation is known as the second equation of motion. to find the 3 fastest horses without
using a stopwatch?
Example 09:
2. If an aeroplane makes a round
A car is traveling along a straight horizontal road with a constant acceleration trip and a wind is blowing, is the trip
of 0.75 m/s. A car is traveling along a straight horizontal road with a constant time shorter, longer or the same?
acceleration of 0.75 m/s2.The car is traveling at 8 m/s when it passes a pillar box. 12
3. The diagram shows the tracks
seconds later it passes a lamp post. Find
from a two wheeled bicycle. Was the
1.The distance between the pillar box and the lamp post, bicycle traveling from left to right or
right to left?
2. The speed with which the car passes the lamp post..The car is traveling at 8 m/s
when it passes a pillar box. 12 seconds later it passes a lamp post.
Solution
Positive direction
8 ms-1

s
1.a = 0.75, u = 8, t = 12, s = ?
1 2
s = ut + at
2
1
= 8 × 12 + × 0.75 + 122 4. A man has a rectangular garden,
2
55 m ('B' in the diagram,) by 40
= 96 + 54 = 150 m('A',) and he makes a diagonal path,
The distance between the piller box and the lamp post is 150 m 1 m wide, exactly in the manner
2. a = 0.75, u = 8, t = 12,v = ? indicated in the diagram.What is the
area of the path?
v = u + at
= 8 + 0.75 × 12
= 17 ms-1
The speed of the car at the lamp post is 17 ms-1

Third equation of Motion (Equation for position velocity relation)


From first equation of motion:
v = u + at
v-u
t=
a
We know that the second equation of motion is,

1 at2
s = ut +
2

122 CLASS - IX
MOTION IN A STRAIGHT LINE

By substituting the value of t, we get


2
æ v - u ö÷ 1 æ v - u ö÷
s = u çç + aç
çè a ÷÷ø 2 ççè a ÷÷ø
v - u 1 (v- u)2
s = u´ + a
a 2 a2
2 (uv - u2 ) + ( v - u)
2

s=
2a
2as = 2uv - 2u2 + v 2 + u2 - 2uv
2as + u2 = v 2
v 2 = u2 + 2as

This is called the third equation of


motion.
Example 10:
A ball B is projected vertically upwards
from a point O with speed 12 m/s. Find
1.The greatest height above O reached
by B,
2.The total time before B returns to O.
Solution:
Greatest height

Positive
direction hm
12 ms-2

1. u = 12, v= 0, a = -9.8 , s = h,
v2 = u2 + 2as
02 = 122 - 2 × 9.8 × h
122 144
h= = = 7.4
2 × 9.8 19.6
The greatest height above 0 reached
by B is 7.4 m
2. s = 0, u = 12, a= -9.8, t = ?
1 2
s = ut + at
2
yellow.
Traditional equations
are

for
Rest are the rearranged

solving problems. The


"without"
should be read as, the
equation without the

1
0 = 12t - × 9.8 × t = t(12 - 4.9t)
2
highlighted term.
forms, useful
motions

12
t= = 2.4
highlighted

4.9
column

The time taken for B to return to


0 is 2.4 s
of

PHYSICS 123
Equations of motion by graphical method
We already know about equations of motion when an object moves along
a straight line with uniform acceleration. We already know how to derive them
but these equations can also be derived by graphical method.

Equation for velocity time relation


Consider the velocity–time graph of an object that moves under uniform
acceleration as shown below in the Graph.
E B

Velocity (v)
From this graph, you can see that initial velocity of the object is u (at
point A) and then it increases to v (at point B) in time t. The velocity changes at
a uniform rate a. Again from the graph it is clear that time t is represented by v
OC, initial velocity u by OA, and final velocity of object after time t by BC. D
A
From graph , it is clear that: BC = BD + DC = BD + OA u
So we have,
O Time t C
v = BD + u
We should now find out the value of BD. From the velocity–time graph v–t graph
the acceleration of the object is given by
Change in velocity BD BD BD
a = = = =
time taken AD OC t

This gives, BD = at
Put this value of BD in equation we get
v = u + at
This is the equation for velocity–time relation.

Example 11:
From the diagram given below which is a v - t graph of a body,

10 B

v 5 C
A D

F E
0 3 4 5
t
(i) Find the deceleration of the body in the region BC.
(ii) find the toatal distance travelled bt the body.

Solution:
Given v is in ms-1 and time in s
(10 − 5) ms −1
Deceleration = slope of BC = = 5 ms-1
(4 − 3) s
Total distance travelled = Area of the triangle ABC + Area of rectangle OADE
æ1 ö
S =çç ´4´5÷÷÷ + (5´5) = 35 m
çè 2 ø

124 CLASS - IX
MOTION IN A STRAIGHT LINE

Equation for position time relation


Let us consider that the object has travelled a distance s in time t under uniform
acceleration a. The distance travelled by the object is obtained by the area enclosed
within OABC under the velocity-time graph AB. Thus, the distance s travelled by
the object is given by
Zero gravity flight
Almost everyone dreams of s = Area OABC
floating effortlessly like astronauts in = Area of the  OADC + Area of the  ABD
space. Currently, NASA uses a modified
C-9 plane to create simulations of 1
So, s = OA × OC + (AD × BD)
a weightless environment, both for 2
training purposes and to conduct Substituting OA = u, OC=AD= t and BD= at,
weightlessness experiments. Until
recently, only a select few had the 1
we get, s = (u × t ) + × (t × at)
privilege of experiencing these flights. 2
Today, a company called Zero Gravity 1 2
Corporation (ZERO-G) offers this s = ut + at
2
experience to the general public.
This is the equation of position–time relation.

Equation for position velocity relation


Again considering graph. We know that the distance travelled (s) by a body in
time t is given by the area under line AB which is area of trapezium OABC. So
we have, distance traveled (s)= Area of trapezium OABC
(sum of pareallel sides ) × height
s=
2
(OA + CB) X OC
=
2
Since, OA+CB = u + v and OC=t
( u + v) t
s=
2
From velocity–time relation
v− u
t=
a
Substituting, t in equation for s. we get,

s = (u + v) × (v − u)
2 a

or v2 = u2+2as

(0,0) u=0 Body falling freely under gravity


A body released near the surface of the earth is accelerated downward under
H the influence of force of gravity.

(a) Time of Flight


1
From equation S = ut + at2
2

PHYSICS 125
S = -H, u = 0, a = -g
t = T (Let assume)
1 2H
-H = (0) T - gt2 ⇒ T =
2 g
(b) Final Velocity when body reaches the ground from

v2 – u2 = 2as

S = - H, v = v, u = 0, a = -g,

vf2 - 0 = 2(-g) ⇒ (-H) v f 2gh

(i) Assuming u = 0 for a freely falling body:


1
As h =   gt2 i.e h α t2
2
Distance fallen in time t, 2t, 3t etc will be in the ratio of 12 : 22 : 32 ..... i.e. square
of integers.
1
(ii) The distance fallen in nth sec = g (2n - 1)
2
So distance fallen in Ist, 2nd, 3rd sec will be in the ratio 1:3: 5 i.e. odd integers only.

Body is projected vertically up


It includes two types of motion
(i) Decelerated motion from A to B because the direction of velocity and
acceleration is opposite. So speed decreases
(ii) Accelerated motion from B to C because the direction of velocity and
acceleration is same (downward). So speed increases
B
upward
motion downward motion
(deaccelerated (accelerated motion)
motion) u
A u

(a) Time of flight:


It is the time taken by the particle to reach the ground. If the particle is thrown
vertically upward with initial velocity u then
ui = u
a = – g (take downward direction negative)
From equation,
1 2
S = ut + at ⇒ Snet = 0 (when particle again reaches the ground)
2
t = T (time of Flight)
1 2
0 = uT - gt
2

126 CLASS - IX
MOTION IN A STRAIGHT LINE

2u
T=
g

(b) Maximum Height:


From v2 - u2 + 2as
at maximum height v = 0, s = Hmax

⇒ 0 = u2 - 2 gHmax

u2
Hmax =
2g

(c) Final Velocity


From v = u + at
2u
v = vf , a = -g, t = T =
g

æ 2u ö
vf = u - g çç ÷÷÷
çè g ÷ø

i.e. the body reaches the ground with the same speed with which it was thrown
vertically upwards as it thrown vertically upward.
Taking initial position as origin and direction of motion (i.e. vertically up) as
positive.
(a) At the highest point v = 0 (b) a=–g

V C It is clear that in case of motion under gravity:


D
(a) Time taken to go up is equal to the time taken to fall down through the same
B distance.
V
E
V
(b) The speed with which a body is projected up is equal to the speed with which
it comes back to the point of projection.
A
(c) The body returns to the starting point with the same speed with which it was
V thrown.

Note Uniform Circular motion


..........................................
In the previous topics, we studied about the change in the magnitude of
(i) Circular motion in horizontal velocity when a body undergoes rectilinear motion. Now we will consider the
plane is usually uniform circular situation where a body can be accelerated without changing the magnitude of
motion. velocity.
(ii) Remember that equations of We observe that any ,body moving along a circular path changes its direction
motion are not applicable for circular continuously, even though the speed is constant. Thus, the body is continuously
motion. accelerated. This acceleration acts towards the centre of the circular path and
(iii)Centripetal force: is called centripetal acceleration.The direction of centripetal acceleration is
along the radius towards the centre.
a. Always acts towards centre.
b. 
Centripetal force is required to v2
ac =
move a particle in a circle r

PHYSICS 127
If a body moves along a circular path with constant speed such that its
acceleration is uniform, then the body is said to be in uniform circular motion

To Calculate the Speed of a Body in Uniform Circular Motion


When a body takes one round of a circular path, then it travels a distance equal
to its ‘circumference’ which is given by 2πr , where r is the radius of the circular
path (see figure). The speed of a body (or object) moving along a circular path is
given by the formula:
2pr
v=
t
Where v = speed
22
π =
7
r = radius of circular path
t = time taken for one round of circular path

r
(radius)
Circumference
of circular path
A circular path of radius r. The circumference of this circular path is 2πr
Example 12:
A cyclist goes around a circular track once every 2 minutes. If the radius of the
22
circular track is 105 metres, calculate his speed. (Given π = )]
7
Solution:
Brain Power On
We know that for a body moving in a circular path:
..........................................
2pr
v= On a circular path two person are
t
walking in anti direction. How many
Here, Speed, v = ? (To be calculated) times they will cross each other in
one full round ?
22
π= L meters
7
Radius of circular track, r = 105 m
And, Time taken for 1 round, t = 2 minutes
= 2 × 60 seconds
= 120 s
Now, putting these values of π, r and t in the above formula, we get:
2 ´ 22 ´ 105
v=
7 ´ 120
= 5.5 m/s
Thus, the speed of cyclist on the circular track is 5.5 metres per B A
second.

128 CLASS - IX
M I N D M A P

Motion

Uniform Motion Non-Uniform Motion

Along a Along a circular path Description of motion


stright line (uniform circular motion)

Speed Velocity Acceleration

Uniform Average Non-Uniform Uniform Average Non-Uniform Uniform Non-Uniform


Speed Speed Speed velocity velocity velocity acceleration acceleration

Graphical Representation of motion

Distance-time graph Velocity-time graph

Object moving with Object moving with Object moving with Object moving with Object moving with
uniform speed non-uniform speed uniform acceleration non-uniform acceleration zero acceleration

Equation of motion

Velocity-time relation Position-time relation Position-velocity relation

PHYSICS
v = u + at 1 2 v2 - u2 = 2as
S = ut + at
2

129
MOTION IN A STRAIGHT LINE
Do at Class!
I. Only One Option Correct Type:
1. A body goes from A to B with a velocity of 20 m/s (a) Uniform acceleration
and comes back B to A with a velocity of 30 m/s. (b) Non-uniform retardation
The average velocity of the body during the whole (c) Uniform speed
journey is (d) Initial velocity OA and is moving with uniform
(a) Zero (b) 25 m/s retardation
(b) 24 m/s (d) None of these 8. In the given velocity-time graph, AB shows that
2. If an object covering distances in direct proportion the body has
to the square of the time lapsed, then the A B
acceleration is V
(a) Increasing (b) Decreasing
(c) Constant (d) None of these

3. 
Distance travelled by a freely falling body is O t
proportional to .....................
(a) Mass of the body (a) Uniform acceleration
(b) Square of the acceleration due to gravity (b) Uniform retardation
(c) Square of the time of fall (c) Uniform velocity throughout its motion and has
(d) Time of fall zero initial velocity
(d) None of these
4. The rate of change of displacement with time is __
9. If the displacement-time graph for the two particles
(a) Speed (b) Acceleration A and B are straight lines inclined at angles of
(c) Retardation (d) Velocity 30° and 60° with the time axis, then ratio of the
5. Which of the following is not vector quantity? velocities vA: vB will be .............

(a) Retardation A
Displacement

(b) Acceleration due to gravity


(c) Average speed B
(d) Displacement 60o
30o
6. 
If the time-displacement graph of a particle X
is parallel to the time-axis, then velocity of the O Time
particle is .................... (a) 1 : 2 (b) 1 : 3
(a) Infinity (c) 3 : 1 (d) 3: 1
(b) Unity
10. In the given figure, velocity of the body at is ..........
(c) Equal to acceleration of the body
(d) Zero A
Displacement

7. In the velocity-time graph, AB shows that the body


has .............
A
V
O Time B
(a) Zero (b) Unity
B
O (c) Maximum (d) Infinite
t
130 CLASS - IX
MOTION IN A STRAIGHT LINE

11. 
The velocity-time graph for a body with non- 18. Driver of a train travelling at 115 km/hr sees on
uniform motion is a ............... a same track, 100m in front of him, a slow train
(a) Curved line travelling in the same direction at 25 km/hr. The
(b) Straight line parallel to x-axis least retardation that must be applied to faster train
(c) Straight line parallel to y-axis to avoid a collision is .................
(d) None of these (a) 3.125 m/s2 (b) 3.5 m/s2
(c) 2.75 m/s2 (d) 3.0 m/s2
12. Area under a velocity-time graph gives .................
(a) Time taken by a moving object 19. Distance of the moon from the earth is 4 × 108 m.
(b) Distance travelled by a moving object The time taken by a radar signal transmitted from
(c) Acceleration of moving object the earth to reach the moon is .................
(d) Retardation of a moving object (a) 5.2 s (b) 1.3 s
(c) 2.6 s (d) 0.70 s
13. If a body is thrown up with an initial velocity u
and covers a maximum height of h, then h is equal 20. A stone is dropped into a well in which the level of
to ......................... water is h, below the top of the well. If v is velocity
(a) 16 m/s (b) 15 m/s of sound, then time T after which the splash is
(c) 14 m/s (d) 12 m/s heard is equal to .......................
2h 2h h
(a) (b) +
14. A body is thrown vertically upwards and rises to a v v g
height of 10m. The velocity with which the body
2h h h 2h
was thrown upwards is (g = 9.8 m/s2) (c) + (d) +
u2 u g v 2g v
(a) (b)
2g 2g 21. A stone weighing 3 kg falls from the top of a tower
(c) 2u g
2
(d) None of these 100 m high and buries itself 2 m deep in the sand.
The time of penetration is ...................
15. A truck running along a straight line increases its
speed uniformly from 30 m/s to 60 m/s over a (a) 0.09 sec (b) 0.9 sec
time interval 1 min. The distance travelled during (c) 2.1 sec (d) 1.3 sec
this time interval is
22. The velocity of a body at any instant is 10 m/s.
(a) 900 m (b) 1800 m After 5 sec, velocity of the particle is 20 m/s. The
(c) 2700 m (d) 3600m velocity at 3 seconds before is ...............
16. A car travels one third distance on a straight road (a) 8 m/sec (b) 4 m/sec
with a velocity of 10 km/hr, next one third with (c) 6 m/sec (d) 7 m/sec
velocity 20 km/hr and the last one third with
23. A body covers 200 cm in the first 2 sec, and 220
velocity 60 km/hr. What is the average velocity of
cm in next 4 sec. What is the velocity of the body
the car in the whole journey?
at the' end of 7th second?
(a) 4 km/hr (b) 6 km/hr
(a) 40 cm/sec (b) 20 cm/sec
(c) 12 km/hr (d) 18 km/hr
(c) 10 cm/sec (d) 5 cm/sec
17. A motor ship covers the distance of 300 km between
24. If two bodies of different masses m1 and m2 are
two localities on a river in 10 hrsdownstream and
dropped from different heights h1 and h2, then
in 12 hrs upstream. Find the flow velocity of the
ratio of the times taken by the two to drop through
river assuming that these velocities are constant.
these distances is .....................
(a) 2.0 km/hr (b) 2.5 km/hr
(a) h1 : h2 (b) h1 / h2
(c) 3 km/hr (d) 3.5 km/hr
(c) h1 : h 2 (d) h12 : h22

PHYSICS 131
25. Name the instrument used to measure instantaneous 32. The acceleration of a point on the rim of flywheel
speed of a vehicle. 1 m in diameter, if it makes 1200 revolutions per
minute is ...............
(a) Accelerator (b) Speedometer
(c) Ammeter (d) Multicenter

26. A ball is dropped on the floor from a height of 10


m. It rebounds to a height of 2.5 m. If the ball is
in contact with the floor for 0.01 sec, then average
acceleration during contact is .................
(a) 2100 m/s2 (b) 1400 m/s2
(c) 700 m/s2 (d) 400 m/s2 (a) 8 π2 m/s2 (b) 80π2 m/s2
27. A stone is thrown vertically upward with an initial (c) 800 π2 m/s2 (d) none of these
velocity u from the top of a tower, reaches the 33. A phonograph record on turn table rotates at 30
ground with a velocity 3u. The height of the tower rpm. The linear speed of a point on the record
is ..................... at the needle at the beginning of the recording
3u2 4u2 when it is at a distance of 14 cm from the centre is
(a) (b)
g g ..................

6u2 9u2
(c) (d)
g g
28. If a ball is thrown up with a certain velocity. It
attains a height of 40 m and comes back to the
thrower, then .................
(a) Total distance covered by it is 40 m
(b) Total displacement covered by it is 80 m (a) 22 cm/sec (b) 44 cm/sec
(c) Total displacement is zero (c) 48 cm/sec (d) 52 cm/sec
(d) Total distance covered by it is zero
34. The relationship between average speed, time and
29. Acceleration of a body projected upwards with a distance is
certain velocity is ..................
(a) Average speed = distance × time
(a) 9.8 m/s2 (b) – 9.8 m/s2 (b) Average speed =total distance / total time
(c) Zero (d) Insufficient data (c) Time = average speed/distance
30. If a body of mass 0.10 kg is moving on circular (d) Distance = average speed × time
path of diameter 1.0 m at the rate of 10 revolutions 35. A body moving along a circular path has .................
per 31.4 sec, then centripetal force acting on the
body is ................ (a) Constant speed
(b) Constant velocity
(a) 0.2 N (b) 2.0 N (c) No radial acceleration
(c) 0.02 N (d) 20.0 N (d) No tangential velocity
31. 
The earth's radius is 6400 km. It makes one 36. A rubber ball dropped from a certain height is an
revolution about its own axis in 24 hrs. The example of ...................
centripetal acceleration of a point on its equator is
nearly .............. (a) Uniform acceleration
(b) Uniform retardation
(a) 340 cm/s2 (b) 34 cm/s2 (c) Uniform speed
(c) 3.4 cm/s2 (d) 0.34 cm/s2 (d) Non-uniform speed

132 CLASS - IX
MOTION IN A STRAIGHT LINE

37. 
If the velocity of a body does not change, its 2. Aadhan, went towards the east, after travelling 60
acceleration is ...................... m he turned towards the north and travelled 80 m
as in figure. Find
(a) Zero (b) Infinite
(c) Unity (d) None of these North
C

38. 
When the distance an object travels is directly 80 m
West East
A 60 m B
proportional to the length of time, it is said to
travel with ...............
(a) Zero velocity (i) Distance travelled from A to C
(b) Constant speed (ii) Displacement from A to C
(c) Constant acceleration 3. 
The graph (figure 1) given below show the
(d) Uniform velocity positions of a body at different times. Calculate the
speed of the body as it moves from
39. 
A body moves on three quarters of a circle of
D
radius r. The displacement and distance travelled 8
7

displacement(m)
B C
by it are: ..................... 6
B C

velocity (km/h)

velocity (km/h)
5 40 40
4 30 A 30
3
(a) Displacement = r, distance = 3r 2 20 20
1 A 10 D 10
3pr 1 2 3 4 5 6 7 8 1 2 3 4
(b) Displacement 2r distance = 1 2 3 4 5 6 7
time (s) time (h) time (h)
2
figure 1 figure 2 figure 3
3pr
(c) Displacement 2r, distance =
2 (i) A to B (ii) B to C
3pr (iii) C to D
(d) Displacement 0, distance =
2 4. 
From the velocity-time graph (Figure 2) shown
40. For the motion on a straight line path with constant above, find
acceleration the ratio of the magnitude of the
displacement to the distance covered is .............. (i) The velocity at point C.
(ii) The acceleration of the body between A and B.
(a) = 1 (b) ≥ 1 (iii) The acceleration of the body between B and C.
(c) ≤ 1 (d) < 1 (iv) The acceleration of the body between C and D.
II. Graph based Questions 5. 
From the velocity-time graph (Figure 3) shown
above. Find the
1. A particle is moving in a circle as shown in figure.
Find: (i) Acceleration (ii) Retardation and
(iii) Distance
B
6. From the velocity-time graph given below, calculate
B
25
2m
20
velocity (m/s)

C A
15 C
10
5 D
D A
1 2 3 4 5 6
(i) Distance travelled from C to A time (s)
(ii) Distance travelled from C to C (i) The acceleration from A to B.
(iii) Displacement from C to A (ii) The retardation from B to C.
(iii) The distance covered in the region ABCD.
(iv) Displacement from C to C (iv) The average velocity from C to D.

PHYSICS 133
7. 
A body is moving in a straight line and its 3. If t1 = 2s and t2 = 4s, find acceleration for each of
displacements at various instants of time are given the following cases:
in the following table:
(i) v1 = 8 m/s, v2 = 12 m/s
Time (s) 0 1 2 3 4 5 6 7 (ii) v1 = 16 m/s, v2 = 12 m/s
Displacement (m) 4 6 10 10 10 16 20 24
4. A body moves along a straight line with constant
Plot the displacement-time graph and calculate the acceleration a = 4 m/s2. The initial velocity is 3
average velocity, in the time interval of 1 s to 5 s. m/s, find:
8. The following table represents the distance of a car (i) velocity at t = 2s
with time in a fixed direction. (ii) distance travelled by body when its velocity is 5
m/s
Time (s) 0 1 2 3 4
Displacement (m) 0 10 20 30 40 5. A body starts from rest, moves in a straight line
with constant acceleration, and covers a distance of
Draw distance-time graph and with its help find 64 m in 4s.
(i) The distance travelled by the car at the end of (i) Find acceleration of the body
2.5 s. (ii) What is the final velocity?
(ii) The speed of the car. (iii) How much time was required to cover half the
total distance?
9. 
A boy moves on the path ABCD. What is
total distance moved by boy? What is his net 6. A farmer moves along the boundary of a square
displacement? field of side 10 m in 40 s. What will be the
A magnitude of displacement of the farmer at the
D end of 2 minutes 20 seconds?
50 m
20 m
7. During an experiment, a signal from a spaceship
B 40 m C reached the ground station in five minutes. What
was the distance of the spaceship from the ground
10. Speed-time graph of a particle is shown in a figure. station? The signal travels at the speed of light, that
Discuss the motion of this particle. Draw is 3 × 108 ms–1.
30
Speed 8. A bus decreases its speed from 80 km h–1 to 60 km
h–1 in 5 s. Find the acceleration of the bus.
m s-1 9. 
A bus starting from rest moves with a uniform
3 6 acceleration of 0.1 ms–2 for 2 minutes. Find(a) the
time(s) speed acquired. (b) The distance travelled
(i) velocity-time 10. A stone is thrown in a vertically upward direction
(ii) acceleration-time graphs for the same motion. with a velocity of 5 ms–1. If the acceleration of the
stone during its motion is 10 ms–2 in the downward
III. Numerical problems
direction, what will be the height attained by the
1. A car travels along a straight line at a speed of 40 stone and how much time will it take to reach
km/hr from A to B and returns from B to A at a there?
speed of 60 km/hr. Find the average speed of the
car and its average velocity. IV. Statement type
2. A car travelled the first third of a distance x at a Directions: In each of the following questions, a statement
speed of 10 km/hr. The second third at a speed of of Assertion (A) is given followed by a corresponding
20 km/hr and the last third at a speed of 60 km/ statement of Reason (R) just below it. Of the statements,
hr. Find the average speed over the entire distance mark the correct answer as
x.
134 CLASS - IX
MOTION IN A STRAIGHT LINE

(a) If both assertion and reason are true and reason is time axis.
the correct explanation of assertion Reason(R) : It is possible that position of a body
(b) If both assertion and reason are true but reason is does not change with time.
not the correct explanation of assertion 9. Assertion(A) : Displacement of a body is the signed
(c) If assertion is true but reason is false sum of the area under velocity-time
(d) If assertion is false but reason is true. graph.
1. Assertion(A) : The distance "x" in which a car can
Reason(R) : D
 isplacement is a vector quantity.
be stopped depends on the initial
velocity. 10. Assertion(A) : 
The average speed of an object
may be equal to arithmetic mean of
Reason(R) : Change in mass has no role to play.
individual speed.
2. Assertion(A) : A body can have acceleration even if
Reason(R) : 
Average speed is equal to total
its velocity is zero at a given instant
distance travelled divided by total
of time.
time taken.
Reason(R) : A body is momentarily at rest when
11. Assertion(A) : The speedometer of an automobile
it reverses its direction of motion.
measure the average speed of the
3. Assertion(A) : Displacement of a body may be zero automobile.
when distance travelled by it is not
Reason(R) : 
Average velocity is equal to total
zero.
displacement divided by total time
Reason(R) : 
The displacement is the longest taken.
distance between initial and final
12. Assertion(A) : A negative acceleration of a body is
position.
associated with a slowing down of a
4. Assertion(A) : 
The relative velocity between any body.
two bodies moving in opposite
Reason(R) : Acceleration is vector quantity.
direction is equal to sum of the
velocities of two bodies. 13. Assertion(A) : 
Motion with uniform velocity is
always along a straight line path.
Reason(R) : S
 ometimes relative velocity between
two bodies is equal to difference in Reason(R) : 
In such a motion speed is the
velocities of the two bodies. magnitude of the velocity and is
equal to the instantaneous velocity.
5. Assertion(A) : 
Velocity-time graph for an object
in uniform motion along a straight 14. Assertion(A) : When an object returns to its initial
path is a straight line parallel to the position, its displacement is zero, but
time axis. the distance covered is not zero.
Reason(R) : 
In uniform motion of an object Reason(R) : It means that its average velocity and
velocity increases as the square of average speed are both zero.
time elapsed. 15. Assertion(A) : The accelerated motion of an object
6. Assertion(A) : 
A body may be accelerated even may be due to change in magnitude
when it is moving at uniform speed. of velocity or direction of velocity
or both.
Reason(R) : 
When direction of motion of the
body is changing then body may Reason(R) : Acceleration can be produced only
have acceleration. by change in the magnitude of the
velocity.
7. Assertion(A) : The motion of a body moving in a
circular path with constant speed is 16. Assertion(A) : 
Displacement of an object is the
an example of variable acceleration. algebraic sum of the area under
velocity-time graph.
Reason(R) : Acceleration varies due to change in
direction. Reason(R) : Displacement is the shortest distance
between the initial and final position.
8. Assertion(A) : The position-time graph of a body
may be a straight line parallel to
PHYSICS 135
Do at Home!
I. Choose the correct option:
1. 
A bus increases its speed from 36 km/h in 10 7. A car of mass 1000 kg is moving with a velocity
seconds. Its acceleration is: of 1 ms–1. If the velocity-time graph for this car is
a horizontal line parallel to the time axis, then the
(a) 5 m/s2 (b) 2 m/s2 velocity of car at the end of 25 s will be:
(c) 3.6 m/s2 (d) 1 m/s2
(a) 25 ms–1 (b) 40 ms–1
2 
A bus moving along a straight line at 20 m/s (c) 10 ms–1 (d) 250 ms–1
undergoes an acceleration of 4m/s2. After 2 seconds,
its speed will be: 8. A motorcycle is being driven at a speed of 20 m/s
when brakes are applied to bring it to rest in five
(a) 8 m/s (b) 12 m/s seconds. The deceleration produced in this case
(c) 16 m/s (d) 28 m/s will be:
3. The slope of a speed-time graph gives: (a) +4 m/s2 (b) –4 m/s2
(a) Distance travelled (b) Velocity (c) + 0.25 m/s2 (d) –0.25 m/s2
(c) Acceleration (d) Displacement 9. A sprinter is running along the circumference of a
4. The area under a speed-time graph represents a big sports stadium with constant speed. Which of
physical quantity which has the unit of: the following do you think is changing in this case?

(a) m (b) m2 (a) Magnitude of acceleration being produced


(c) m s–1 (d) m s–2 (b) Distance covered by the sprinter per second
(c) Direction in which the sprinter is running
5. If the displacement of an object is proportional to (d) Centripetal force action on the sprinter
the square of time, then the object is moving with:
10. In the speed-time graph for a moving object shown
(a) Uniform velocity here, the part which indicates uniform deceleration
(b) Uniform acceleration of the object is: R
B
(c) Increasing acceleration
S
(d) Decreasing acceleration (a) ST T

(b) QR
P Q
Speed

6. Four cars A, B, C and D are moving on a levelled, (c) RS


straight road. Their distance-time graphs are shown (d) PQ O A
Time
in the given figure. Which of the following is the
correct statement regarding the motion of these 11. A student draw a distance-time graph for a moving
cars? scooter and find that a section of the graph is a
horizontal line parallel to the time axis. Which
C of the following conclusion is correct about this
D section of the graph?
Distance (m)

A (a) The scooter has uniform speed in this section


B (b) The distance travelled by scooter is the
maximum in this section
(c) The distance travelled by the scooter is the
Time (s) minimum in this section
(d) The distance travelled by the scooter is zero in
(a) Car A is faster than car D this section
(b) Car B is the slowest
(c) Car D is faster than the car C 12. Which of the following is most likely not a case of
(d) Car C is the slowest uniform circular motion?
(a) Motion of the earth around the sun
136 CLASS - IX
MOTION IN A STRAIGHT LINE

(b) Motion of a toy train on a circular track 18. If a body starts from rest and moves with uniform
(c) Motion of a racing car on a circular track acceleration, then :
(d) Motion of hours’ hand on the dial of a clock
(a) v ∝ t (b) s ∝ t
13. The ratio of magnitude of displacement to distance (c) v ∝ s (d) s ∝ t
is always ...................
19. A bus travels the first one-third distance at a speed
(a) Less than 1 (b) Greater than 1 of 10 km h–1, the next one-third distance at a speed
(c) Equal to 1 (d) Less than or equal to 1 of 20 kmh–1 and the next one-third distance at a
speed of 30 kmh–1. The average speed of the bus is.
14. The variation of the velocity of a particle moving 50 -1
along a straight line is illustrated in the graph (a) 20 m s–1 (b) ms
11
given below. The distance covered by the particle 180 -1
(c) ms (d) 30 m s–1
in 4 seconds is ......................... m. 11
(a) 20
20 20. The velocity of a body is given by the equation
(b) 35 10 v = 6 - 0.02 t, where t is the time taken. The body
v
(ms-1) is undergoing .....................
(c) 40
0 1 2 3 4
(s)
t (a) Uniform retardation
(d) 55
(b) Uniform acceleration
15. An ant moves from one corner of a hall to the (c) Non-uniform acceleration
diagonally opposite corner. If the dimensions of (d) Zero-acceleration
the floor of hall are 8 m × 6 m, the displacement
of the ant is ...................... m. 21. A body starts from rest and moves with uniform
acceleration for 2s. It then decelerates uniformly
(a) 14 (b) 10 for 3s and stops. If deceleration is 4 m s–2, the
(c) 28 (d) 2 acceleration of the body is ..................... m s–2.
16. The figure given below shows the displacement- (a) 10 (b) 8.7
time graph of the two particles P and Q. Which of (c) 4 (d) 6
the following statements is correct?
22. Density is a ................. quantity.
P (a) Scalar
Q (b) Derived
s
(c) Neither (a) nor (b)
(d) Both (a) & (b)
O
t 23. A particle revolves along a circle with a uniform
speed. The motion of the particle is ...................
(a) Both P and Q move with uniform equal speed
(a) One dimensional (b) Two dimensiona
(b) P is accelerated and Q is retarded. (c) Translatory (d) Oscillatory
(c) Both P and Q move with uniform speed, but the
24. Which one of the following graph indicates that
speed of P is more than the speed of Q.
the body is at rest?
(d) Both P and Q move with uniform speeds but the
Y Y
speed of Q is more than the speed of P.
S V
17. 
When brakes are applied, the velocity of a car t X t X
(a) O (b) O
changes from 40 ms–1 to 10 ms–1 in 10 s. The
acceleration produced in its is .................... ms–2. Y Y
v s
(a) –3 (b) 3 X X
(c) O t (d) O t
(c)–5 (d) 5

PHYSICS 137
25. 
A particle moves along a circular track of 6 m 3. Time is a vector quantity.
radius such that the arc of the circular track covered
4. 
If the velocity of the body decreases with time,
subtends an angle of 30o at the centre. The distance
the acceleration is negative (retardation), and the
covered by the body is ........................
motion is called decelerated motion.
II. Match the entries in Column A with the 5. 
The area of the velocity time graph gives
appropriate ones in Column B. displacement of the body.
Column A Column B 6. Acceleration is a scalar quantity.
7. Motion and rest are relative terms.
(A) Uniform velocity (a) s 8. An object can be moving with Uniform speed but
variable acceleration.
t
IV. Give one line answers
Uniform acceleration with s 1. What is a reference point?
(B) (b)
initial velocity
t 2. 
An object has moved through a distance. Can
it have zero displacement? If yes, support you
answerwith an example.
(C) Uniform acceleration (c) s
3. Are all physical quantities that have magnitude and
t direction vectors? Give example to support your
answer.

(D) Increasing acceleration (d) v 4. Under what condition is the magnitude of average
velocity of an object equal to its average speed?
t
5. State which of the following situations are possible
and give an example for each of these;
(E) Uniform retardation (e) s
(a) An object with a constant acceleration but with
t zero velocity.

(b) An object moving in a certain direction with


Decreasing acceleration at a acceleration in the perpendicular direction.
(F) (f)
steady rate
t 6. A boy hits a football high up into the air. He runs
and catches the football before it hits the ground.
Which of the two, the boy or the football has had
Uniform acceleration with a greater displacement?
(G) (g)
initial displacement
t 7. A car manufacturer advertises that the brakesare so
perfect that the car stops instantaneous.Comment
Body at rest with initial a 8. How do you think a graph of decelerationwould
(H) (h)
displacement differ from the graph of acceleration? Explainyour
t reasoning

III. State True or False: 9. Is acceleration possible in uniform motion? justify
your answer
1. Displacement can be zero but distance never.
2. 
Displacement magnitude can be greater than 10. What would be motion in two dimension? Discuss.
distance travelled by the object.
138 CLASS - IX
MOTION IN A STRAIGHT LINE

Space For Rough Work

PHYSICS 139
Scope
Chemistry is one of the basic sciences. Knowledge of
chemistry is indispensable for the pursuit of any of the other
sciences. We need it for the study of Technology, Pharmacology,
Mineralogy, and what not. It is indispensable for the study of
physics, geology, biology and physiology.
Chapter 1 Learning Checklist
Matter and its classification
Kinetic theory of matter
Properties of gases
MATTER Gases- Ideal gas & Real gas
Properties of Liquids
AND ITS SURROUNDINGS Properties of Solids
Phase changes

What is matter?
Chemists study the structures, physical properties, and chemical properties of
material substances. These consist of matter, which is anything that occupies
space and has mass.The mass of an object is the quantity of matter it contains.
This is the earliest known
Gold and iridium are matter, as are peanuts, people, and postage stamps. Smoke,
steam engine, invented about A.D.
smog, and laughing gas are matter. Energy, light, and sound, however, are not
60 by Hero, a scientist who lived in
matter; ideas and emotions are also not matter.
Alexandria, Egypt. This model also
Classification of matter demonstrates the four most familiar
states of matter: solid, liquid, gas,
Since matter exists in countless form, its study can be simplified by dividing it and plasma. This steam engine spins
into two different ways as steam escapes through the nozzles.
Physical Classification: Depending upon the rigidity, volume and shape i.e.
physical, nature matter can be classified as solids, liquids, and gases.
Chemical Classification: Depending upon the chemical composition of the
substances, the matter may be classified into elements, compounds, and mixtures.
Solid
THE UNIVERSE
Gas

Matter Energy

Physical Classifications Chemical Classifications


Liquid
Plasma
Solids Liquids Gases

Elements Compounds Mixtures

Metals Non - Metals Metalloids Homogeneous Heterogeneous

Inorganic Compounds Organnic Compounds

Note: This chapter deals only with physical classification of matter.

Another state of matter, plasma, occurs naturally in the interiors of stars.


A plasma is a gaseous state of matter that contains appreciable numbers of
electrically charged particles. The presence of these charged particles imparts
unique properties to plasmas that justify their classification as a state of matter
distinct from gases. In addition to stars, plasmas are found in some other high-
temperature environments (both natural and man-made), such as lightning
strikes, certain television screens, and specialized analytical instruments used to
detect trace amounts of metals. A plasma torch can be used to cut metal

142 CLASS - IX
MATTER AND ITS SURROUNDINGS

Properties of matter
Physical properties of matter
The physical properties are those which define the matter without changing
An aurora (plural: auroras composition makeup. It does not include any chemical change. The overall
or aurorae), sometimes referred to as composition of the material remains the same. Examples of physical properties
polar lights, northern lights (aurora include color, texture, flexibility,density, mass etc. We can observe or measure the
borealis), southern lights (aurora properties with its two types. The two types of physical properties
australis), is a natural light display in Intensive Properties
the Earth's sky, predominantly seen in
the high-latitude regions (around the The properties which do not depend upon either the size of the system or
Arctic and Antarctic).Auroras, like the the quantity of matter present in it are known as intensive properties. For example
aurora borealis seen here, form when Pressure, temperature, density, specific heat, surface tension, viscosity, refractive
high-energy plasma collides with gas index, melting and boiling points etc.
particles in the upper atmosphere. Extensive properties
The properties which depend upon the quantity of the matter present in the
system. For example Mass, volume, energy, enthalpy, work etc.

Chemical properties of matter


It is the ability of the matter to react with surroundings or other substances.
During a chemical process matter composition changes. Chemical properties
include a change in color, texture,density, mass etc. For example: chemical
properties of burning a piece of paper define that how it reacts with the air.
Factors that affect the chemical properties are: Temperature, Pressure,Volume,
The amount of substance (moles).

Measuring matter
The mass of an object is a measure of the amount of matter in it. One way to
measure an object’s mass is to measure the force it takes to accelerate the object. It
takes much more force to accelerate a car than a bicycle because the car has much
more mass. A more common way to determine the mass of an object is to use a
balance to compare its mass with a standard mass.
Brain Power On
.......................................... Although weight is related to mass, it is not the same thing. Weight refers to
the force that gravity exerts on an object. This force is directly proportional to
1. If all of your school books the mass of the object. The weight of an object changes as the force of
combined have a mass of 3 kg, what gravity changes, but its mass does not. An astronaut’s mass does not change
is their total weight in newtons? just because she goes to the moon. But her weight on the moon is only one-
sixth her earth-bound weight because the moon’s gravity is only one-sixth that
2. You may sometimes hear on the of the earth’s. She may feel “weightless” during her trip when she experiences
radio or on television that astronauts negligible external forces (gravitational or any other), although she is, of course,
are “weightless” in space. Explain never “massless”.
why this is not true.
Composition of matter
3. A jar contains 30 mL of glycerin The smallest particle of matter that is capable of free existence retaining all the
(mass =37.8 g) and 60 mL of corn characteristics of it is called molecule and molecules are made up of atoms. The
syrup (mass = 82.8 g). Which liquid molecules are constituent particles of matter and characteristics of the molecules
is on top? Explain your answer. and their arrangement in various states of matter is explained by kinetic theory of
matter.

CHEMISTRY 143
Class Room
The kinetic theory of matter
Activity-01
1. All matter is made up of very tiny particles called molecules. Which is heavier hot air or
2. In the solid state, the particles are rigidly held in positions about which cold?
they can only vibrate. Drill holes 6 inches from each
3. In the liquid state, the particles are in continuous motion, but are not end of a narrow 3-foot length of
completely separated from each other. wood, such as a yardstick. Then,
4. (a) In the gaseous state, the particles are in continuous random motion, make a hole in the exact center of
almost independent of each other. the stick, 18 inches from each end.
(b) The particles in a gas travel much longer distances than in a liquid Balance an “empty” baby bottle
before they collide with other particles or with the walls of the vessel. on one end of your yardstick and
a tin can on the other as shown in
(c) The collision of the particles with the walls of the vessel gives rise to
the figure. Put sand in the can if
the pressure of the gas.
needed.
5. The particles possess some energy as a result of their motion. This energy
Hold a candle flame for one
is called kinetic energy (KE).
minute near the mouth of the
6. T
 he KE possessed by the particles is dependent on temperature. The bottle. Remove the flame and
higher the temperature, the higher is the KE; the lower the temperature, balance the scale again.
the lower is the KE.
Observation:The bottle goes up
Properties of matter — Gases when heat is applied to the air in
it. You must remove sand from the
Particle size can on the other end to balance
Gases consist of small particles that are separated from one another by empty the scale.
space. The volume of the particles is small compared with the volume of the Explanation: Warm air weighs
empty space. Because gas particles are far apart, they experience no significant less than cold air occupying the
attractive or repulsive forces. same space.

Particle motion Discuss: Why hot-air balloons


float gracefully above the earth,
Gas particles are in constant, random motion. Particles move in a straight While balloons you blow up fall to
line until they collide with other particles or with the walls of their container. the ground?
Collisions between gas particles are elastic. An elastic collision is one in which 3

no kinetic energy is lost. Kinetic energy can be transferred between colliding


particles, but the total kinetic energy of the two particles does not change. 2

Particle energy 1

Two factors determine the kinetic energy of a particle: mass and velocity.
The kinetic energy of a particle can be represented by the following equation:

KE = _
1
mv2
2
KE is kinetic energy, m is the mass of the particle, and v is its velocity.Velocity
reflects both the speed and the direction of motion. In a sample of a single gas,
all particles have the same mass, but all particles do not have the same velocity.
Therefore, all particles do not have the same kinetic energy. Temperature is a
measure of the average kinetic energy of the particles in a sample of
matter.

Compression and expansion


Kinetic energy can be transferred
If you squeeze a pillow made of foam, you can compress it; that is, you can reduce
between gas particles during an elastic
its volume. The foam contains air pockets. The large amount of empty space collision.
144 CLASS - IX
MATTER AND ITS SURROUNDINGS

Class Room
Activity-02 between the particles in the air in those pockets allows the air to be pushed easily
into a smaller volume. When you stop squeezing, the random motion of the
Diffusion of gases particles fills the available space, and the pillow expands to its original shape.
Figure below illustrates what happens to the density of a gas in a container as it
A long glass tube is filled at one
is compressed and as it is allowed to expand.
end with a plug of cotton wool
soaked in conc. hydrochloric acid
sealed in with a rubber bung (for
health and safety).
A similar plug of conc. ammonia
solution is placed at the other end. Compression Expansion
The soaked cotton wool plugs
will give off fumes of HCl and NH3
respectively, and if the tube is left
In a closed container, compression and expansion change the volume occupied by
undisturbed and horizontal, despite
a constant mass of particles.
the lack of tube movement, a white
cloud forms about 1/3rd along from
Diffusion and effusion
the conc. hydrochloric acid tube end.
According to the kinetic-molecular theory, there are no significant forces of
attraction between gas particles.Thus, gas particles can flow easily past each other.
Often, the space into which a gas flows is already occupied by another gas. The
random motion of the gas particles causes the gases to mix until they are evenly
distributed.
Diffusion is the gradual mixing of gases due to the motion of their component
particles even in the absence of mechanical agitation such as stirring. The result
Explanation: The colourless gases, is a gas mixture with uniform composition. The term might be new, but you are
ammonia and hydrogen chloride, probably familiar with the process. If food is cooking in the kitchen, you can
diffuse down the tube and react to smell it throughout the house because the gas particles diffuse. Particles diffuse
form fine white crystals of the salt from an area of high concentration (the kitchen) to one of low concentration
ammonium chloride. (the other rooms in the house).
NH3(g) + HCl(g) → NH4Cl(s) Effusion is a process related to diffusion. During effusion, a gas escapes
through a tiny opening. What happens when you puncture a container, such as
The smaller the molecular mass,
a balloon or a tire? In 1846, Thomas Graham conducted experiments to measure
the greater the average speed of the
the rates of effusion for different gases at the same temperature. Graham designed
molecules
his experiments so that the gases effused into a vacuum — space containing no
Therefore the smaller the matter. He discovered an inverse relationship between effusion or diffusion rates
molecular mass, the faster the gas and molar mass. Graham’s law of effusion or diffusion states that, the rate of
diffuses. diffusion or effusion for a gas is inversely proportional to the square
Molecular mass of NH3 = (14 x1) root of its molar mass.
+ (1 x 3) = 17 u, moves faster than Graham’s law of effusion or diffusion
Molecular mass of HCl = (1 x 1) + 1
(35.5 x 1) = 36.5 u And that's why Rate of effusion or diffusion a
molar mass
they meet nearer the HCl end of the
tube.
Gas Pressure
Discuss: Of all the gases, hydrogen
To understand gas pressure, picture a typical gas in a closed container. Each
diffuses fastest. What can you tell
time a gas particle collides with and ricochets off one of the walls of its container,
from that?
it exerts a force against the wall.The sum of the forces of these ongoing collisions

CHEMISTRY 145
of gas particles against all the container’s interior walls creates a continuous a
pressure upon those walls. Pressure is force divided by area.

Force due to particle collisions with the walls


Gas pressure =
Area of the walls
Force
Pressure =
Area

The accepted SI unit for gas pressure is the pascal, Pa. Other units used to
b
describe gas pressure are the atmosphere (atm), torr, millimeter of mercury (mm
Hg), and bar. The relationships between these pressure units are tabulated :
Comparison of Pressure Units
Unit Number Equivalent Number Equivalent
to 1 atm to 1 kPa
Kilopascal (kPa) 101.3 kPa —
Atmosphere (atm) — 0.009869 atm a. Diffusion b. Effusion
Millimeters of mercury
760 mm Hg 7.501 mm Hg
(mm Hg)
Torr 760 torr 7.501torr
Pounds per square inch
14.7 psi 0.145 psi
(psi or lb / in2)
Bar 1.01 bar 100 kPa

Example 01:
(a) The pressure of a gas contained in a cylinder with a movable piston is 300 Pa.
The area of the piston is 0.5 meter square. Calculate the force that is exerted on
the piston.
Solution:
Pressure = Force ÷ Area
F = Pressure × Area
F = (300 Pa)(0.5 m2)
F = 150 N

(b) The Gas molecules are moving with the force of 200 N in the area of 20
meter square. Calculate its gas pressure.
Pressure = Force ÷ Area
P = 200 N ÷ 20 m2
P = 10 pascals
These molecules
contribute to the
Air or Atmospheric Pressure These molecules
pressure (weight)
at this altitude
contribute to the
One important type of pressure is the pressure exerted on objects from the air pressure (weight)
or the Earth's atmosphere. This is actually the measurement of the weight of at this altitude

the gas above an object on a given surface area. The higher the elevation,
the lower the atmospheric pressure exerted because there is less air pressing
down on the object. One example of this is the boiling point of water. At higher
elevations where the air pressure is lower, water will boil at a lower temperature. Surface air pressure = weight of air in column above unit area

146 CLASS - IX
MATTER AND ITS SURROUNDINGS

Gas laws
Created in the early 17th century, the gas laws have been around to assist in
B o s e - E i n s t e i n finding volumes, amount, pressures and temperature when coming to matters of
Condensate (B.E.C.) is another state gas.The gas laws consist of three primary laws: Charles' Law, Boyle's Law and
of matter which is formed from matter Avogadro's Law (all of which combines into the General Gas Equation or Ideal
that has been cooled to near absolute
Gas Law).
zero (–273ºC). When a group of atoms
is cooled to a very low temperature,
the velocity decreases because they Ideal Gases
have very low energies. This causes the Ideal gas, or perfect gas, is the theoretical substance that helps establish the
individual atoms to overlap each other
relationship of four gas variables, pressure (P), volume(V), the amount of gas(n)
forming a single super atom with all of
its constituting atoms sharing a single and temperature(T). It has characters described as follow:
energy state. 1. The particles in the gas are extremely small, so the gas does not occupy any
spaces.
2. The ideal gas has constant, random and straight-line motion.
3. 
No forces between the particles of the gas. Particles only collide elastically
with each other and with the walls of container.

Visualization of vortices in rotating B.E.C Real Gases


A rotating B.E.C. could be used as Real gas, in contrast, has real volume and the collision of the particles is
model black hole, allowing light to enter not elastic, because there are attractive forces between particles. As a result, the
but not to escape. Condensate can also be volume of real gas is much larger than of the ideal gas, and the pressure of real gas
used to 'free' pulses of light, to be released
is lower than of ideal gas. All real gases tend to perform ideal gas behavior
again when condensate break down.
Research in this field is going on. at low pressure and relatively high temperature.
The compressibility factor (Z) tells us how much the real gases differ from
2.0 ideal gas behavior.
N2
CH4 H2
PV
1.5 CO2
Z=
nRT
Ideal gas
Z 1.0
For ideal gases, Z = 1. For real gases, Z ≠ 1
0.5
Thus in case of real gases Z can be < 1 or > 1.
When Z < 1, it is a negative deviation. It shows that the gas is more
0 200 400 600 800 1000
P(atm) compressible than expected from ideal behaviour. Example: CO2 , CH4.
When Z > 1, it is a positive deviation. It shows that the gas is less
compressible than expected from ideal behaviour. Example: H2

Vol. Vol. Boyle's Law


6 6
5 5 In 1662, Robert Boyle discovered the correlation between Pressure (P)and
4 4
3 3
Volume (V) (assuming Temperature(T) and Amount of Gas(n) remain constant):
2 2 1
1 1 P∝ → PV = x
V

Where x is a constant depending on amount of gas at a given temperature.


1.00 300 1.33 300
Pressure Temperature Pressure Temperature Pressure is inversely proportional to volume.

CHEMISTRY 147
Therefore, as seen, as

Pressure
volume increases,
pressure decreases

P1
Weather balloons demonstrate
P2 a practical use of Boyle’s law. A weather
balloon carries instrument into the
V1 V2
Volume atmosphere to collect information used
Another form of the equation (assuming there are 2 sets of conditions, and to predict the weather. This balloon is
setting both constants to each other) that might help solve problems is: filled with only a small amount of gas
because the pressure of the gas decreases
P1V1 = x = P2V2 and the volume increases as the balloon
Example 02: rises. If the balloon were filled with too
much gas, it would pop as the volume of
A 17.50 mL sample of gas is at 4.500 atm.What will be the volume if the pressure
the gas is increased. A small parachute
becomes 1.500 atm, with a fixed amount of gas and temperature?
attached with the balloon returns the
Solution: Initial Case Final Case instrument to the earth.

4.500 atm at the same 1.500 atm


17.50 mL? temperature Vnew?

P1 .V1
V2 =
P2
4.500 atm.17.50 ml
=
1.500 atm
= 52.50 ml

Charles' Law
In 1787, French physicists Jacques Charles, discovered the correlation between
Temperature(T) and Volume(V) (assuming Pressure (P) and Amount of Gas(n)
remain constant):
V ∝ T → V = yT
where y is a constant depending on amount of gas and pressure. Volume is
directly proportional to Temperature.
(Volume)

V
Volume (V1) Volume (V2)
TK (Temperature
in kelvin)
Another form of the equation (assuming there are 2 sets of conditions, and
setting both constants to eachother) that might help solve problems is:
Temperature Temperature
V1 V (T1) (T2)
=y= 2
T1 T2

148 CLASS - IX
MATTER AND ITS SURROUNDINGS

Kelvin (K)  Kelvin (K)  Absolute scale of temperature or absolute zero


Celsius(oC) Fahrenheit(oF) If the graph between V and T is extra plotted, it intersects X-axis at -273.16o C
5 At -273.16o C volume of any gas theoretically becomes zero as indicated by the
TK = TC - 273.16 TK= (TF -459.67)
9 graph.
9 But practically volume of a gas can never become zero. Actually no gas
TC = TK-273.16 TF = (TK-459.67)
5 can achieve the lowest possible temperature and before -273.16o C all gases
are condensed to liquid. This temperature is referred to as absolute scale or
absolute zero. At -273.16o C all molecular motions are ceased.
(Volume)
V Example 03:
A sample of Carbon dioxide in a pump has volume of 20.5 mL and it is at 40.0 oC.
When the amount of gas and pressure remain constant, find the new volume of
Carbon dioxide in the pump if temperature is increased to 65.0o C.
-273.16oC TC Final Case
(Temperature Initial Case
in celcius)

40.0 + 273.15 K 60.0 + 273.15 K


Brain Power On 20. 5 mL Vnew = ?
..........................................
Each graph below illustrates a gas Solution:
V1. V2
law. However, the variable on one V2 =
T1
axis of each graph is not labeled.
20.5 mL.(60 + 273.15 K)
Answer the following questions =
40 + 273.15 K
for each graph: 1. As the volume
= 22.1 mL
increases, what happens to the
missing variable? 2. Which gas law Avogadro's Law
is shown? 3. What label belongs on
In 1811, Amedeo Avogadro discovered the correlation between the Amount
the axis? 4. Is the graph linear or
of gas(n) and Volume(V) (assuming Temperature(T) and Pressure(P) remain
nonlinear? What does this tell you?
constant):
Graph A V ∝ n → V = zn
m
where z is a constant depending on Pressure and Temperature. n =
M
Volume

Volume(V) is directly proportional to the Amount of gas(n)

+ =
?
2 mol H2 1 mol O2 2 mol H2O
Graph B
m = Give mass (or) required mass (g)
n = number of moles (mol)
Volume

M = Molar Mass (g/mol)


Another form of the equation (assuming there are 2 sets of conditions, and
setting both constants to each other) that might help solve problem is
P1 P
? =z= 2
n1 n2

CHEMISTRY 149
Example 04:
A 3.80 g of oxygen gas in a pump has volume of 150 mL. constant temperature
and pressure. If 1.20 g of oxygen gas is added into the pump.What will be the new
volume of oxygen gas in the pump if temperature and pressure held constant?
A balloon filled with helium
Solution:
weighs much less than an identical
Final Case balloon filled with air. Both balloons
Initial Case contain the same number of molecules.
Helium atoms have lower mass than
either oxygen molecules or nitrogen
molecules in air, so the helium balloon is
lighter.
3.80 g of oxygen gas 5.00 g of oxygen gas
150 mL Vnew = ?
At the same
temperature

V1 = 150 mL
m1 3.80 g
n1 = = = 0.118 moles
MO 32 g/mole
2
m2 5.00 g
n2 = = = 0.156 moles
MO 32 g/mole
2

V1 . n 2 150 ml × 0.156 moles


V2 = =
n1 0.118 moles Brain Power On
..........................................
V2 = 198.3 ml `
To extinguish a fire, you need to
Ideal Gas Law take away fuel, oxygen, or heat. The
The ideal gas law is the combination of the three simple gas laws. By setting fire extinguisher contains carbon
all three laws directly or inversely proportional to Volume, you get: dioxide, which displaces oxygen but
does not burn. It also has a cooling
V ∝ nT effect due to the rapid expansion of
P the carbon dioxide as it is released
Next replacing the directly proportional to sign with a constant(R) you get: from the nozzle.
RnT Explain Why does carbon
V=
P dioxide displace oxygen?

PV = nRT From ideal gas equation derive


and finally get the equation:
an expression for the density of
Where P= the absolute pressure of ideal gas the gas.
• V= the volume of ideal gas (or) Volume of container
• n = moles of gas
• T = The absolute temperature
• R = The gas constant
The value of R is determined by experimental results. Its numerical value
changes with units.
R = 8.3145 Joules • mol-1 • K-1 (SI Unit)
= 0.082057 L • atm•K-1 • mol-1

150 CLASS - IX
MATTER AND ITS SURROUNDINGS

Note Example 05:


.......................................... At 655 mm Hg and 20.0oC, a sample of Chlorine gas has volume of 750 mL.
Standard Temperature and How many moles of Chlorine gas at this condition?
Pressure (STP) • P = 655 mm Hg
• T = 25 + 273.15 K
The conditions of 1 atm and
273.15 K (0°C) are defined as being • V = 750 mL = 0.75 L
standard temperature and pressure n=?
(STP).These conditions are generally Solution: From ideal gas equation we know that,
used when reporting measurements PV
n=
on gases. Keep in mind that the RT
standard temperature for gases (0°C) 1atm
655 mmHg . . 0.75 L
is different from the standard state 760 mmHg
=
or thermodynamic measurements 0. 082057 L.atm.mol−1.K −1 .(25 + 273.15 K)
(25°C). At 1 atm and 0°C, 1 mole
= 0.026 mol
of any ideal gas occupies a volume
of 22.414 L (the standard molar Properties of matter — Liquids
volume): Although the kinetic-molecular theory was developed to explain the behavior
nRT
V=
P
of gases, the model also applies to liquids and solids. When applying the kinetic-
(1.00 mol)(0.08206 L atm K-1mol-1)(273.15K) molecular theory to the solid and liquid states of matter, you must consider the
1.00 atm forces of attraction between particles as well as their energy of motion. Liquid can
= 22.4 L take the shape of its container but its volume is fixed. In other words, the particles
can flow to adjust to the shape of a container, but the liquid cannot expand to fill
its container, as shown in Figure.
According to the kinetic-molecular theory, individual particles do not have
fixed positions in the liquid. Forces of attraction between particles in the liquid
limit their range of motion so that the particles remain closely packed in a fixed
volume.

Density and compression


The density of a liquid equals the mass of the liquid divided by its volume.
Mass
Density =
Volume
The density of water is 1 gram per cubic centimeter. At 25 °C and 1 atm of
air pressure, liquids are much denser than gases. The density of a liquid is much
greater than that of its vapor at the same conditions. For example, liquid water
is about 1250 times denser than water vapor at 25 °C and 1 atm of pressure.
Because they are at the same temperature, both gas and liquid particles have the
same average kinetic energy. Thus, the higher density of liquids is due to the
intermolecular forces that hold particles together.

Liquid Pressure
It is also important to figure out the pressure under water or in a liquid.
The pressure under water increases with how deep you are. The equation for
calculating pressure under a liquid is:
Liquids flow and take the shape of Pressure = D * g * h
their container, but they do not expand where D is the fluid's density, g is a standard gravity (9.8 m/s2), and h is the
to fill their container like gases. depth of the object in the liquid.
CHEMISTRY 151
Example 06:
What is the difference between the hydrostatic pressure of blood between the
brain and the soles of the feet of a person whose height 165 cm (suppose the
density of blood = 1.0 × 103 kg/m3, acceleration due to gravity = 10 m/s2)
Given:
Height (h) = 165 cm = 165/100 m = 1.65 meters
Density of bloods (D) = 1.0 × 103 kg/m3
Acceleration due to gravity (g) = 10 m/s2
Solution :
P=Dgh
= (1.0 × 103)(10)(1.65)
= (1.0 × 104)(1.65)
P = 1.65 x 104 N/m2

Fluidity
Gases and liquids are classified as fluids because they can flow and diffuse. One Gases and liquids have the ability to
liquid diffusing through another liquid. Liquids usually diffuse more slowly than flow and diffuse.These figures show
gases at the same temperature, because intermolecular attractions interfere with one liquid diffusing through another
the flow. Thus, liquids are less fluid than gases. liquid.

Class RoomActivity-03
Build your own Density Tower
Materials required:
I. Tall, narrow, clear container (500 mL or 1000 mL graduated cylinders)
II. 50 mL of each : lamp oil , rubbing alcohol ,vegetable oil ,tap water ,dish soap ,milk , maple syrup ,corn syrup ,
and honey
Experiment: Start your column by pouring the honey into the cylinder. Now, you will pour each liquid SLOWLY
into the container, one at a time. It is very important to pour the liquids slowly and into the centre of the cylinder. Make
sure that the liquids do not touch the sides of the cylinder while you are pouring.The layers will always even themselves
out because of the varying densities. Make sure you pour the liquids in the following order:
Honey Corn syrup, Maple syrup, Milk, Dish soap, Water,Vegetable oil,
Rubbing alcohol, Lamp oil,
Observation: After letting the liquid layers settle, you'll notice that Ping Pong Ball
they remain in the order you poured them into the cylinder and that Lamp oil
they are clearly distinguishable from each other. Take the various small Rubbing Alcohol Soda Cup
objects and drop them into the column. Drop them in the following Vegetable oil Beads
order: Metal nut or bolt, Popcorn kernel, Board game die, Plastic bead,
water
Soda cap, Ping pong ball. Each of the objects will sink through or float Cherry Tomato
on a different layer of the density column. Dish Soap
Die
Milk
Discuss:
100% Maple Syrup Popcorn kernel
I.  What scientific principle do you think contributes to the
column's layers? Corn Syrup

II. What makes some objects sink deeper into the column while Honey
Bolt
some hardly sink at all?

152 CLASS - IX
MATTER AND ITS SURROUNDINGS

Viscosity
You are already familiar with viscosity if you have ever tried to get honey out of a
bottle.Viscosity is a measure of the resistance of a liquid to flow.The particles in a
liquid are close enough for attractive forces to slow their movement as they flow past
one another.The viscosity of a liquid is determined by the type of intermolecular
forces in the liquid, the size and shape of the particles, and the temperature.

Surface tension
Intermolecular forces do not have an equal effect on all particles in a liquid.
Particles in the middle of the liquid can be attracted to particles above them,
below them, and to either side. For particles at the surface of the liquid, there are
no attractions from above to balance the attractions from below. Thus, there is a
net attractive force pulling down on particles at the surface.The surface tends
Side view
to have the smallest possible area and to act as though it is stretched tight like the
head of a drum. For the surface area to increase, particles from the interior must
At the surface of water, the particles move to the surface. It takes energy to overcome the attractions holding these
are drawn toward the interior until particles in the interior.The energy required to increase the surface area of
attractive and repulsive forces are a liquid by a given amount is called surface tension. Surface tension is a
balanced. measure of the inward pull by particles in the interior.

Cohesion and adhesion


When water is placed into a narrow container, such as the glass tubes as shown
in figure. You can see that the surface of the water is not straight. The surface
forms a concave meniscus; that is, the surface dips in the center. Figure models
what is happening to the water at the molecular level. There are two types of
forces at work: cohesion and adhesion.
Cohesion describes the force of attraction between identical molecules.
Adhesion describes the force of attraction between molecules that are different.
Because the adhesive forces between water molecules and the silicon dioxide in
glass are greater than the cohesive forces between water molecules, the water rises
along the inner walls of the cylinder.

Brain Power On
..........................................
Why Ice floats in a Glass of water ?
Why water freezes from top to
bottom ? Cohesion Adhesion
Water molecules have cohesive and adhesive properties.

Properties of matter — Solids


For a substance to be a solid rather than a liquid at a given temperature, there
must be strong attractive forces acting between particles in the solid. These
forces limit the motion of the particles to vibrations around fixed locations in the
solid. Thus, there is more order in a solid than in a liquid. Because of this order,
solids are not fluid. Only gases and liquids are classified as fluids.
CHEMISTRY 153
Density
In general, the particles in a solid are more closely packed than those in a
liquid. Thus, most solids are more dense than most liquids. When the liquid
and solid states of a substance coexist,the solid almost always sinks in the liquid.
Solid cubes of benzene sink in liquid benzene because solid benzene is more
dense than liquid benzene.
There is about a 10% difference in density between the solid and liquid states
of most substances. Because the particles in a solid are closely packed, ordinary
amounts of pressure will not change the volume of a solid.

Types of solids Crystalline


A crystalline solid is a solid whose atoms, ions, or molecules are arranged in
an orderly, geometric structure.The locations of particles in a crystalline solid can
be represented as points on a framework called a crystal lattice.
An amorphous solid is one in which the particles are not arranged in a
regular, repeating pattern. The term amorphous is derived from a Greek word
that means without shape. Glass, rubber, and many plastics are amorphous solids.

Isotropy and Anisotropy


Amorphous substances differ from crystalline solids and resemble Amorphous
liquids in another important respect.The properties such as electrical conductivity,
thermal conductivity, mechanical strength, and refractive index are the same in all
directions. Amorphous substances are, therefore, said to be isotropic. Liquids and
gases are also isotropic. Crystalline solids, on the other hand, are anisotropic, i.e.,
their physical properties are different in different directions.
For example, the velocity of light passing through a crystal varies with the direction
in which it is measured.Thus, a ray of light entering such a crystal may split up into two
components each following a different path and traveling with a different velocity.
This phenomenon is known as double refraction.
Thus, anisotropy in itself is a strong evidence for the existence of ordered
molecular arrangements in such materials. This can be shown on reference to
B
figure in which a simple two-dimensional arrangement of only two different D
kinds of atoms is depicted.
If the properties are measured along the direction indicated by the slanting
line CD, they will be different from those measured in the direction indicated by
the vertical line AB.The reason is that while in the first case, each row is made up
of alternate type of atoms, in the second case, each row is made up of one type
of atoms only.
C A
In amorphous solids as well as in liquids and gases, atoms or molecules are
arranged at random and in a disorderly manner and, therefore, all directions are Isotropy and Anisotropy
identical and all properties are alike in all directions.

154 CLASS - IX
MATTER AND ITS SURROUNDINGS

Comparison of solids, liquids and gases

Property Solid Liquid Gas

Arrangement of
particles
Particles are closely packed and Particles are loosely packed and Particles are very far apart and
have an orderly arrangement there is no orderly arrangement are randomly arranged.

Forces of
attraction between
Very strong force of attraction. Strong forces of attraction. Weak forces of attraction
constituent
particles.

Particles are in random motion


Particles can vibrate about a Particles can roll, slide past
Motion of particles and are moving about at great
fixed point. each other.
speed.

Possess moderate kinetic


Kinetic Energy Possess low kinetic energy. Possess high kinetic energy.
energy.

Mass They have definite mass. They have definite mass. They have definite mass

They do not have a definite They do not have a definite


Shape They have a definite shape. shape i.e. have the shape of shape i.e. have the shape of
container. container.

They do not have a definite


volume and fill the container
Volume They have definite volume. They have definite volume.
completely i.e., have the
volume of container.
Intermolecular spaces larger
In solids the intermolecular In gases the intermolecular
than solids so the density of
Density space is minimum so they have spaces are maximum so gases
liquids is less as compared to
maximum density. have least density.
solids.

Highly rigid. Particles in solids Less rigid in comparison to


Not rigid. In gases particles are
Rigidity can vibrate only about their solids. In liquids free movement
free to move in all Directions
mean position of particles is possible.

Diffusion (or
No diffusion Slightly diffusible Highly diffusible
miscibility)

CHEMISTRY 155
Phase changes
Gas
Most substances can exist in three states depending on the temperature and
pressure. A few substances, such as water, exist in all three states under ordinary on
Co
nd
ati en
conditions. States of a substance are referred to as phases when they coexist as bli
m Va
po
sat
ion
Su tion riz
physically distinct parts of a mixture. Ice-water is a heterogeneous mixture osi ati
D ep on
Melting
with two phases, solid ice and liquid water.When energy is added or removed
from a system, one phase can change into another. Freezing
Solid Liquid
Gaining heat in three phases of matter increase average kinetic energies of
particles. For same matter kinetic energy of particles in gas phase is larger than the
The six possible transitions between phases.
kinetic energies in solid and liquid states. Definitions of some concepts related to
phase change are given below.
Note
Melting: Solid matter changes its state to liquid. ..........................................
Freezing: Opposite process of melting is called freezing. Liquid matter loses The red arrows in the figure indicates
heat and changes its state to solid. endothermic process, absorbs energy
Boiling: Liquid matters gain heat and change their states to gas. in the form of heat.

Condensation: Opposite process of boiling is called condensation. Gas The blue arrows in the figure
molecules lose heat and change its phase to liquid. indicates exothermic process, releases
energy in the form of heat
Sublimation: The change of state from a solid directly into a gas.
Deposition : The phase transition in which gas transforms into solid without
passing through the liquid phase. One example of deposition is the process by
which, in sub-freezing air, water vapor changes directly to ice without first
becoming a liquid.
During phase change, temperature of matters stay constant. Graphs of phase
change are given below.
Temperature
O
C
gas Condensation

Condensation gas-liquid
Point liquid freezing

freezing liquid-solid
point
solid

I II III IV V Time(s)
How Clouds are formed?
Clouds form when warm, moist air rises and
Temperature vs. time graph of heated pure solid substance is given below cools. Water vapor condenses on tiny particles
to form liquid water or ice crystals.
In intervals I. III. and V. temperature of matter increases. Since matter is
pure, kinetic energy of it also increases. We can find heat gained in these intervals Water vapor
with following formula; condenses on tiny
particcles in the
Q=m.c.ΔT air, forming a
cloud.
where; m is mass, c is specific heat capacity and ΔT is change in the temperature Warm, moist
(Tfinal-Tinitial) air rises from the At a certain height,
surface. As air air cools to the dew
reises, it cools. point and condensation
In intervals II. and IV. temperature of matter stays constant because begins.
matter is changing phase. Since temperature of matter is constant, kinetic energy
of it is also constant. On the contrary, during phase change, distances between

156 CLASS - IX
MATTER AND ITS SURROUNDINGS

molecules increase, thus potential energy of matter also increases. In these


intervals, we have heterogeneous mixtures, for example in interval II. we have
solid+liquid mixture and in interval IV. we have liquid+gas mixture.We find heat
required in these intervals with following formulas;
Water never has an absolute Q=m.Lfusion or Q=m.Lvaporization
density because its density varies with
temperature. Water has its maximum where, m is mass, Lfusion is latent heat of fusion and Lvaporization is latent heat of
density of 1 g/cm3 at 4 degrees Celsius. vaporization.
When the temperature changes from
either greater or less than 4 degrees, Melting
the density will become less then What does happen to ice cubes in a glass of ice water? When ice cubes are
1 g/cm3. Water has the maximum density of placed in water, the water is at a higher temperature than the ice. Heat flows from
1 g/cm3 only when it is pure water. the water to the ice. The process of melting, that is ,change of solid state into
liquid state is also known as fusion. The amount of heat energy that is required
to change 1 kg of solid into liquid at atmospheric pressure at its melting point is
known as latent heat of fusion.
The melting point of a crystalline solid is the temperature at which the forces
holding its crystal lattice together are broken and it becomes a liquid. It is difficult
H2O(g)
molecules
to specify an exact melting point for an amorphous solid because they tend to
(water vapor) melt over a temperature range.

Example 07: Find heat required to increase temperature of 100 g ice from
0ºC to 40 ºC. (cwater = 1 cal/g ºC, Lfusion = 80 cal/g)
H2 O(l) Solution:
molecules
We melt ice first, then heat it to 40 ºC.
Q=m.Lfusion
Q1 = 100 g × 80 × cal/g
Q1 = 8000 cal
Open container Now we increase temperature from 0 to 40 ºC
Q2 = m × c × ▵ T
Q2 = 100 × 1 × (40-0)
H2O(g) Q2 = 4000 cal
molecules
(water vapor)
Qtotal = Q1+Q2=8000 cal+4000 cal = 12000 cal.

Vaporization
H2 O(l)
molecules
Vaporization is the process by which a liquid changes to a gas or vapor. If
the input of energy is gradual, the molecules tend to escape from the surface of
the liquid. Remember that molecules at the surface are attracted to fewer other
molecules than are molecules in the interior. When vaporization occurs only
at the surface of a liquid, the process is called evaporation. Even at cold
temperatures, some water molecules have enough energy to evaporate. As the
Closed container
temperature rises, more and more molecules enter the gas phase.

Evaporation occurs in both open and Figure compares evaporation in an open container with evaporation in a
closed containers. In an open container, closed container. If water is in an open container, all the molecules will eventually
water molecules that evaporate can evaporate. The time it takes for them to evaporate depends on the amount of
escape from the container.Water vapor water and the available energy. In a partially filled, closed container, the situation
collects above the liquid in a closed is different. Water vapor collects above the liquid and exerts pressure on the
container. surface of the liquid. The pressure exerted by a vapour over a liquid is called
vapor pressure.
CHEMISTRY 157
Evaporation 101.3 kPa
(1 atm) _70º
It is a process of liquid changes to gas below its boiling point. It is a
surface phenomena.The factors effecting the evaporation are as follows:
1. Temperature
On increases in the temperature, the evaporation increases.
2. Surface area
Evaporation is faster when the surface area of the liquid is more.
3. Humidity
Humidity is the amount of water vapour present in air. If the humidity
is high, the rate of evaporation decreases. Below the boiling point
4. Wind speed
The more the wind speed, the more will be the evaporation. _100º
101.3 kPa
5. Nature of liquid (1 atm)
Volatile liquids are easily evaporated.

Note
...........................................................................................
Evaporation causes cooling:
During the evaporation process, the particles of liquid which acquired
the latent heat will leave the surface of the liquid and leaving behind the
lower energy particles. Thus, the lower energy particles effects the low in
temperature and causes cooling.

Boiling At the boiling point

The temperature at which the vapor pressure of a liquid equals the


As temperature increases, water
external or atmospheric pressure is called the boiling point. It is a molecules gain kinetic energy.Vapor
bulk phenomena. The latent heat of vaporisation is the heat energy required pressure increases (black arrows) but
to change 1 kg of a liquid to gas at atmospheric pressure at its boiling point. Use is less than atmospheric pressure (red
figure to compare what happens to a liquid at temperatures below its boiling arrows).A liquid has reached its boiling
point with what happens to a liquid at its boiling point. At the boiling point, point when its vapor pressure is equal to
molecules throughout the liquid have enough energy to vaporize. Bubbles of atmospheric pressure. At sea level, the
vapor collect below the surface of the liquid and rise to the surface. boiling point of water is 100°C.

Comparison of boiling and evaporation


Basis For Boiling Evaporation
Comparison
Boiling implies a vaporization process Evaporation is a natural process, wherein the
Meaning that turns liquid into gas, when heated. liquid changes its form to gas even without
It is a fast process. heating. It is a slow process.
Phenomenon Bulk Surface
Temperature Occurs only at boiling point. Occurs at any temperature.
Appearance It forms bubbles It does not forms bubbles.
Energy Source of energy is required. Energy is supplied by the surrounding.
Temperature of
Remains constant Decreases
liquid

158 CLASS - IX
MATTER AND ITS SURROUNDINGS

Sublimation
Many substances have the ability to change directly from the solid phase
to the gas phase. Sublimation is the process by which a solid changes directly
to a gas without first becoming a liquid. Examples of sublime substances at
room temperature are, dry ice (CO2), I2, NH4Cl, Benzoic acid, camphor,
naphthalene.

Freezing
Ice melts, but dry ice, on the right,
Suppose you place liquid water in an ice tray into a freezer. As heat is removed
turns directly into a gas.
from the water, the molecules lose kinetic energy and their velocity decreases.
The molecules are less likely to flow past one another. When enough energy has
been removed, the hydrogen bonds between water molecules keep the molecules
fixed, or frozen, into set positions. “Freezing is the reverse of melting.” The
freezing point is the temperature at which a liquid is converted into a crystalline
solid.

Condensation
When a water vapor molecule loses energy, its velocity decreases. The water
vapor molecule is more likely to form a hydrogen bond with another water
molecule. The formation of a hydrogen bond releases thermal energy and
indicates a change from the vapor phase to the liquid phase.The process by which
a gas or a vapor becomes a liquid is called condensation. Condensation is the
reverse of vaporization.

Deposition
When water vapor comes in contact with a cold window in winter, it forms
a solid deposit on the window called frost. Deposition is the process by which a
subfreezing air substance changes from a gas or vapor to a solid without first becoming a liquid.
Deposition is the reverse of sublimation. Snowflakes form when water vapor high
up in the atmosphere changes directly into solid ice crystals.

Summarizing the Changes of State


Endothermic or
Change of state Direction Example
exothermic?

Melting solid → liquid endothermic Ice melts into liquid water at 0º C.

Freezing liquid → solid exothermic Liquid water freezes into ice at 0º C.

Vaporization liquid → gas endothermic Liquid water vaporizes into steam at 100º C.

Condensation gas → liquid exothermic Steam condenses into liquid water at 100º C.

Sublimation solid → gas endothermic Solid dry ice sublimes into a gas at -78º C.

CHEMISTRY 159
160
M I N D M A P

Physical Classification of Matter MATTER

CLASS - IX
Particle Nature Physical Nature
of Matter of Matter
Bose-Einstein
Solid Liquid Gas Plasma
condensate
• Very small Ex: Colour, Density,
• Have space between them Odour, Melting point,
Definite shape, • Constantly moving Boiling point, Hardness
Fixed volume Neither fixed
Distinct • Attract each other
but no fixed shape nor fixed
boundaries,
shape volume
Fixed volumes • Has mass
• Occupies space
• Has inertia
Least Higher Highest • Cannot be destroyed
compressibility compressibility compressibility

Inter conversion of matter


High density Low density Low density into different states

Sublimation

Melting
By Altering Temperature By Altering Pressure
Freezing Temperature

Surface area
Deposition
Below boiling point Evaporation Factors affecting Humidity
Boiling or vapourisation
Wind speed

Pressure
Condensation
MATTER AND ITS SURROUNDINGS

Do at Class !
Choose the correct answer: 8. According to the kinetic-molecular theory, particles
of an ideal gas
Level 01:
1. Two gases with unequal masses are injected into (a) attract each other but do not collide.
opposite ends of a long tube at the same time and (b) repel each other and collide.
allowed to diffuse toward the centre. They should (c) neither attract nor repel each other but collide.
begin to mix (d) neither attract nor repel each other and do not
(a) in approximately five minutes. collide.
(b) closer to the end that holds the heavier gas. 9. Which is an example of gas diffusion?
(c) closer to the end that holds the lighter gas.
(d) exactly in the middle. (a) inflating a flat tire
(b) the odour of perfume spreading throughout a
2. According to the kinetic-molecular theory, particles room
of matter (c) a cylinder of oxygen stored under high pressure
(a) are in constant motion. (d) All of the above
(c) have different colours.
(b) have different shapes. 10. By which process do gases take the shape of their
(d) are always fluid container?
(a) evaporation (b) expansion
3. Which process can be explained by the kinetic- (c) adhesion (d) diffusion
molecular theory?
(a) combustion (b) oxidation 11. 
According to the kinetic-molecular theory, how
(c) condensation (d) displacement reactions does a gas expand?
(a) Its particles become larger.
4. According to the kinetic-molecular theory, which (b) Collisions between particles become elastic
substances are made of particles? (c) Its temperature rises.
(a) gases only (b) liquids only (d) Its particles move greater distances.
(c) all matter (d) all matter except solids
12. Which is an example of effusion?
5. According to the kinetic-molecular theory, particles (a) air slowly escaping from a pinhole in a tire
of matter are in motion in (b) the aroma of a cooling pie spreading across a
(a) gases only. (b) gases and liquids only. room
(c) solids, liquids, and gases. (d) solids only. (c) helium dispersing into a room after a balloon
pops
6. An ideal gas is a hypothetical gas
(d) oxygen and gasoline fumes mixing in an
(a) not made of particles. automobile carburettor
(b) that conforms to all of the assumptions of the
kinetic theory. 13. The particles in a liquid are usually
(c) whose particles have zero mass. (a) closer together and lower in energy than those in
(d) made of motionless particles. a solid
7. A real gas (b) farther apart and higher in energy than those in a
gas.
(a) does not obey all the assumptions of the kinetic-
(c) closer together and lower in energy than those in
molecular theory.
a gas.
(b) consists of particles that do not occupy space.
(c) cannot be condensed (d) farther apart and lower in energy than those in a
(d) cannot be produced in scientific laboratories. solid

CHEMISTRY 161
14. Which term best describes the process by which 21. At pressures greater than 1 atm, water will boil at
particles escape from the surface of a non boiling
(a) a temperature higher than 100°C
liquid and enter the gas state?
(b) a temperature lower than 100°C
(a) sublimation (b) evaporation (c) 100°C
(c) surface tension (d) aeration (d) 4°C

15. 
A solid forms when the average energy of a 22. One day in chemistry lab you take some ice cubes
substance's particles from the freezer and put them in a beaker. You
turn on the Bunsen burner and begin heating the
(a) increases.
ice cubes. As the temperature rises the cubes begin
(b) decreases.
to melt. What is this experiment an example of?
(c) decreases then increases.
(d) creates a random arrangement. (a) freezing (b) condensation
(c) heat of vaporization (d) heat of fusion
16. Particles within a solid
23. Rank the states of matter from weakest to strongest
(a) do not move.
intermolecular forces.
(b) vibrate about fixed positions.
(c) move about freely. (a) gas, solid, liquid (b) gas, liquid, solid
(d) exchange positions easily. (c) solid, liquid, gas (d) solid, gas, liquid

17. The compressibility of solids is generally 24.The diagram shows three kinds of liquids that have
(a) lower than the compressibility of liquids and different densities. Water floats on corn syrup and
gases. sinks in oil. Which correctly classifies the relative
(b) higher than the compressibility of liquids only. density of water?
(c) about equal to the compressibility of liquids and
gases.
Oil
(d) higher than the compressibility of gases only.
Water
18. Which causes the high density of solids? Corn Syrup
(a) The particles are more massive than those in (a) less than oil (b) greater than oil
liquids. (c) equal to corn syrup (d) greater than corn syrup
(b) The intermolecular forces between particles are
weak. Level 02:
(c) The particles are packed closely together.
25. A student has two identical glasses of milk except
(d) The energy of the particles is very high. that the temperature of the milk in one glass is
19. If the rate of evaporation from the surface of a 40ºF and the temperature of the milk in the other
liquid exceeds the rate of condensation, glass is 80ºF. The milk at which temperature has
more thermal energy?
(a) the system is in equilibrium.
(b) the liquid is boiling. (a) The milk at 40ºF has more thermal energy.
(c) energy as heat is no longer available. (b) The milk at 80ºF has more thermal energy.
(d) the concentration of the vapour is increasing. (c) Both the milk at 40ºF and the milk at 80ºF have
the same amount of thermal energy.
20. When energy as heat is applied to a liquid-vapour (d) Neither the milk at 40ºF nor the milk at 80ºF
system at equilibrium, a new equilibrium state will has any thermal energy.
have
26. 
During which change of state do atoms or
(a) a higher percentage of liquid
molecules become more ordered?
(b) a higher percentage of vapour.
(c) equal amounts of liquid and vapour (a) boiling (b) condensation
(d) all liquid (c) melting (d) sublimation

162 CLASS - IX
MATTER AND ITS SURROUNDINGS

27. 
Which of the following changes of state is 36. If 0.5 L of O2(g) reacts with H2 to produce 1 L of
exothermic? H2O(g), what is the volume of H2O(g) obtained
from 1 L of O2(g)?
(a) evaporation (b) sublimation
(c) freezing (d) melting (a) 0.5 L (c) 2 L
(b) 1.5 L (d) 2.5 L
28. 
What happens to the volume of a gas inside a
piston if the temperature does not change but the 37. Air is about 78% nitrogen, 21% oxygen, and 1%
pressure is reduced? other gases. After all the oxygen is removed from
a sample of air in a glass tube and the temperature
(a) increases (b) stays the same
remains constant, the pressure exerted by the
(c) decreases (d) not enough information
remaining air
29. Convert the pressure 0.840 atm to mm Hg. (a) does not change. (b) is reduced by 78%.
(a) 365 mm Hg (c) 638 mm Hg (c) is reduced by 50%. (d) is reduced by 21%.
(b) 437 mm Hg (d) 780 mm Hg
38. If a balloon containing 3000 L of gas at 39 and
30. Standard temperature is exactly 99 kPa rises to an altitude where the pressure is
45.5 kPa and the temperature is 16°C, the volume
(a) 100ºC (c) 0ºC
of the balloon under these new conditions would
(b) 273ºC (d) 0 K.
be calculated using the following conversion factor
31. Three samples of gas each exert 740. mm Hg in ratios: ........................
separate 2 L containers. What pressure do they 99 16
exert if they are all placed in a single 2 L container? (a) 3000 L × ×
45.5 39
(a) 247 mm Hg (c) 1.48 × 103 mm Hg 289 99
(b) 3000 L × ×
(b) 740 mm Hg (d) 2.22 × 103 mm Hg 312 45.5
312 45.5
32. Calculate the volume of 12.0 g of helium at 100°C (c) 3000 L × ×
289 99
and 1.2 atm.
39 45.5
(a) 0.013 L (b) 76.5 L (d) 3000 L × ×
16 99
(c) 7,750 L (d) 56.0 L
39. Which of the following statements is false?
33. 
5.0 L of a gas goes from 1.0 atm to 1.3 atm.
(a) A characteristic gas constant has units of J/(kg K)
Calculate the final volume of this gas.
(b) STP conditions are 273 K and 101.325 kPa
(a) 6.5 L (b) 0.26 L (c) All gases are ideal gases
(c) 4.1 L (d) 3.8 L (d) An ideal gas is one that obeys the gas laws
34. If V, P, and T represent the original volume, pressure, 40. A gas has a volume of 3 m3 at a temperature of
and temperature in the correct units, and V', P', 546 K and a pressure of 101.325 kPa The volume
and T' represent the new conditions, what is the it occupies at STP is:
combined gas law?
(a) 3 m3 (b) 1.5 m3
P’V PV’
(a) PV = P’V’ (b) = (c) 6 m3 (d) 9 m3
T T T T
PV’ P’V PV P’V’ 41. 
Which of the following represents the correct
(c) = (d) =
T T T T decreasing order of evaporation?
35. The volume of a gas collected when the temperature
(a) Alcohol, water, petrol, kerosene oil
is 11.0ºC and the pressure is 710 mm Hg measures
14.8 mL. What is the calculated volume of the gas (b) Petrol, alcohol, water, kerosene oil
at 20.0ºC and 740 mm Hg? (c) Water, alcohol, kerosene oil, petrol
(a) 7.8 mL (c) 14.6 mL
(d) Kerosene oil, alcohol, petrol, water
(b) 13.7 mL (d) 15 mL

CHEMISTRY 163
42. Ice at 0 °C feels cooler than water at 0 °C, because 48. What is the physical state of A, C and E?

(i) water has additional latent heat. E

Temperature (K)
373
D
(ii) ice is a solid C
273
(iii) water has more specific heat capacity than ice. A
B

Change of state
(a) (i) only (b) (i) & (iii)
(c) (ii) only (d) (i), (ii) and (iii) (a) A - solid, C - liquid, E - gas
(b) A - solid, C - gas, E - liquid
43. According to Graham’ law at a given temperature (c) A - liquid, C - solid, E - gas
and constant pressure, the ratio of the rates of (d) A - liquid, C - gas, E - solid
diffusion rA /rB of gases A and B is given by:
(Where M and P represents molecular mass and 49. A crystalline solid ................
pressure respectively) (a) changes abruptly from solid to liquid when
(a) (MA / MB) 1/2
(b) (PA / PB) 1/2 heated
(c) (PA / MA)1/2 (d) (MB / MA)1/2 (b) has no definite melting point
44. The order of the rate of diffusion of gases NH3, (c) undergoes deformation of its geometry easily
SO2, Cl2 and CO2 is: (d) has an irregular 3-dimensional arrangements
(a) NH3 > SO2 > Cl2 > CO2
50. 
Which of the following is NOT a property of
(b) NH3 > CO2 > SO2 > Cl2 solids?
(c) Cl2> SO2 > CO2 > NH3
(a) Solids are always crystalline in nature.
(d) None of these
(b) Solids have high density and low compressibility.
45. A crystalline solid has ...............
(c) The diffusion of solids is very slow.
(a) long range order
(d) Solids have definite volume.
(b) short range order
(c) disordered arrangement II. Fill in the blanks:
(d) none of these
1. Matter is made up of small .......................
46. Amorphous substances have ................
2. The forces of attraction between the particles are
(i) definite heat of fusion ....................... in solids, ....................... in liquids
(ii) only short range order and ....................... in gases.
(iii) only long range order 3. 
....................... is the change of gaseous state
(iv) indefinite heat of fusion directly to solid state without going through liquid
state, and vice-versa
(a) (i) and (iii) are correct
(b) (ii) and (iii) are correct 4. Evaporation causes .......................
(c) (iii) and (iv) are correct
5. Latent heat of fusion is the amount of heat energy
(d) (ii) and (iv) are correct
required to change 1 kg of solid into liquid at its
47. Amorphous solids ................ .......................

(a) possess sharp melting points 6. 


Solid, liquid and gas are called the three
....................... of matter.
(b) exhibit anisotropy
7. The smell of perfume gradually spreads across a
(c) do not undergo clean cleavage when cut
room due to .......................
with knife
(d) possess orderly arrangement over long distances 8. Rapid evaporation depends on the .....................
area exposed to atmosphere.
164 CLASS - IX
MATTER AND ITS SURROUNDINGS

9. 
As the temperature of a system increases, the IV. Graph Based Questions:
pressure of the gases .......................
1. Complete the following phase diagram:
10. As the volume of a specific amount of gas decreases,
Phase Change Diagram
it’s pressure ....................... ----------------------

11. As the temperature of a gas decreases, I’s volume

Temperature
.......................
12. Gas molecules at higher temperatures have more ----------------------

....................... than at cooler temperatures.


----------------------

13. 
Usually the total charge of a plasma is
Heat Energy
.......................
A. In the diagram given above, label all of the states
14. The pressure inside of a sealed tube if you raise the
of matter.
temperature go .......................
B. Assuming the above graph is for water; label the
15. Forces of attraction in liquids are ....................... temperatures of the flat portions of the graph.
than in olid
2. Using Charles law, determine the accuracy of the
16. Liquids that move quickly downhill are described data plotted in the graph
as having ........................ 800
700
III. Give one line reasons for the following: (300 K, 600mL)
600
Volume (mL)

1. Why hot water cools faster than warm water? 500


400
(200 K, 400mL)
2. Why do we see water droplets collected on the 300
outer surface of a glass container, containing ice? 200
(100 K, 200mL)
100
3. Why is it advisable to use pressure cooker at higher
altitudes? 0
0 50 100 150 200 250 300 350 400
Temperature (K)
4. The melting points of solids (A), (B), (C) and (D)
are 50 °C, 250°C, 110°C and 160°C respectively. 3. The data in table show the volume of hydrogen
5. Name the solid which has strongest inter particle gas collected at several different temperatures.
forces of attraction. Illustrate these data with a graph. Use the graph
to complete the table. Determine the temperature
6. 
Are plasticine and play dough solids or liquids? at which the volume will reach a value of 0 mL.
Explain why. What is this temperature called?
7. Have you ever noticed that when you dive down
Volume of H2 Collected
to the bottom of a swimming pool, your ears start
Trial T(ºC) V(mL)
to hurt? The further you dive down, the more it
1 300 48
hurts.
2 175 37
8. 
Why does it take more energy to boil 10 g of 3 110
liquid water than to melt an equivalent mass of 4 0 22
ice? 5 15
6 -150 11
9. A diver can cut through water while swimming.
Name the property of matter shown by this activity.
4. Kate placed 100 mL of water in five different pans,
10. W
 hy addition of common salt to water increases placed the pans on a windowsill for a week, and
the boiling point of the solution? measured how much water evaporated. Draw a

CHEMISTRY 165
graph of her data, shown below, with surface area 4. If 15.0 L of neon at 25.0 °C is allowed to expand
on the x-axis. Is the graph linear or nonlinear? to 45.0 L, what must the new temperature be to
What does this tell you? maintain constant pressure?
Pan number 1 2 3 4 5 5. One mole of H2S gas escapes from a container by
Surface area (cm )
2
44 82 20 30 65 effusion in 77 seconds. How long would it take
Volume evaporated (mL) 42 79 19 29 62 one mole of NH3 gas to escape from the same
container?
V. Numerical problems: 6. Convert 630.0 Torr to atmospheres and kilopascals.
1. A balloon is filled with 35.0 L of helium in the 7. Calculate the density of O2 at STP. Assume ideal
morning when the temperature is 20ºC By noon gas behaviour.
the temperature has risen to 45ºC What is the new
volume of the balloon? 8. 
Water has a heat of vaporization of 2266 J/g.
How many grams of water can be condensed by
2. A 35 L tank of oxygen is at 315 K with an internal releasing 2023 J of energy?
pressure of 190 atmospheres. How many moles of
gas does the tank contain? 9. 
Find the relative rate of diffusion for the gases
chlorine, Cl2 and ethane, C2H6.
3. 6.0 L of gas in a piston at a pressure of 1.0 atm are
10. At 22.0°C and a pressure of 755 torr, a gas was
compressed until the volume is 3.5 L. What is the
found to have a density of 1.13 g/L. Calculate its
new pressure inside the piston?
molecular mass.

Do at Home !
I. Fill in the blanks: 1. 
Each type of matter has certain properties that
make it fmrjfv..........................................
1. The temperature at which the vapour pressure of a
liquid is equal to the external atmospheric pressure 2. All matter has mass and takes up hkzxv....................
is called ........................... point.
3. All gases will completely fill any closed xlmgzrmvi.
2. Collisions between gas molecules are assumed to ....................................
be perfectly ...................................
4. Although liquids have no shape of their own, their
3. The ...................... theory states that tiny particles elofnv can be measured ....................................
in all forms of matter are in constant motion.
5. 
Changes in temperature can cause “changes of
4. 
Unlike the other states of matter, .................... hgzgv” in matter. ....................................
cannot flow.
6. Ezklirazgrlm is the process of a liquid becoming a
5. The temperature at which the motion of particles
gas .....................................
theoretically ceases is known as .................. zero.
6. The molar volume of a gas at ........................... 7. 
When a solid is heated, its molecules become
occupies 22.4 L energized and move zkzig ..................................

8. Nlovxfovh make it possible for matter to change


II. Reverse alphabet:
states .....................................
Use the code below to uncover the words in bold and
complete each sentence. III. State True/ False:
A = Z, B = Y, C = X, D = W, E = V, F = U, G = T,
1. Boiling is a bulk phenomenon.
H = S, I = R, J = Q, K = P, L = O, M = N,
2. Evaporation is a surface phenomenon.
N = M, O = L, P = K, Q = J, R = I, S = H, T = G,
U = F, V = E, W = D, X = C, Y = B, Z = A 3. 
The rate of evaporation depends only on the
166 CLASS - IX
MATTER AND ITS SURROUNDINGS

surface area exposed to the atmosphere. 2. Match the following:


Column I Column II
4. 
Latent heat of vaporization is the heat energy
required to change 1 kg. of a liquid to gas at Liquid to gas at a fixed
(A) Evaporation (i)
atmospheric pressure at its melting point. temperature
(B) Vaporisation (ii) Solid to gas
5. Water at room temperature is a liquid (C) Sublimation (iii) Gas to solid
6. Atoms in a liquid are father apart than the atoms Liquid into gas at any
(D) Hoar frost (iv)
in a gas. temperature

7. The molecules in a gas are in constant motion. (a) (A)-(iv), (B)-(i), (C)-(ii), (D)-(iii)
(b) (A)-(i), (B)-(ii), (C)-(iii), (D)-(iv)
8. 
Gases present in air have the same pressure
throughout the entire atmosphere. (c) (A)-(ii), (B)-(iii), (C)-(iv), (D)-(i)
9. All materials move from solid to liquid to gas as (d) (A)-(iv), (B)-(i), (C)-(iii), (D)-(ii)
the temperature increases.
10. Because electrons have been stripped away from V. Give very short answers:
atoms in plasma, plasmas have a negative charge.
1. What is the effect on surface tension of temperature?
11. It is just as easy to compress a liquid, as it is to
compress a gas. 2. Liquid nitrogen is used as a commercial refrigerant
to flash freeze foods. Nitrogen boils at -196ºC What
12. 
Evaporation and boiling are the same processes is this temperature on the Kelvin temperature
because molecules move from a liquid to gaseous scale?
state.
3. At a given temperature, one liquid has a vapour
13. If we pour liquid nitrogen(N2) into a glass, it will
pressure of 240 torr and another measure 420 torr.
change its state to a solid
Which liquid probably has the lower boiling point?
14. You may find plasma in a star. Which probably has the lower heat of vaporization?

15. A system that changes from a solid state to a liquid 4. 


Why does a summer rainstorm lower the
state gains energy. temperature?
16. Plasmas are all made of the same ions. They have 5. At the petrol station, the safety sign asks for the car
different colours due to different amounts of engine to be switched off before you fill the petrol
electricity. tank. Why is this necessary?
IV. Matching List Type: 6. Why do your ears "pop" in an airplane?
1. Match the column I with column II and select the
7. What are standard temperature and pressure? Why
correct option from the codes given below:
is a standard necessary?
Column I Column II
(A) Liquid (i) Highly compressible 8. What produces more severe burns, boiling water
(B) Gas (ii) Definite volume or steam? Explain why?
(C) Plasma (iii) Super low density
9. 
Arrange the following substances in increasing
Bose-Einstein
(D) (iv) Super energetic order of intermolecular force of attraction: water,
condensate
sugar, oxygen
(a) (A) - (i), (B) - (ii), (C) - (iii), (D) - (iv)
(b) (A) - (ii), (B) - (i), (C) - (iii), (D) - (iv) 10. A drop of dettol got evenly distributed in water.
(c) (A) - (ii), (B) - (i), (C) - (iv), (D) - (iii) How?
(d) (A) - (iii), (B) - (i), (C) - (ii), (D) - (iv)
CHEMISTRY 167
Scope
Biology is a vast and intriguing subject and there is so
much more all the future scientists and biologists can contrib-
ute to develop biological science. All the health workers such
as doctors, nurse, health assistants, dentists etc come from
biological background
Chapter 1

Learning Checklist
Discoveries in cell biology
THE FUNDAMENTAL
CELL - UNIT OF LIFE Cell size, shape and Number
Plant and animal cell
Structural organisation of a cell

Introduction
Cytology is the study of the structure and composition of cell. The body
of all organisms from the simplest to the most complex one is made up of tiny
microscopic units which carry out the processes that make the organisms a living Micrographia is a historically
entity. Such a structural and functional unit of living body is called a cell. All significant book by Robert Hooke
living beings are composed of cells. Cells are the smallest structural and about his observations through various
functional unit of an organism,capable of independent existence. Any lenses. It is particularly notable for
function performed by the organism is the outcome of the acitivity of the cell. being the first book to illustrate insects,
Since all the activities of an organism are present in miniature form in each and plants etc. as seen through microscopes.
every cell, it is called the basic unit of life. Published in January 1665, the first
major publication of the Royal Society,
Discoveries in cell biology it became the first scientific best-seller,
Robert Hooke (1665), an English botanist, observed thin sections of cork of inspiring a wide public interest in the
bark of a tree under a self- designed compound microscope and noticed honey- new science of microscopy.
comb like compartments. He coined them as cells. The term cell was derived
from a Latin word (cella = a little room). He explained his observations in a book
namely, Micrographia. He actually observed the rigid cell walls of dead cells.
Anton Von Leeuwenhock (1674), a Dutch drapper, was the first to observe
living cells like bacteria (from tartar of teeth), erythrocytes of a fish), sperms (were
called animalcules) and protozoans, (e.g.,Vorticella).
In 1838, Matthias Schleiden, a German botanist, examined a large number
of plants and observed that all plants are composed of different kinds of cells
which form the tissues of the plant.
At about the same time, Theodore Schwann (1839), a British Zoologist,
studied different types of animal cells and reported that cells had a thin outer layer
which is today known as the ‘plasma membrane’. He also concluded, based Eyepiece
on his studies on plant tissues, that the presence of cell wall is a unique character Oil Lamp
of the plant cells. On the basis of this, Schwann proposed the hypothesis that the Water Flask
bodies of animals and plants are composed of cells and products of cells.
Barrel
Schleiden and Schwann together formulated the cell theory. This theory
however, did not explain as to how new cells were formed. Rudolf Virchow Focusing Screw
Objective
(1855) first explained that cells divided and new cells are formed from pre-existing Specimen Holder
cells (Omnis cellula-e cellula). He modified the hypothesis of Schleiden and
Schwann to give the cell theory a final shape. Cell theory as understood today is:
Robert Hooke’s microscope

170 CLASS - IX
CELL (THE FUNDAMENTAL UNIT OF LIFE)

Cell Theory
Eyepieces 1. All living organisms are composed of cells or cell products.
Trinocular (Ocular)
(camera) Port 2. All new cells arise as a result of division of pre-existing cells.
3. All cells are basically alike in chemical composition and metabolic processes.
4. The function of an organism as a whole is the outcome of the combined
Objective
Lens activities and interactions of the constituent cells.
Specimen
Goes here
5. A cell is the structural and functional unit of all living beings.
Stage

Fine Condenser Compound Microscope


Focusing Light
Bulb The ordinary light or compound microscope is used extensively in laboratories
Coarse Focusing these days. It is a greatly improved design of Hooke’s microscope. It consists of
two lenses, the eyepiece lens and the objective lens, which are combined to
Light microscope produce a greater magnification.
These microscopes use light (generally sunlight) to illuminate the object. So,
these compound microscopes are called light microscopes. In these microscopes,
many lenses are combined together and their magnification power ranges from
300 to 1500 times, good enough to see cells, larger organelles, and bacteria.

Cell Size, Shape, and Number


Amazing Electron Microscope Look at the structure of onion peel cells. Do all cells look alike in shape and
Images size? Yes, all cells are of same shape and almost of same size.

Salt and pepper Nucleus


Cell wall
Cytoplasm

Onion peel cells


The eye of a needle, threaded with
red cotton Cell Size
The size of the cell ranges between broad limits. Some cells are extremely
small and can be seen only when magnified and is visible only with a microscope
since, they are only a few micrometres in diameter. Cells are limited in size by the
ratio between their outer surface area and their volume. Small cells have more
surface area for their volume of cytoplasm than large cells. As the cell grows, the
Dust, magnified 22million times amount of surface area becomes too small to allow materials to enter and leave
the cell quickly enough. Cell size is also limited by the amount of cytoplasmic
activity that the cell’s nucleus can control.
Actually, the size of the animal is dependent on the number of cells
and not the size of the cells.The cell size remains constant for a particular cell and
is independent of the size of the organism. The large size of the elephant is due
50x zoom of human eyelash hairs to the larger number of cells present in the body.
BIOLOGY 171
Flu
virus
Animal
cell
Mitochondria Human Chicken
egg egg
Protein
Atom

Plant Frog
Lipids cell egg Ostrich
Bacteria egg Adult
female
Relatives sizes on a logarithmic scale

0.1 nm 1 nm 10 nm 100 nm 1 μm 10 μm 100 μm 1 mm 10 mm 100 mm 1m

Naked eye

Light microscope

Electron microscope

Cell shape
The shape of the cells is more variable than their size. The shape of the cell
may be spherical, oval, elliptical, spindle shaped, polygonal, or flat-plate like. The
shape of the cell is determined by the specific function of the cell. Some
of the cells like amoeba and white blood cells can change their shape, while most
of the plant cells and animal cells have almost fixed shapes. The smallest cell found is
Among unicellular organisms, for example, in amoeba, the shape of the cell a mycoplasma cell (PPLO),
(body) is irregular, while among multi-cellular organisms like in plants, the which is about 0.1 micron
xylem and phloem elements are elongated tubular structures so as to help in diameter. The longest cells are the
in conduction of water and minerals; the dividing meristematic cells are nerve cells, measuring about a metre in
isodiametric in shape. Similarly in animals cells, the nerve cells are elongated and length. The largest cells are represented
thread like, which enables an efficient communication system, the muscle cells by eggs of Ostrich, which is about
are elongated and contractile that helps in the movement of bones, while the red 170–135mm.
blood cells are biconcave shaped.This kind of shape enables RBCs to easily move
through capillaries and thus permit gaseous exchange.
Ovum or egg cell
Cheek cell

Bone cell
Sperm cell

Smooth muscle cell

Cell from
Intestinal lining Connective tissue cell
Nerve cell

Some of the examples of animal cells


172 CLASS - IX
CELL (THE FUNDAMENTAL UNIT OF LIFE)

Cell number
The number of cells varies in all living organisms. Based on the number of
cells, the organisms are divided into two types :
(i) Unicellular organisms (Single-celled organisms): The organisms made
up of single cells are called unicellular organisms.Amoeba,paramecium,bacteria,
etc., are examples of unicellular organisms. The single cells have the ability to
perform all the life processes like digestion, respiration, excretion, growth and
reproduction. In these cells, there is no division of labour as the single cells
have to perform all the activities. A flagellum (plural: flagella) is a long, whip-
like structure that helps some single celled organisms like bacteria, archaea, to
move. It is composed of microtubules.They help propel cells and organisms in
a whip-like motion.

a b
Pseudopodia Contractile vacuole
Cytoplasm
Cell membrane
Murein cell wall
Cell capsule
Food vacuole
Mitochondrion
Nucleoid Nucleus
Plasmid
Endoplasm
Membrane
Flagellum

a. Bacteria, b. Amoeba

(ii) Multi-cellular organisms (Multi-celled organisms): The organisms


made up of a number of cells are called multi-cellular organisms. Fungi,
plants, and animals are examples of multi-cellular organisms. All the cells
of multicellular organisms have a similar basic structure and similar basic
life activities.These cells group together to produce different tissues.Tissue
forms organs and organs give rise to organ system that performs specialized
functions.

Unicellular organisms Multi-cellular organisms


The organisms made up of large
The organisms made up of single cells
Note number of cells are called multi-
are called unicellular organisms.
.......................................... cellular organisms.
There is no division of labour. A single There is division of labour. Cells
All the cells of a multi-cellular cell performs all the activities of the are specialized to perform different
organism develop from a single cell organism. functions.
called the zygote. The zygote divides
The life span of multi-cellular
to form a number of cells that The life span of the organism is short.
organism is long.
differentiate to have different shapes,
Examples include amoeba, bacteria, Examples include fungi,plants, and
structure, and functions.
paramecium, etc; animals.
BIOLOGY 173
Cells are of two types
(i) Plant cell (ii) Animal cell.
Though many things are common between animal and plants cell they differ
in some way.

Cilia
Lysosome
Plant Cell
-10 - 100 micrometers in length
Vacuoles
- Typically rectangular or
cubic in shape

Cell membrane

Centrioles Vacuole
Cell Type Lifespan
Granulocytes:
Animal Cell Lysosome eosinophils
10 hrs to 3 days
basophils,
-10 - 30 micrometers in length
Cell wall neutrophils
- Typically round or irregular
Plasmodesmata Stomach lining
in shape Cell membrane 2 days
Chloroplasts cells

Sperm cells 2-3 days


Plant Cell Animal Cell
Plant cells are larger in size compared An animal cell is comparatively Stomach lining
2 days
to animal cell. smaller in size. cells
Cell wall is present around cell
Cell wall is absent. Colon cells 3-4 days
membrane.
Plastids are present. Plastids are absent.
Epithelia of
A single large central vacuole is 1 week or less
Few small vacuoles are present. small intestine
present.
Golgi body present in the form of Platelets 10 days
Golgi body is well developed.
units known as dictyosomes.
Nucleus lies on one side in the Skin epidermal
Nucleus lies in the center of the cell. 2 - 4 weeks
peripheral cytoplasm. cells
Centrosomes and centrioles are Centrosome with centrioles are 2 months - a year
absent. present. Lymphocytes
(highly variable)
Structural organisation of a cell Red blood cells 4 months
A cell usually possesses a number of components called cell organelles. Each
cell organelles performs specific functions. Hence, a cell may be defined as a unit
Macrophages months - years
of protoplasm, bounded by a cell membrane or plasma membrane. Protoplasm
is the life giving substance and includes cytoplasm and nucleus. Cytoplasm, on
other hand, contains organelles such as mitochondria, endoplasmic reticulum, Endothelial cells months - years
golgibodies, lysosomes, vacuoles, plastids, and ribosomes. A typical cell, either a
plant cell or an animal cell, is formed of three basic parts: Pancreas cells 1 year or more
(i) Cell membrane (ii) Nucleus (iii) Cytoplasm
Plant cell, fungi, and some bacteria contains an additional covering called cell Bone Cells 25 - 30 years
wall.
174 CLASS - IX
CELL (THE FUNDAMENTAL UNIT OF LIFE)

Cell

Cell Wall Protoplast

Cell membrane Protoplasm

Cytoplasm Nucleus

Nuclear membrane
Cytosol Cell Organelles
Nucleoplasm
Chromatin
Membrane bound Nonmembranous
cell organelles cell organelles Nucleolus

Endoplasmic reticulum (ER) Ribosomes


Smooth ER, Rough ER
Golgi complex
Centrosome
Mitochondria
Note
.......................................... Plastids
Cell staining is a technique Vacuoles
that can be used to better visualize
cells and cell components under a Microbodies
Lysosomes,
microscope. By using different stains, Peroxysomes
one can preferentially stain certain Glyoxysomes
cell components, such as a nucleus
or a cell wall, or the entire cell.
Common stains and their uses are as I. Cell Membrane–Plasma Membrane
follows: It is the outermost, extremely delicate elastic membranous covering of each
Iodine: Stains carbohydrates in plant cell. It separates the cytoplasm of the cell from its environment. Plasma membrane
and animal specimens brown or blue- is a living, selectively permeable membrane. It allows some selected materials to
black. Stains glycogen red. move in and out of the cell,and prevents the entry and exit of other substances.
Methylene blue: Stains acidic cell
parts (like nucleus) blue. The plasma membrane is made up of a bilayer of lipids and proteins. Small
carbohydrates are attached at places to one surface of lipids and proteins. Several
Eosin Y: Stains alkaline cell parts
models have been proposed regarding the arrangement of proteins and lipids.The
(like cytoplasm) pink.
Fluid Mosaic Model proposed by Singer and Nicholson is widely accepted.
Crystal Violet: Stains bacteria
purple. According to fluid mosaic model
(i) The cell membrane is composed of lipid bilayer of phospholipids molecules
into which a variety of globular proteins are embedded.
(ii) Each phospholipids molecule has two ends - an outer head hydrophilic
(water attracting) and the inner tail hydrophobic (water repelling).
(iii) Protein molecules are arranged in two ways. Peripheral proteins or
extrinsic proteins are present on the outer and inner surface of lipid
bilayer, while integral proteins or intrinsic proteins are embedded inside
Stained polled grains, visualized
under compound microscope. the lipid bilayer partially or wholly.

BIOLOGY 175
Lipid-anchor
protein
Peripheral
Cytoplasm membrane
protein Hydrophilic
phospholipid
head group

Phospholipid
bilayer
Hydrophobic fatty
Cell acyl side chains
Lipid - anchored Peripheral membrane
protein protein
Integral membrane
protein

Fluid mosaic model


Functions of Plasma Membrane
(i) It provides definite shape to the cell.
Hypertonic
(ii) 
It functions as a mechanical barrier between external and internal
environment of the cells. Hypertonic
(iii) It regulates the movement of molecules in and out of cell.
H2O
(iv) The flexibility of membrane helps the cell to engulf food and other
substances from its external environment by endocytosis. H2O

Transport Across The Membrane Plasmolyzed


(A) 
Passive transport : The transport of substances across the plasma Plasmolyzed
membrane without the expenditure of energy is known as passive transport. Isotonic
It is of two types: Diffusion and Osmosis.
H2O H2O
(i) Diffusion : The process of movement of substance from the region of
higher concentration to the region of lower concentration, so as to spread H 2O H2O
the substance uniformly in the given space is known as diffusion. The process
does not require energy. H2O
(ii) Osmosis : It is defined as the diffusion of water from region of higher
Flaccid
concentration to region to lower concentration across the semi-permeable
membrane. This kind of movement is along concentration gradient. There
is no expenditure of energy. Hypotonic
H2O
H2O Vacuole
Types of Osmosis
(i) Endosmosis : Endosmosis is the osmotic entry of water into the cell. H2O
H2O
(ii) Exosmosis : Exosmosis is the osmotic withdrawal of water from the cell.
There are three types of osmotic solution, which can cause osmosis across
semi-permeable membrane.
Turgid
 ypotonic solution (Hypo: less than or lower) : Hypotonic is the
(i) H
solution which has lower osmotic concentration than the cell.
Types of osmotic solution
176 CLASS - IX
CELL (THE FUNDAMENTAL UNIT OF LIFE)

 ypertonic solution (Hyper: more or higher): Hypertonic is the


(ii) H
solution that has higher water concentration than the cell.
(iii) I sotonic solution (Iso: same): It is the solution that has the same osmotic
concentration as inside the cell.

Diffusion Osmosis
It is the movement of water from the region of higher
It is the movement of substances from the region of
concentration to a region of lower concentration
higher concentration to a region of lower concentration
through a semi-permeable membrane.
It takes place in all three medium-solid, liquid, and gas. It takes place only in liquid medium.
It does not involve any semi-permeable membrane. It requires a semi-permeable membrane.
It is not much influenced by the presence of other It is influenced by the concentration and type of solute
substances. particle.
Plasmolysis
Plasmolysis is the phenomenon of the loss of water from a plant cell by osmosis
when kept in a hypertonic solution, so that the cytoplasm shrinks away from the
cell wall. If a plant cell is kept in a solution more concentrated than the cell sap,
i.e., in hypertonic solution, water moves out to the outside solution. Loss of
water causes the vacuole to shrink and pull the plasma membrane with the cell
contents away from the cell wall. This phenomenon is known as plasmolysis.
(B) Active transport: Active transport is the process of transport of
molecules aross the plasma membrane against the concentration gradient. The
process requires the use of energy. For active transport, the cell membrane
possesses ATP - mediated carrier protein. Glucose, amino acids, and ions pass
through plasma membrane by active transport.

II. Cell Wall


Plant cells Plant cells, in addition to the plasma membrane, have another rigid outer
Middle
lamella covering called cell wall. The cell wall lies outside the plasma membrane.
The plant cell wall is mainly composed of cellullose, hemicellulose and pectin.
Primary
cell wall Cellulose is a complex substance and provide structural strength to plants.
Cellulose is permeable to water, solute and gases. In fungi, the cell wall is made up
Plasma
membrane of chitin A cementing layer called middle lamella is present between the walls
of two adjacent cells. Middle lamella is formed of pectin, calcuim and magnesium
Cellulose
Pectin Hemicellulose microfibrils pectate. Animal cells lack a cell wall.

Structure of cell wall Cell wall Plasma membrane


It is found in both plant and animal
It is found only in plant cells.
cells.
It lies on the outside of animal cell
It is outermost layer of the plant cell.
and inner to the cell wall in plant cell.
It is thick and rigid. It is thin and flexible.
It is permeable. It is selectively permeable.
It is formed of cellulose and pectin. It is formed of lipids and proteins.
It acts as mechanical barrier and
It provides protection and strength to
regulates the movement of molecules
the cell
in and out of cells.

BIOLOGY 177
Functions of Cell Wall : Class Room Activity-01
1. It gives a defnite shape to the cells.
2. It provides mechanical strength to plants. Experiment 01-
3. I t protects the cell against mechanical injury and pathogens. Remove the shell of an egg by
4. I t helps in the transport of various substances across it. dissolving it in dilute hydrochloric
5. Cell wall helps the plant cells to withstand a lot of variation in the acid or vinegar. The shell is mostly
surrounding environment. calcium carbonate. A thin outer
6. Cell wall prevents the bursting of cell on endosmosis as it is quite thick and skin now encloses the egg. Put the
rigid. egg in pure water and observe after
5 minutes.
III. Nucleus
Naked egg membrane
Nucleus is the prominent, spherical structure found at the center of the cell.
Nucleus was discovered by an English biologist, Robert Brown in 1831. It is the Movement of
largest organelle present in cell. Basically,nucleus is the controlling centre of all water
cell activities and hence, it has been described as the brain of the cell.
In plant cell,nucleus lies towards the periphery due to the presence of large
central vacuole while in animal cell,nucleus lies in the central position.
Vinegar
The nucleus has a double layered covering called nuclear membrane.
The nuclear membrane has pores which allow the transfer of material from What do we observe?
inside the nucleus to its outside, that is,to the cytoplasm. The nucleus contains The egg swells because water
chromosomes, which are visible as rod-shaped structures only when the cell is passes into it by osmosis.
about to divide. Chromosomes contain information for inheritance of features
from parents to next generation in the form of DNA (Deoxyribo Nucleic Experiment 02-
Acid) molecules. Chromosomes are composed of DNA and protein. DNA Place a similar de-shelled egg in a
molecules contain the information necessary for constructing and organising concentrated salt solution or corn
cells. Functional segments of DNA are called genes. In a cell which is not syrup and observe for 5 minutes.
dividing, this DNA is present as part of chromatin material. Chromatin material The egg shrinks. Why? Water
is visible as an entangled mass of thread-like structures.Whenever the cell is about passes out of the egg solution into
to divide, the chromatin material gets organised into chromosomes. the salt solution because the salt
The nucleus plays a central role in cellular reproduction, the process by which a solution is more concentrated.
single cell divides and forms two new cells. It also plays a crucial part, along with Naked egg membrane
the environment, in determining the way the cell will develop and what form it
will exhibit at maturity, by directing the chemical activities of the cell.
Movement of
In some organisms like bacteria, the nuclear region of the cell may be poorly water
defined due to the absence of a nuclear membrane. Such an undefined nuclear
region containing only nucleic acids is called a nucleoid. Such organisms,
whose cells lack a nuclear membrane, are called prokaryotes (Pro = primitive
or primary; karyote ≈ karyon = nucleus). Organisms with cells having a nuclear Corn syrup
membrane are called eukaryotes.

Gene
Cell Nucleus Chromosome DNA (Segment of DNA)

178 CLASS - IX
CELL (THE FUNDAMENTAL UNIT OF LIFE)

Prokaryotic cells also lack most of the other cytoplasmic organelles present
in eukaryotic cells. Many of the functions of such organelles are also performed
by poorly organised parts of the cytoplasm. The chlorophyll in photosynthetic
prokaryotic bacteria is associated with membranous vesicles but not with plastids
as in eukaryotic cells .

a b Nucleus Nucleolus Mitochondrion


Cytoskeleton
Ribosomes
Cytoplasm

Golgi
Nucleoid
apparatus

Capsule
Cell Wall
Cytoplasmic
Membrane
Ribosomes Rough
endoplasmic
reticlum
Pili Centrioles
Smooth
Flagella endoplasmic
Peroxisome Plasma reticulum
membrane

Structure of a.Prokaryotic cell b.Eukaryotic cell

Nucleoplasm (Nuclear sap) :


It is the colourless, cytoplasm within nuclear membrane in which chromatin
and nucleoli are suspended.

Nucleolus:
It is the rich in protein and RNA. Basically, Nucleolus is the site of ribosome
formation. Ribosome, in turn helps in protein synthesis in the cytoplasm.
Nucleolus is known as the factory of ribosomes.

Prokaryotes Eukaryotes
An organized nucleus is absent.Hereditary material lies free
A well-organized nucleus is present. It contains hereditary
in cytoplasm and is known as nucleoid.Nuclear membrane
material covered by a nuclear membrane.
is absent.
A single chromosome is present and is represented by a Numbers of rod-like chromosomes,with two or several
single circular DNA only. DNA molecules are present.

Membrane-bound cell organelles are absent. Membrane-bound cell organelles are present.

Distinct compartments in the cell i.e. the cytoplasm and the


No compartments are found in prokaryotes.
nucleus is present in eukaryotes.
Examples: Bacteria and Cyanobacteria. Examples: Plants, Animals, and Fungi.

BIOLOGY 179
Functions of Nucleus:
(i) Nucleus is the chief controlling center of cell. It contains chromosomes
which carry genes. Genes are responsible for the transmission of hereditary
characters from parents to offspring.
(ii) Nucleus controls all metabolic activities taking place in the cell. If you unwrap all of the
(iii) It participates directly in cell division to produce genetically identical DNA you have in all your cells, you
daughter cell by a process known as mitosis. could reach the moon 6000 times.
IV. Cytoplasm: One gram of DNA holds as
Cytoplasm occpies the major part of the cell. It is a living component of much informations as 1 trillion CD`s.
cell, consisting of transparent, semi-fluid granular substance. It is limited on the 99.9 % of our DNA sequence
outside by the cell membrane. Water is the main component of the cytoplasm. is same as other humans.
Cytoplasm has two major parts:
(i) Cytosol: Cytosol is the fluid part of cytoplasm. It is viscuous, and contains a
number of substances like water, ions, enzymes, vitamins, carbohydrates, lipids,
and proteins. All major biochemical reaction take place in the cytoplasm.
(ii) Cell organelles: These organelles include Mitochondria, Endoplasmic
reticulum, Golgi bodies. Plastids,Lysosome, Peroxisomes, Ribosomes, and
Centrosomes.
 n the basis of membranous covering, cell organelles are divided into three
O
types.
(i) M
 embrane-less organelles—Example : Ribosomes.
(ii) 
Single membrane-bound organelles Example: Endoplasmic reticulum,
Golgi bodies,Lysosomes,Vacuoles.
(iii) 
Double membrane-bound organelles Examples : Mitochondria and
plastids.

V. Endoplasmic Reticulum(ER):
Endoplasmic reticulum is a complex network of membrane - bound structure
which runs through the cytoplasm. Cisternae are spaces within the folds of the
ER membranes. It is connected to both the outer nuclear membrane as well Smooth
Endoplasmic
Endoplasmic Reticulum
as the cell membrane. The membrane has the same structure as the plasma Reticulum
Nuclear Pore
membrane but ribosomes do not have membranes. It looks like long tubules or
Nuclear
round or bags(vesicles). Envelope
Cisternal Space
Depending on the presence or absence of ribosome on the surface of ER,
it is divided into two types: (i) rough endoplasmic reticulum (RER) and (ii)
smooth endoplasmic reticulum (SER). RER looks rough under a microscope Rough Endoplasmic Cisternae
because it has particles called ribosomes attached to its surface. The ribosomes, Reticulum
Ribosomes
which are present in all active cells, are the sites of protein manufacture. The
Structure of ER
manufactured proteins are then sent to various places in the cell depending on
need, using the ER.The SER helps in the manufacture of fat molecules, or lipids,
important for cell function. Some of these proteins and lipids help in building the
cell membrane. This process is known as membrane biogenesis.

Functions of endoplasmic reticulum:


(i) Endoplasmic reticulum helps in intracellular and intercellular transport of
materials. It is the “transport system” of the cell.
(ii) I t provides large surface area for various metabolic reaction.
180 CLASS - IX
CELL (THE FUNDAMENTAL UNIT OF LIFE)

(iii) RER is the site of protein synthesis.


(iv) SER helps in lipid synthesis.
(v) SER in liver cells helps in detoxifying many drugs and poisons.
Incoming transport vesicle (vi) P
 roteins and lipids synthesized on ER are used for making cell membrane.
cis face
cisternae lumen VI. Golgi Bodies :
The Golgi apparatus, first described by Camillo Golgi, consists of a system of
membrane-bound vesicles arranged approximately parallel to each other in stacks
called cisterns. These membranes often have connections with the membranes
of ER and therefore constitute another portion of a complex cellular membrane
system. The material synthesised near the ER is packaged and dispatched to
trans face various targets inside and outside the cell through the Golgi apparatus.
newly forming vesicle secretory vesicle
Functions of Golgi bodies :
Golgi apparatus
(i) It is involved in the synthesis and repair of cell membrane.
(ii) I t is also involved in the formation of lysosomes and peroxisomes.
(iii) Secretion is the major function of Golgi apparatus. All types of substances
that are secreted and excreted are packed in vesicles by Golgi bodies for
passage to the outside. It is the secretory organelle of the cell.
(iv) Golgi apparatus also takes part in storage, modification and packaging of
various biochemical products produced by different components of the cell.
Camillo Golgi Golgi body is a single complex in animal cells, while in plant cells, it is
formed of separate units called dictyosomes.
Camillo Golgi is an Italian
doctor and n e u rologist. He is VII. Lysosomes
known for discover y of a cell
Lysosomes are a kind of waste disposal system of the cell. Lysosomes help to
organelle,named after him, i.e., Golgi
keep the cell clean by digesting any foreign material as well as worn-out cell
apparatus. In 1898, he discovered
organelles. Foreign materials entering the cell, such as bacteria or food, as well as
the Golgi apparatus in nerve cells
old organelles end up in the lysosomes, which break them up into small pieces.
of Barn owl and Cat. He named
Lysosomes are able to do this because they contain powerful digestive enzymes
the organelle “ apparato reticolore
capable of breaking down all organic material. During the disturbance in cellular
interns”. The name Golgi apparatus
metabolism, for example, when the cell gets damaged, lysosomes may burst and
was bestowed by Cajal in 1914.
the enzymes digest their own cell. Therefore, lysosomes are also known as the
Camillo Golgi shared the Nobel prize
“suicide bags” of a cell. Structurally, lysosomes are membrane-bound sacs filled
of 1906 with Santiago Ramony
with digestive enzymes. These enzymes are made by RER.
Cajal for their work on the structure
of nervous system. Phagosome

Bacterium Phagocytosis
Primary
Lysosome
Primary Lysosome

Endocytosis

Secondary
Early Endomsome Lysosome Or
Late Endomsome Endolysosomes
Mitochondria
Primary
ER Lysosome

Autophagosome
Autophagocytosis

Structure of Lysosomes
BIOLOGY 181
VIII.Peroxisome and glyoxysomes
Peroxisome, membrane-bound organelle occurring in the cytoplasm of
eukaryotic cells. Peroxisomes contain enzymes that oxidize certain molecules
normally found in the cell, notably fatty acids and amino acids. Those oxidation
reactions produce hydrogen peroxide, which is the basis of the name peroxisome.
However, hydrogen peroxide is potentially toxic to the cell, because it has the
ability to react with many other molecules. Therefore, peroxisomes also contain
enzymes such as catalase that convert hydrogen peroxide to water and oxygen,
thereby neutralizing the toxicity. In that way peroxisomes provide a safe location
for the oxidative metabolism of certain molecules. Glyoxysomes are specialized
peroxisomes found in plants (particularly in the fat storage tissues of germinating
seeds) and also in filamentous fungi.

IX. Mitochondria
ATP synthase
Richard Altman in 1890 establihed them as cell organelles and called them Inter particles
membrane
Bioblast. The term Mitochondria was coined by Carl Benda in 1898. space
Matrix
(i) 
Mitochondria are the site of cellular respiration. They use molecular
Cristae
oxygen from air to oxidize the carbohydrates and fats present in the cell to Ribosome
Granules
carbondioxide and water vapour.
Glucose + Oxygen → CO2 + Water + ATP Inner
membrane
Outer
DNA membrane
(ii) Mitochondria provide energy in the form of ATP for various metabolic
activities of living cells. Since, the mitochondria synthesize ATP, it is also Structure of Mitochondrion
known as power house of cell.

X. Cytoskeleton
Cytoskeleton, a system of filaments or fibres that is present in the cytoplasm
of eukaryotic cells The cytoskeleton organizes other constituents of the cell,
maintains the cell’s shape, and is responsible for the locomotion of the cell itself
and the movement of the various organelles within it.The filaments that comprise
the cytoskeleton are so small that their existence was only discovered because of
the greater resolving power of the electron microscope.
Three major types of filaments make up the cytoskeleton: actin filaments,
microtubules, and intermediate filaments. Actin filaments occur in a cell in
the form of mesh works or bundles of parallel fibres; they help determine the Cytoplasm

shape of the cell and also help it adhere to the substrate. The constantly changing Microfilament

arrays of actin filaments help move the cell and mediate specific activities within Intermediate
filament
it, such as cell cleavage during mitosis.
Microtubules are longer filaments that are constantly assembling and Microtuble
disassembling; they play a crucial role in moving the daughter chromosomes to the
newly forming daughter cells during mitosis, and bundles of microtubules form
Organelles
the cilia and flagella found in protozoans and in the cells of some multicellular immobilized by
the cytoskeleton
animals. mesh

Intermediate filaments, in contrast to actin filaments and microtubules, are


very stable structures that form the true skeleton of the cell. They anchor the
nucleus and position it within the cell, and they give the cell its elastic properties
Structure Of cytoskeleton
and its ability to withstand tension.

182 CLASS - IX
CELL (THE FUNDAMENTAL UNIT OF LIFE)

XI. Plastids :
The term plastids was coined by Ernst Heackel Plastids are found only in
plant cells. They are absent in animal cells. There are two types of plastids –
chromoplasts (coloured plastids) and leucoplasts (white or colourless plastids)
(i) Chloroplasts : Chloroplasts are green colour plastids, found in leaves. The
green colour is due to the presence of chlorophyll. Chlorophyll traps the
solar energy which is used for manufacturing food. They are the sites of
photosynthesis. So, chloroplasts are the “Kitchen of the cells.”
(ii) Chromoplasts : They are coloured plastids. The colour varies from red,
orange, yellow, etc., due to the presence of carotenoids. They are mostly
found in flowers and fruits. They provide colouration to organs for attracting
pollinators.
(iii) Leucoplasts : Leucoplasts are colourless plastids. They are found in storage
cells of roots,seeds and underground stems. They take part in the storage
Outer Granum
membrane Lumen of food. Leucoplasts are of three types depending on the storage products.
Amyloplasts stores starch. Aleuroplasts stores protein while elaioplasts
stores oil and fat.
Structure of chloroplasts : A chloroplast is bounded by two membranes i.e.
outer membrane and inner membrane. The inside of the chloroplast is clearly
marked into a colourless ground matrix called stroma.
Inner Membrane Stroma
(i) Stroma is a homogenous matrix in whichgrana is embedded. Stroma contains
Thylakoids
a variety of photosynthetic enzymes, DNA, and ribosomes. It is the site where
Structure of Chloroplast all chemical reaction occurs and starch (sugar) is synthesized.
(ii) Grana are stacks of membrane - bound, flattened sacs containing the
molecules of chlorophyll. One thylakoid stack is known as granum. Each
thylakoid have chlorophyll molecules on their surface that trap sunlight and
take part in process of photosynthesis.
The stack of grana are connected by stromal lamellae. The lamellae act like
the skeleton of chloroplast, keeping all sacs in safe distance from the other sac.
Grana are main functional units of chloroplasts.

Functions of Chloroplast:
(i) Chloroplasts trap solar energy, which is used to manufacture food through
photosynthesis. Photosynthesis is the process of synthesizing food (sugar) from
carbondioxide and water in the presence of sunlight.
(ii) 
Chromoplasts provide colour to fruits and flowers to attract insects for
pollination.
(iii) L
 eucoplasts take part in the storage of proteins, starch, and oil.

XII. Vacuoles:
Vacuoles are storage sacs for solid or liquid contents.Vacuoles are small sized
Granum is the site of light reaction,
in animal cells while plant cells have very large vacuoles. The central vacuole
during photosynthesis, while stroma
of some plant cells may occupy 50 – 90% of the cell volume. In plant cells,
is the site of dark reaction during
vacuoles are full of cell sap and provide turgidity and rigidity to the cell. Many
photosynthesis.
substances of importance in the life of the plant cell are stored in vacuoles. These

BIOLOGY 183
include amino acids, sugars,various organic acids, and some proteins. In single-
celled organisms like Amoeba, the food vacuole contains the food items that the
Amoeba has consumed. In some unicellular organisms, specialised vacuoles also
play important roles in expelling excess water and some wastes from the cell.

Apoptosis is a form of programmed cell


Nucleus
death, or “cellular suicide.” It is different
Cell membrane
(pressed close to cell wall ) from necrosis, in which cells die due to
injury. Apoptosis is an orderly process
Vacuole
(contains a fluid in which the cell’s contents are packaged
called cell sap)
Cytoplasm into small packets of membrane for
Chloroplasts “garbage collection” by immune cells.

Structure of Plant Cell

Functions of Vacuole :
In plant cells :
(i) Vacuoles help to provide turgidity and rigidity to the cell.
(ii) Vacuole acts as a store house for pigments and waste products. It also stores
useful minerals and salts.
(iii) 
Sap vacuole maintains an osmotic concentration which is required for
absorption of water.

In animal cells :
(i) In single - celled organisms, like Amoeba, food vacuole helps in the digestion
of food particles.
(ii) 
Contractile vacuole takes part in osmoregulation (the maintenance of
constant osmotic pressure in the fluids of an organism by the control of water
and salt concentrations) and excretion.
(iii) They store materials such as food, water, sugar, and waste products.

Ectoplasm Food vacuole


Endoplasm

Food vacuole

Contractile
vacuole

Pseudopodium Nucleus

Structure of Amoeba

184 CLASS - IX
CELL (THE FUNDAMENTAL UNIT OF LIFE)

Brain Power On
.........................................................................................................................................
The Cell Cross Word Puzzle:
ACROSS DOWN
2 These cells contain chloroplasts 1 Protein factories
6 A cell with a nucleus 3 Break down of old cell parts at this location
7 Person who named cells 4 DNA is found here
9 Surrounds and protects plant cells 5 Semi-permeable membrane surrounding the cell
12 Transports and repackages 8 Chemical process thhat converts sunlight to sugar
14 Powerhouse of the cell 9 Gives the cell support
15 Heredity material found in the cell 10 Does not contain a cell wall
16 A cell without a nucleus 11 Organelles located in this liquid
17 Sunlight is converted to sugar at this organelle 13 Stores food and water for the cell
18 Division of the nucleus
19 First to realize animals are made of cells
1

2 3 4 5

9 10 11

12

13

14

15

16

17

18

19

BIOLOGY 185
ER -membranous channels that process & modify proteins
Mind Map

ENDOMEMBRANE SYSTEM
Smooth ER
-no ribosomes Cell Structure
-various lipid synthesis
&
Rough ER Function
-protein synthesis

Golgi Apparatus
-stack of membranous saccules
-processing, packaging & distributing of proteins & lipids

Lysosome
-produced by golgi apparatus
-membranous sacs/vesicles containing digestive enzymes
-intracellular digestion:
“suicide cells” destroys bad cells by taking it in and self-destructing

-True Nucleus surrounded by nuclear envelope


- contains sacs inside that can move between cytoplasm and nucleoplasm
- chromatin (a complex of DNA and protein)
-when cell divides DNA is found in separate structures called chromosomes
-nucleolus (special region of chromatin) forms ribosomal RNA
-Both plant and animal cells

EUKARYOTIC CELL
Animal Cells
-centrioles only in animal cells, involved in the process of nuclear division

Plant Cells
-only have chloroplast
-It builds up carbohydrates and mitochondria breaks it down
-process of cellular respiration to build a supply of ATP(energy currency of cells)
-uses molecules that mitochondira gives off
-mainly CO2 and H20 as raw material for photosynthesis which uses energy from the sun

-inner membrane (cristae), great surface area for proteins & production of ATP
Mitochondria
-matrix contains DNA, ribosomes & enzymes for breaking down carbohydrates

-has plasma membrane, cell wall, ribosomes and DNA


-does not have nucleus PROKARYOTIC CELL
does not have membrane-bounded organelles

-cells need constant energy, only form it can use: ATP (adenosine triphosphate)
ENERGY

Photosynthesis & Cellular Respiration

-ATP uses energy and becomes ADP


-ADP is converted by mitochondria to form ATP

186 CLASS - IX
CELL (THE FUNDAMENTAL UNIT OF LIFE)

1) Cells are the basic living unit, structure determines function in organisms
2) All organisms are cellular, composed of one or more cells
3) All cells come from pre-existing cells
CELL THEORY

Changed the way humans thought about living things


Simplified with -resolution: ability to distinguish between 2 objects as separate parts
the Microscope -light microscopes can see living organisms
-use light rays, longer wave lengths: 200 x 10-6 mm

size of cells
-small cell=more surface area/volume
-large cell=less surface area/volume
-nutrients & waste enters & leaves the cell at it’s surface
-the amount of the surface affects the ability to get material in & out of the cell
-surface area of a large cell actually decreases proportionately, cell size stays small

The Fluid-Mosiac Model


-structure refers to the fluidity of the phospholipid bilayers and pattern of
proteins in the membrane
-consist of lipid bilayer
-hydrophilic heads & hydrophobic tails
-selectively permeable: allows only some molecules through
-Proteins make up the mosiac part
PLASMA MEMBRANE

Diffusion
- lipid-soluble compounds, gas, water can simply move across the membrane
-no energy required from high to low concentrations

Osmosis
- water moving across membrane
-tonicity: the amount of dissolved solute particles
-isotonic solution: cell neither gains or lose mass (water)
-hypotonic solution: cell gains mass (water)
-lysis: cell overflows with water, may burst
-hypertonic solution: cell loses mass (water)
-crenation: cell shrivels up in a hypertonic solution

-large substances can exit the cell


Exocytosis
-secretion: vesicles (often from golgi) fuse with plasma membrane

-large substances can enter the cell


Endocytosis
-secretion: vesicles (often from golgi) fuse with plasma membrane

BIOLOGY 187
Do at Class!
I. Choose the correct answers: 10. The membrane that surrounds the vacuole is .........
(a) Tonoplast (b) Plasma membrane
1. Which of the following organelles is smallest in
(c) Cell wall (d) Nuclear membrane
size?
(a) Ribosome (b) Mitochondrial 11. The organelle involved in cell secretion is ..............
(c) Chloroplast (d) Lysosome (a) Plastids (b) ER
2. 
Which of the following organelles have double (c) Golgi bodies (d) Nucleolus
membrane? 12. Plasmolysis occurs due to ................
(a) Chloroplast (b) Ribosome (a) Diffusion (b) Endosmosis
(c) Lysosome (d) Vacuole (c) Exosmosis (d) Absorption
3. Which of the following organelle is not present in
13. The solution that has higher water concentration
an animal cell?
than the cell is known as ................
(a) Ribosome (b) Plastid
(a) Hypertonic (b) Hypotonic
(c) Mitochondrial (d) Nucleus
(c) Isotonic (d) None of these
4. Plastid that is colourless is ................
14. Cell nucleus was discovered by ................
(a) Chromoplast (b) Leucoplast
(a) Robert Hooke (b) Robert Brown
(c) Chloroplast (d) Lysosome
(c) Virchow (d) Leuwenhoek
5. Plant cell wall is mainly composed of ................
15. 
Which process requires the energy provided by
(a) Cellulose (b) Lipid ATP?
(c) Protein (d) Sugar
(a) Osmosis (b) Diffusion
6. 
The infoldings of the inner membrane of (c) Active transport (d) Plasmolysis
mitochondria are known as ................
16. 
Cellular respiration is related to ................ as
(a) Stroma (b) Grana ................ is related to chloroplasts.
(c) Cristae (d) Oxysome
(a) Mitochondria, Light
7. The site of aerobic respiration, in an animal cell, (b) Mitochondria, Photosynthesis
is................ (c) Chloroplast, Light
(a) Ribosome (b) Mitochondrial (d) Chloroplast, Photosynthesis
(c) Chloroplast (d) Nucleus
17. Lipid molecules in the cell are synthesised by
8. Ribosomes are the site of ................ (a) SER (b) RER
(a) Protein synthesis (b) Lipid synthesis (c) Golgi bodies (d) Ribosomes
(c) Respiration (d) Photosynthesis
18. A cell that contains a large central vacuole is
9. Which of the following statements is correct? (a) Plant cell (b) Animal cell
(a) Prokaryotic cells are surrounded by a cell (c) Bacterial cell (d) Yeast cell
membrane
19. Old organelles, viruses and bacteria that a cell can
(b) Prokaryotic cells have a nucleus ingest are broken down in
(c) Eukaryotic cells have genetic information (a) Ribosomes (b) Lysosomes
(d) Eukaryotic cells have membrane-bound (c) SER (d) RER
organelles
188 CLASS - IX
CELL (THE FUNDAMENTAL UNIT OF LIFE)

20. 
A slide of human cheek cell is stained with 28. Who proposed the cell theory?
methylene blue and mounted in glycerine. Which
(a) Singer and Nicholsen
of the following cellular organelle would you be
(b) Schwann and schleiden
able to see under a microscope?
(c) Hook and Brown
(a) Plasma membrane (b) Cell wall (d) Robertson
(c) Mitochondrial d Lysosome
29. Who proposed that new cells arise through cell
21. 
A cell “X” contains a cell wall, large central division of pre-existing cells?
vacuole and a nucleus at the periphery. The cell
(a) Robert Hook (b) Rudolf Virchow
“X” is ...................
(c) Robert Brown (d) Singer
(a) Plant cell (b) Animal cell
(c) Bacterial cell (d) Prokaryotic cell 30. It is the Smallest Cell ................

22. In human cheek cells, the nucleus is located at the (a) Bacteria (b) Mycoplasm
....................... (c) Yeast (d) Blue green algae

(a) Center of the cell 31. Prokaryotic cells have which architectural regions?
(b) The left side of the cell (a) Cell (b) Appendages
(c) The right side of the cell (c) Nucleus (d) a-b-c,all
(d) None of these.
32. The cells involved in large amount of lipid synthesis,
23. 
Which term is used to refer the process of do not possess this organelle on Endoplasmic
absorption of water by raisins from kheer? reticulum.
(a) Exosmosis (b) Endosmosis (a) Mitochondrion (b) Ribosomes
(c) Diffusion (d) Imbibition (c) Golgi apparatus (d) lysosome
24. 
Raisins soaked in high concentrated solution of
33. The materials essential for dark reaction are located
sugar ................ i .................The process involved
in
is known as ................ ii .................
(a) Circular-DNA (b) Thylakoids
(a) i- shrinks, ii- endosmosis
(c) Stroma (d) Ribosomes
(b) i- swells, ii- Exosmosis
(c) i- shrinks, ii- exosmosis 34. Microfilaments are made up of
(d) i- swells, ii- endosmosis
(a) Fat (b) Protein
25. When raisins are kept in water, the water moves (c) Carbohydrates (d) Nucleic acid
................ i ................ the raisins. This makes the
35. It possess Flagell(a)
raisins to ................ ii .................
(a) Paramoecium (b) Euglena
(a) i- inside, ii- swell (b) i- inside, ii- shrink
(c) Amoeba (d) Yeast
(c) i- outside, ii- swell (d) i-outside, ii- shrink
26. 
What will happen, a when a human RBC is a 36. In human which cell lacks nucleus.
placed in a hypotonic environment? (a) Lymphocyte (b) RBC
(a) It undergoes plasmolysis (c) Monocytes (d) Neutrophils
(b) It undergoes turgidity 37. No membrane surrounds in this organelle.
(c) It is at equilibrium
(d) None of these (a) Lysosome (b) Nucleolus
(c) Golgi body (d) Nucleus
27. 
It is responsible for beginning of the life of
organisms. 38. It shows presence of Nucleoid.

(a) Tissue (b) Zygote (a) Plant cell (b) Bacteria


(c) Cell (d) Embryonic layer (c) Animal cell (d) Virus

BIOLOGY 189
39. 
The cell wall of fungi is made up of which 49. Which organelle is responsible for degradation of
substance? worn out cells?
(a) Starch (b) Chitin (a) Lysosome (b) Golgi apparatus
(c) Cellulose (d) Pectin (c) Vacuoles (d) Endoplasmic Reticulum

40. 
Which organelle is not considered as a part of 50. Which plastids possess chlorophyll pigments?
Endomembrane system? (a) Chloroplast (b) Xanthophyllus
(a) Vacuole (b) Chloroplast (c) Anthrocyanin (d) Carotene
(c) Endoplasmic reticulum (d) Lyso some
51. How many grana present in one chloroplast?
41. Which is the example of unicellular organism? (a) 40-60 (b) 42-47
(a) Chlamydomonas (b) Spirogyra (c) 60-80 (d) 02-100
(c) Mushroom (d) Chiton 52. Each granum possesses how many thylakoids?
42. Who mentioned that cells had a thin layer around (a) 02-100 (b) 90-93
them? (c) 19-89 (d) 19-38
(a) Schwann (b) Virchow 53. Various colours in flower fruit and seeds are due to
(c) Schleiden (d) Robert Hook presence of which pigment?
43. Who mention that the presence of a cell wall is a (a) Anthocyanin (b) Chlorophyll
unique character of the plant cell? (c) Chloroplast (d) a,b,c-all
(a) Schwann (b) Virchow 54. Which organelle possess hydrolase enzyme?
(c) Schleiden (d) Robert Brown
(a) Lysosome (b) Golgi apparatus
44. Which organelles are found only in animal cell? (c) Mitochondria (d) Chloroplast
(a) Centriole (b) Mitochondria 55. 
The number of mitochondria per cell depends
(c) Golgi apparatus (d) Chloroplast upon the...

45. Which is biggest animal cell? (a) Physiological activity of the cell
(b) Types of cell
(a) Ostrich’s egg (b) Hen’s egg (c) Shape of cell
(c) PPLO (d) Mycoplasma (d) Size of cell
46. What is the function of SER? 56. How many basic shapes of Bacteria are there?
(a) Synthesis of Steroid hormone (a) 4 (b) 6
(b) Synthesis of protein (c) 9 (d) 1
(c) Synthesis of enzyme
(d) a,b,c,all II. Give one line answers for the following:

47. What is produced when vesicle are separated from 1. Are any bacteria made of more than one cell?
golgi body?
2. Are there living organisms without cells?
(a) Lysosome (b) Vacuoles
3. Which is the smallest cell in the world?
(c) Ribosomes (d) Chloroplast
4. What is the life span of sperm cell?
48. The area the cytoplasm without any cytoplasm are
called as... 5. 
Are membranes only present as the outside of
(a) Vacuoles (b) Chloroplast cells?
(c) Cytoplasmic Gap (d) Mitochondria 6. 
Which type of cell evolved first, the eukaryotic
cell or the prokaryotic cell?
190 CLASS - IX
CELL (THE FUNDAMENTAL UNIT OF LIFE)

7. What is the plasma membrane of the cell? What III. Fill in the blanks:
are its main functions?
1. The ..................... is the semiliquid portion of the
8. 
What chemical substances compose the plasma cell in which the cell parts are located.
membrane? 2. The ..................... is referred to as the headquarters
of the cell operations.
9. What is the difference between a plasma membrane
and a cell wall? 3. The structure found in plant cells, but not animal
cells, that carries out the process of photosynthesis
10. What are the main respective components of cell is the ......................
walls in bacteria, protists, fungi and plants? 4. Proteins are manufactured by the ......................
11. Do the cells of bacteria have a nucleus? 5. The structure that surrounds the cell and regulates
what enters and leaves the cell is the ......................
12. 
What substances is chromatin made up of?
What is the difference between chromatin and a 6. 
Nickname the “powerhouse of the cell ”, the
chromosome? ..................... are involved in energy production
for the cell.
13. What is the fluid that fills the nucleus called? 7. 
The ..................... surrounds the nucleus and
14. What are cytoplasmic inclusions? controls what enters & leaves it.
8. ..................... are structures that contain digestive
15. Where in the cell can ribosomes be found? What enzymes.
is the main biological function of ribosomes?
9. 
In addition to the cell membrane, plant cells
16. What is the difference between the smooth and also have a ..................... that serves to provide
rough endoplasmic reticulum? strength and support to the cell.

17. A net-like membrane complex of superposed flat 10. 


Storage chambers within the cell are called
saccules with vesicles detaching from its extremities ......................
seen is observed during electron microscopy. 11. Found in animal cells, the ..................... plays a
What is the observed structure called? What is its role in cell division.
biological function? 12. 
The cell structure that prepares and packages
protein either for use within the cell or for shipment
18. Which organelle of the cell structure is responsible
out of the cell is the ......................
for intracellular digestion? What is the chemical
content of those organelles? 13. Located within the nucleus, the ..................... is
involved in making ribosomes for the cell.
19. Why are lysosomes known as “the cleaners” of cell
14. The cell structure that receives, packages, and ships
waste?
materials is called ......................
20. Why can mitochondria be considered the “power 15. 
Another name for the DNA material located
plants” of aerobic cells? within the nucleus of the cell is ......................
21. What is the molecule responsible for the absorption 16. The cell structures that makes lipids, breaks down
of light energy during photosynthesis? Where is glycogen & detoxifies the cell ......................
that molecule located in photosynthetic cells? 17. 
The organelle which is associated with protein
synthesis......................
22. 
What colors (of the electromagnetic spectrum)
18. The new cell that arise after cell division are called
are absorbed by plants? What would happen to
..................... cells.
photosynthesis if the green light waves that reach a
plant were blocked? 19. The plastid that help prepare food for the plant are
......................
23. 
What are plant cell vacuoles? What are their
20. ..................... passing hereditary characters to the
functions? What is the covering membrane of
next generation.
vacuoles called?
BIOLOGY 191
21. ..................... giving rigidity to the cell. 23. Several organ systems together form a .............
22. T
 he organelle storing wastes in the plant cell is ...... organism.
...................... 24. ..................... destroy old and worn out cells.
25. The longest cell in human body is ....................

IV. Complete the following table by writing the name of the cellullar organelle in the right hand column
that matches the structure/function in the left hand column

Structure/Function Cellular organelles

Stores material within the cell

Closely stacked, flattened sacs (plants only)

The sites of protein synthesis

Transports materials within the cell

The region inside the cell except for the nucleus


Organelle that manages or controls all the cell functions in a eukaryotic
cell
Contains chlorophyll, a green pigment that traps energy from sunlight and
gives plants their green color
Digests excess or worn-out cell parts, food particles and invading viruses
or bacteria
Small bumps located on portions of the endoplasmic reticulum

Provides temporary storage of food, enzymes and waste products


Firm, protective structure that gives the cell its shape in plants, fungi, most
bacteria and some protests
Produces a usable form of energy for the cell

Packages proteins for transport out of the cell

Everything inside the cell including the nucleus

Site where ribosomes are made

The membrane surrounding the cell

Provides support for the cell, has two “subparts”

Name for the collection of DNA in the nucleus of eukaryotic cells

Consist of hollow tubes which provide support for the cell

Small hair-like structures used for movement or sensing things

Composed of a phospholipid bilayer

Longer whip-like structures used for movement

192 CLASS - IX
CELL (THE FUNDAMENTAL UNIT OF LIFE)

V. Put a check in the appropriate column(s) to indicate whether the following organelles are found in
plant cells, animal celss or both.

Organelle Plant Cells Animal Cells


Cell Wall
Vesicle
Chloroplast
Chromatin
Cytoplasm
Cytoskeleton
Endoplasmic reticulum
Golgi apparatus
Lysosome
Mitochondria
Nucleolus
Nucleus
Plasma membrane
Central vacuole
Ribosome
Vacuole

VI. Label the locations of the following organelles on the diagram of below:

Fluid mosaic model of plasma membrane Animal Cell


A. ..................................
A
E B. ..................................
K
B J
A C. ..................................
C I
F
B D. ..................................
G H
D E. ..................................
H C
I F. ...................................
G
F G. ..................................
A. ................................ B. ............................... D
H. ..................................
C. ................................ D. ...............................
E. ................................ F. ............................... I. ....................................
E
G. ............................... H. .............................. J. ....................................
I. .................................... K. ..................................
D
Chloroplast A. .............................. A E
F A. ...................................
B
A B B. ..............................
B. ...................................
C C. .............................
C. ...................................
D. .............................
D. ...................................
D
E. ............................. C
E. ...................................
G F E F. ..............................
Prokaryotic cell F. ...................................

BIOLOGY 193
Do at Home!
I. Choose the correct answer: 12. Which of the following imparts colours to flower
to attract insects for pollination?
1. The power house of cell is called ....................
(a) Leucoplast (b) Plastid
(a) Cell wall (b) Mitochondria
(c) Chloroplast (d) Chromoplast
(c) Ribosomes (d) Nucleus
13. Ribosomes are the site of .................... .
2. The kitchen of the cell is called ....................
(a) Photosynthesis (b) Respiration
(a) Cell wall (b) Nucleus (c) Protein synthesis (d) Absorption
(c) Vacuoles (d) Chloroplasts
14. What are the two main categories of cells?
3. The functional unit of life is called ....................
(a) Prokaryotic and Eukaryotic
(a) Cell (b) Egg (b) Animal and Plants
(c) Nucleus (d) None of these (c) Eukaryotic and Plants
(d) Prokaryotic and Animal
4. Chloroplast is found in ....................
15. 
Eukaryotic cells have A. Distinct nucleus B.
(a) Plant cell only (b) Animal cell only
Nucleolus C. nuclear membrane D. Nucleoid
(c) Both of these (d) None of these
(a) A, B and C (b) All of these
5. The control unit of cell is .................... (c) B and C (d) A, B and D
(a) Nucleus (b) Cell wall 16. 
You will not find a cell wall in which of these
(c) Cytoplasm (d) All of these kinds of organisms?
6. Single celled organisms are called .................... (a) Fungi (b) Animal
(a) Unicellular (b) Multi-cellular (c) Plants (d) All of the above
(c) Both of these (d) None of these 17. The energy currency of cell is?
7. Tissue is a .................... (a) GTP (b) ADP
(c) ATP (d) AMP
(a) Group of organs (b) Group of cells
(c) Group of tissues (d) Group of organisms 18. 
Diffusion is the movement of particles from
.................... .
8. Cell is discovered by ....................
(a) An area of equilibrium to an area of high
(a) Robert Brown (b) Robert Hooke
concentration
(c) John Mendal (d) Charse Darwin
(b) An area of high concentration to an area of low
9. Colourless plastids are known as .................... concentration
(a) Chromoplasts (b) Chloroplasts (c) An area of low concentration to an area of high
(c) Leucoplasts (d) Protoplast concentration
10. 
Animal cell lacking nuclei would also lack in (d) All of the above
.................... 19. ATP is .................... .
(a) Ribosome (b) Lysosome (a) Nucleotile (b) Nucleoside
(c) Endoplasmic reticulum (d) Chromosome (c) Energy curreney (d) All of the above

11. What is the full from of ATP? 20. This is a sac-like structure that contains digestive
enzymes. It may digest waste materials or even
(a) Adenosine tetra phosphate
dead cells.
(b) Adeno tri phosphate
(c) Adeno tri phosphorus (a) ER (b) Lysosome
(d) Adenosine tri phosphate (c) Vacuole (d) Vesicle

194 CLASS - IX
CELL (THE FUNDAMENTAL UNIT OF LIFE)

21. 
The control center or brain of the cell is the 31. Plant cell wall
.................... .
(a) Provide rigidity to the cell
(a) Nucleus (b) Mitochondria (b) Maintains cell shape
(c) Cell membrane (d) Endoplasmic reticulum (c) Prevents bursting of cell
(d) All a, b and c.
22. 
Plant cells have all of the following except
.................... . 32. “Proteins icebergs in a sea of lipids” is stated by
.................. .
(a) Cell wall (b) Nucleus
(a) Lamellar Model
(c) Chloroplast (d) Centrioles
(b) Unit-membrane Model
23. This is the jelly-like substance found in all cells (c) Fluid-Mosaic model
.................... . (d) Micellar Model

(a) Centrioles (b) Cytoskeleton 33. Lysosomes arise from .................. .


(c) Chromosomes (d) Cytoplasm (a) Nucleus (b) Endoplasmic reticulum
24. 
Which of the following is associated with the (c) Golgi apparatus (d) Cell membrane
structure of Golgi complex .................. . 34. What are the two faces of the Golgi body?
(a) Cristae (b) Cisternae
(a) Funny face and goofy face
(c) Tubulin (d) Actin
(b) Coated face and non-coated face
25. 
The membrane around the vacuole is called (c) Saving face and loosing face
.................. . (d) Cis face and Trans face
(a) Cytoplast (b) Tonoplast
(c) Amyloplast (d) Elaioplast 35. 
The matrix surrounding the grana in the inner
membrane of chloroplasts is .................. .
26. 
The passage of water from a region of higher
(a) Cytosol (b) Frets
water concentration through a semi–permeable
(c) Stroma (d) Inter-granal lamellae
membrane to region of low water concentration is
.................. . 36. Infoldings of inner membrane in mitochondria are
(a) Diffusion (b) None of these called ..................
(c) Both a and b (d) osmosis (a) Grana (b) Thyallkoids
27. The smooth ER is especially abundant in cells that (c) Cristae (d) Frets
synthesize extensive amounts of .................. . 37. The outer most layer of cell wall is .................. .
(a) Toxins (b) Proteins (a) Primary wall (b) Secondary wall
(c) Enzymes (d) lipids (c) Middle lamella (d) Plasma membrane
28. Ribosomes are made up of .................. subunits.
38. According to mosaic model by Singer and Nicholson
(a) 0 (b) 1 plasma membrane is composed of .................. .
(c) 2 (d) 3
(a) Phospholipids (b) Extrinsic proteins
29. The rough ER is so named because it has abundance (c) Intrinsic proteins (d) All of these
of .................. on it.
39. 
Which of the following correctly matches an
(a) Mitochondria (b) Lysosomes organelle with its function?
(c) Golgi bodies (d) Ribosomes
(a) Mitochondrion . . . photosynthesis
30. Within chloroplasts, light is captured by .................. (b) Nucleus . . . cellular respiration
(a) Grana with cisternae (b) Thylakoids membranes (c) Ribosome . . . manufacture of lipids
(c) Stroma (d) Stromal lamella (d) Central vacuole . . . storage

BIOLOGY 195
40. 
Plant cells contain the following 3 things not in a constantly changing mosaic pattern-often
found in animal cells .................., .................. referred to as the fluid-mosaic model of membrane
and ................... structure.
(a) Plastids / Chlorophyll / Membrane 12. The jellylike matrix within a cell is known as the
(b) Chloroplast / Cell wall / Golgi body cytoskeleton.
(c) Plastids / Cell wall / Chlorophyll 13. Mitochondria may be able to reproduce themselves,
(d) Mitochondria / Cell wall / ribosomes especially in cells that require greater than normal
energy outputs.
41 
The history of the cell began in 1665 with the
publication of Micrographia in London by 14. Apoptosis refers to cell death during which the cell
................... swells, ruptures its membranes, and burst.

(a) Robert Hooke (b) Robert Brown 15. Many more mitochondria are found in the ovum
(c) Strasburger (d) Dujardin (egg cell) compared to sperm; and contain a more
primitive form of DNA molecules that can transmit
42. Which of the following cellular part possess a genetic defects from mother to child.
double membrane? 16. 
The phenomenon of “tolerance” to certain
(a) Nucleus (b) Chloroplast substances such as alcohol and certain drugs may
(c) Mitochondrion (d) All of the above be accompanied by increased amounts of the rough
endoplasmic reticulum, particularly in liver cells.
43. Centriole is associated with ....................
17. Cells were first observed in cork by Robert Hooke
(a) Cell division (b) Respiration in 1665.
(c) DNA synthesis (d) DNA synthesis
18. Cells exhibit variety of shapes and sizes.
44. A cell has a nucleus, is complex and large, and has 19. Plant cell has a big central vacuole unlike a number
membrane bound organelles. what type of cell is of small vacuoles in animal cells.
this?
III. Comprehension Type:
(a) Bacteria
(b) Prokaryotic Read the passage below. Then answer the questions that
(c) There is not enough information follow.
(d) Eukaryotic
The first cells to appear on Earth were prokaryotic
II. True or False: cells. A prokaryote is an organism made of a single
prokaryotic cell. The earliest prokaryotes may have
1. Cells are invisible to the naked human eye.
arisen more than 2.5 billion years ago. Bacteria are
2. Animal cells have a cell wall but plant cells do not. prokaryotes. They are very small cells with a simple
3. Protoplasm includes nucleoplasm & cytoplasm. structure. Prokaryotes do not have a nucleus. This means
that their DNA is not enclosed in a membrane inside
4. Human cheek cells are irregular in shape.
the cell. Instead, prokaryotes have a single loop of DNA
5. Cell wall is non-living & composed of cellulose. that floats in the cell’s cytoplasm. Protein-making bodies
6. Unicellular organisms have one-celled body. called ribosomes also form part of the cytoplasm. Like all
7. Muscle cells are branched structures. cells, prokaryotes have a cell membrane. All prokaryotes
also have a cell wall surrounding the cell membrane.
8.  The basic living structure of an organism is an The cell wall helps provide support and protection for
organ. the cell. Some prokaryotes are enclosed by an additional
9. Amoeba has irregular shape. layer. This layer is called the capsule. The capsule has a
10. The cell membrane is also known as the plasma sticky surface area, so it allows prokaryotes to cling to
membrane. surfaces, such as your skin and your teeth. Prokaryotes
produce chemicals that protect their environment
11. The membrane is not solid, the phospholipid
from competition. In order to reduce the number of
and protein molecules are free to move laterally

196 CLASS - IX
CELL (THE FUNDAMENTAL UNIT OF LIFE)

competitors for resources, a prokaryote produces a 5. 


Which organelle contains the cell’s DNA in a
chemical that harms other strains of prokaryotes but eukaryotic cell?
not itself. We use these chemicals as antibiotics to
(a) Mitochondrion (b) Lysosome
treat certain, primarily bacterial, infections. Although
(c) Ribosome (d) Nucleus
effective against some bacteria, these antibiotics are not
effective against all prokaryotes, particularly the type of 6. Which of the following statements is True?
prokaryotes that produced the chemical.
(a) Prokaryotic cells are bigger than Eukaryotic cells
Eukaryotic cells are more complex than prokaryotic (b) Eukaryotic cells do not have a nucleus,
cells. They all have a cell membrane, ribosomes, and Prokaryotic cells do
DNA as prokaryotic cells do. However, the DNA of (c) Prokaryotic cells came before Eukaryotic cells
eukaryotic cells does not float freely in the cytoplasm.
(d) Eukaryotic cells are simple, Prokaryotic cells are
Instead, it is found in the nucleus, an internal
complex
compartment bound by a cell membrane. The nucleus
is one kind of organelle found in eukaryotic cells. 7. A cell has circular DNA, is small and simple, and
Organelles are structures that perform specific functions. no nucleus or membrane bound organelles. What
Most organelles are surrounded by a membrane. Some type of cell is this?
organelles have membranes that form channels which
(a) Eukaryotic (b) Plant
help transport substances from one part of the cell to
(c) Prokaryotic (d) Animal
another part of the cell.
8. What type of cell is a bacteria cell?
Eukaryotes are organisms made of one or more
eukaryotic cells. The earliest eukaryotes, like the first (a) Plant Cell (b) Eukaryotic Cell
prokaryotes, were single-celled organisms. They arose (c) Prokaryotic Cell (d) Animal Cell
about 1 billion years later than the earliest prokaryotes.
9. A scientist finds a cell that is very old and very
Later, multicellular eukaryotes arose. Every type
simple. This cell is probably a ................. .
of multicellular organism that exists is made up of
eukaryotic cells. (a) Prokaryotic cell (b) A plant cell
(c) Eukaryotic cell (d) A new type of cell
1. 
From which type of cells did multicellular
organisms arise? 10. Ribosomes help in formation of .................... .

(a) Prokaryotic cells (a) Fat (b) Protein


(b) Prokaryotic cells with a capsule (c) Vitamin (d) Glucose
(c) Eukaryotic cells
11. Most organelles in a eukaryotic cell are found in
(d) both prokaryotic and eukaryotic cells
the
2. What do prokaryotic cells and eukaryotic cells have
(a) Cell wall (b) Cytoplasm
in common?
(c) Nucleus (d) Capsule
(a) The both have a nucleus
(b) They both have an endoplasmic reticulum 12. 
Which one of the following is not found in
(c) They both have bacterial flagellum prokaryotic cells?

3. Prokaryotic cells do not have a nucleus. What do (a) Ribosomes


they have in its place? (b) A membrane-bound nucleus
(a) Nucleolus (b) Nucleoid region (c) A cell wall
(c) Deoxyribonucleic acid (d) Chromatin (d) A capsule
4. Which type of cell has membrane bound organelles? 13. In eukaryotic cells, internal membranes ..................
(a) Prokaryotic cells (b) Plant cells (a) Provide additional area where many metabolic
(c) Bacteria cells (d) Eukaryotic cells processes occur.

BIOLOGY 197
(b) Form membranous compartments called IV. Statement Type
organelles. 1. Which of the following statement are correct ?
(c) Contain proteins essential for metabolic A. Chlorophyll is found in chloroplast.
processes. B. Nucleus is surrounded by single membrane
(d) Greatly increase a cell’s total membrane area. called nuclear membrane.
(e) All of the choices are correct. C. Plastids are coloured.
D. Chloroplast contains DNA.
14. What are archaea?
(a) Prokaryotes characterized as extremophiles that (a) B and D (b) All of these
share some bacterial and some eukaryotic traits. (c) B and C (d) A and D
(b) Organisms that are adapted to high temperature 2. Which of the following is incorrect for cell
environments, such as in volcanic springs. theory?
(c) Single-celled organisms that are killed by A.All organisms are made up of cells.
the application of antibiotics at certain B.All living organisms are made up of
concentrations. eukaryotic cell only.
(d) Bacteria-like organisms that can live only in C.Cells arises from pre-existing cells.
extreme salt environments. D.Outermost covering all living cells is cell wall.
15. Can mitochondria be alive when its kept out of (a) A and D (b) All of these
cell? (c) B and C (d) B and D
3. 
Which of the following statements is correct?
16. What is a prokaryote, and when did prokaryotes Please select all that apply.
arise? (a) Hooke was first to observe living cells and
17. Describe three main features inside a prokaryotic introduce the word cell to describe them
cell. (b) Brown, Schleiden and Schwann, all working
18. Describe the structures that form the outside of independently, were first to propose the cell
a prokaryotic cell. Tell whether each structure is theory.
common to all prokaryotes. (c) Remark was first to propose the tenet that ‘all
cells arise from other cells’.
19. What is a eukaryote, and when did eukaryotes first (d) Leeuwenhoek was the first to observe unicellular
arise? organisms in pond water.
20. What are the three main parts of a eukaryotic cell? 4. 
Which of the following statements are true about
21. Justify the assertion that life on earth could not Endoplasmic Reticulum?
exist without prokaryotes. (a) Smooth Endoplasmic Reticulum makes lipids.
(b) It is also called the control center of the cell.
22. 
What are the functions of antibiotics are in the
(C) It processes carbohydrates.
prokaryotes that produce them?
(D) It modifies chemicals that are toxic to the
23. Compare prokaryotes and eukaryotes by filling in cell.
the table below. (a) (a), (b) and (C) (b) (a), (C) and (D)
Prokaryote Eukaryote (c) Only (a) and (D) (d) All are correct.
Structure
(YES or NO) (YES or NO)
5. Which of the following statements are true about
Cell membrane
Eukaryotes?
DNA
(a) They are cells with a nucleus.
Nucleus (b) They are found both in humans and multicellular
Endoplasmic Reticulum organisms.
Golgi apparatus (C) Endoplasmic reticulum is present in
Cell wall Eukaryotes.
(D) They have chemically complexed cell wall.
24. W
 hich helps provide support and protection for
(a) (a), (b) and (C) (b) (a), (C) and (D)
the cell?
(c) (a), (b) and (D) (d) All are correct.
198 CLASS - IX
CELL (THE FUNDAMENTAL UNIT OF LIFE)

V. Subjective Questions: 17. Write a note on nucleus of a cell.


1. 
In 1665, Robert Hooke, an English scientist, 18. What is the function of Nucleus?
published his book Micrographic, in which he
described that pieces of cork viewed under a 19. Draw a neat diagram of a Nerve Cell.
microscope presented small cavities, similar to pores 20. Why Plant cells are more rigid than animal cells?
and filled with air. Based on knowledge discovered
later on, what do you think those cavities were 21. 
Why is the inner membrane of Mitochondria
composed of? What is the historical importance of folded?
this observation? 22. Why are lysosomes known as “suicide-bags” of a
2. What are the two main groups into which cells are cell?
classified?
23. Why does the skin of your finger shrink when you
3. 
If you provided with some vegetables to cook. wash clothes for a long time?
You generally add salt into the vegetables during
24. Why do plant cells possess large sized vacuole?
cooking process. After adding salt. Vegetables
release water. What mechanism is responsible for 25. How does amoeba obtain its food?
this?
26. How do substances like Carbon dioxide(CO2) and
4. Regarding the presence of the nucleus, what is the water (H2O) move in and out of the cell?
difference between animal and bacterial cells?
27. Give an example of Anucleate cell i.e. cell without
5. What is the name of the membrane that encloses nucleus.
the nucleus? Which component of cell structure is
contiguous to this membrane? 28. 
Is plasma membrane permeable or selectively
permeable?
6. 
What are the main structures of the cytoplasm
present in animal cells? 29. 
What would happen if the plasma membrane
ruptures or breaks down?
7. 
What are the morphological, chemical and
functional similarities and differences between 30. 
Why light microscope is called a compound
lysosomes and peroxisomes? microscope ?
8. What are the functions of the cytoskeleton? 31. If the organization of a cell is destroyed dueto some
9. What substances make up the nucleolus? Is there a physical or chemical influence, what will happen ?
membrane around the nucleolus? 32. Where are genes located ?
10. 
Bacteria do not have chloroplast but some 33. 
What is membrane biogenesis? How plasma
bacteria are photoautotrophic in which it perform membrane is formed during this process?
photosynthesis. Which part of bacterial cell
performs this? 34. Why are peroxisomes mostly found in kidney and
liver cells?
11. Name any cell organelle which is non membranous.
35. Fungi are unicellular or multicellular ? or both.
12. Why viruses are neither Prokaryotic nor Eukaryotic?
13. What is the difference between an animal cell and 36. Which phenomena is commonly referred as ‘cell
a plant cell? drinking’?

14. Why are chloroplast found only in plant cells? 37. Virus is an exception to cell theory. Comment on
this statement.
15. Cells are the basic structural and functional unit of
life.Explain. 38. What is ATP? where it is synthesized ?

16. 
What is the difference between eukaryotes and 39. Write a short note on plastids.
prokaryotes?
40. Write short note on glyoxisomes.
BIOLOGY 199

You might also like